Jo's Psych exam review

अब Quizwiz के साथ अपने होमवर्क और परीक्षाओं को एस करें!

Denial

Unconscious failure to acknowledge an event, thought, or feeling that is too painful for conscious awareness

17. A nursing instructor is teaching about suicide. Which student statement indicates that learning has occurred?

Fifty to eighty percent of all people who kill themselves have a history of a previous attempt.

A newly admitted client asks, "Why do we need a unit schedule? I'm not going to these groups. I'm here to get some rest." Which is the most appropriate nursing reply?

"Group therapy provides the opportunity to learn and practice new coping skills."

Manipulative Behavior

S/sx: makes unreasonable requests, divides staff, uses others' faults to own advantage, tries to make others feel guilty, makes personal approach to staff, frequently lies, frequently rationalize

Paroxetine

SSRI. May cause drowsiness, avoid alcohol, contact physician if leg, foot or hand swelling is experienced, may need 2 weeks to see improvement, shake well, do not take sooner than every 24 hours

Fluoxetine, Prozac

SSRI. Monitor for suicide. nausea, insomnia, fatigue, sleepiness, stop MAOI at least 14 days prior to starting. Take in morning or evening with or without food. do not stop abruptly.

What are the first line of drugs used for depression

SSRIs

Antidepressent Medications

SSRIs Tricyclics MAOIs Atypical Antideppresents SNRIs NDRIs

Obsessive-compulsive Personality

- Attempts to control self through the control of others or the environment. - Cold toward others - Perfectionist - Acts with blind conformity and obedience to rules - Preoccupied with lists, rules, details, and orders - Expresses disapproval of those who are different from their own

14. A new nursing graduate asks the psychiatric nurse manager how to best classify suicide. Which is the nurse managers best reply?

Suicide is not a diagnosis, disorder, or affliction. It is a behavior.

Name some SSRIs

Fluoxetine (Prozac) Citalopram (Celexa) Escitalopram (Lexapro) Sertraline (Zoloft) Paroxetine (Paxil)

SSRIs

Fluoxetine (Prozac) Paroxetine (Paxil) Escitalopram (Lexapro) Citalopram (Celexa) Sertraline (Zoloft) Fluvoxamine (Luvox) Vilazone (Viibryd)

Crisis Intervention

Focuses on resolving immediate crisis and can be used when the client is disoriented due to a panic state. Helps identify coping mechanisms and a support system. Goal is to return to pre-crisis level of functioning.

Which therapeutic communication technique is being used in this nurseclient interaction? Client: When I am anxious, the only thing that calms me down is alcohol. Nurse: Other than drinking, what alternatives have you explored to decrease anxiety?

Formulating a plan of action

Alcohol Deterrents

Disulfram (Antabuse) Acamprosate (Campral)

During a nurse-client interaction, which nursing statement may belittle the clients feelings and concerns?

Dont worry. Everything will be alright.

Alcoholism

Drinking causes problems in person's life, drinking increases due to problems, and a physical, psychological dependence develops, during counseling identify problems related to drinking, help person see/admit problem. establish control. AA, disulfiram, assist person to identify triggers.

SNRIs

Duloxetine (Cymbalta) Venlafaxine (Effexor) Desvenlafaxine (Pristiq)

A client who frequently exhibits angry outbursts is diagnosed with antisocial personality disorder. Which appropriate feedback should a nurse provide when this client experiences an angry outburst?

During group, you raised your voice, yelled at a peer, and slammed the door.

Binge Eating Disorder

Eat large amount of foods in short periods of time and report feelings of guilt and shame after binging. Do not purge after nor are they obese. Co-exists with other psychiatric disorders.

Major Depressive Disorder Characteristics

-Most common disorder leading to a disability -Characterized by lose of interest in activities that are normally pleasurable -Impaired social/occupational functioning for longer than 2 weeks -can be mild, moderate, severe -Can happen once (single episode) or multiple times (recurrent episodes) -Can be characterized by episodes of psychosis, catatonic, melancholic (sadness)

A client on an inpatient unit angrily states to a nurse, "Peter is not cleaning up after himself in the community bathroom. You need to address this problem." Which is the appropriate nursing response?

"I can see that you are angry. Let's discuss ways to approach Peter with your concerns."

Persistant Depressive Disorder (Dysthymia)

"down in the dumps" Feeling depressed for most of the day, for more days for at least 2 years No episodes of psychosis

Dependent Personality

- Expresses needs in a demanding way while professing independence and denying dependence - Passive and does not accept responsibility for behavior - Negative self-image/difficulty making decisions

Psychogenic Amnesia with Fugue

- Flight with loss of memory (person leaves home or work with inability to recall identity - Person may assume new identity - Excessive alcohol use can lead to fugue state - Rare

Schizotypal Personality

- Interpersonal conflicts - Socially isolated - Has eccentricities and odd beliefs

Paranoid Personality

- Mistrust in others - Projection (blame for own problems onto others) - Can have odd beliefs or magical thinking - Not in touch with reality - Can be hostile and use accusatory language that is reality-based

Narcissistic Personality

- Perceives self as all-powerful (arrogant) (preoccupied with power) - Critical to others - Needs attention and admiration

Dissociative Identity Disorder

- Presence of two or more distinct personalities - Personalities emerge during stress - Caused by physical, psychological, or sexual child abuse

Histrionic Personality

- Seeks attention by overreacting and exhibiting hyper-excitable emotions - Dramatic, attention-seeking - Angry outbursts - Uses physical appearance to draw attention to self

Antisocial Personality

- Shows aggressive acting out behavior pattern - Manipulative to meet own needs - Lacks conscious or remorse - Difficulty forming relationships

Avoidant Personality

- Socially inhibited - Hypersensitive to negative criticism and rejection - Longs for relationships - Feels inadequate

Schizoid Personality

- Socially/emotionally detached - Avoids relationships - Withdrawn and seclusive - Little expression, dull, humorless

Psychogenic Amnesia

- Sudden inability to recall extensive personal events - Occurs in a traumatic event (near death, natural disaster, intolerable life situation) - MOST COMMON

Depersonalization including Derealization

- Temporary loss of one's reality and ability to feel ad express emotion - Express detachment with regard to surroundings (others are unreal or visually distorted)

Borderline Personality

- Views others as either good or bad (splitting behavior) - Impulsive - Shows intense anger and has difficulty controlling anger - Makes suicidal gestures - Has disturbances with self-image and sexual, social, and occupational roles - Can become verbally abusive

Seasonal Affective Disorder

-depression in which people experience depressive episodes during specific times of the year. -The most common seasonal pattern is for depressive episodes in the fall or winter and diminish in the spring.

Disruptive Mood Dysregulation Disorder

-dx between ages 6-18 -frequent mood swings, lashing out, reacting in a way that is over the top for the situation -symptoms occur for more than 12 months

Vascular Depression

-seen in older adults -blood vessels become less flexible. They may harden and prevent blood from flowing normally to the body's organs, including the brain

Antidepressant precautions

-will take at least 4 weeks to work -don't discontinue abruptly -caution if pregnant -black box warning

A patient who lost his job last month has now been told that his wife wants a divorce. He says, I know I dont have much to offer a woman. She wants more than what I am, and now Im not even bringing home any money. Which nursing diagnosis is most appropriate? 1) Chronic Low Self-Esteem 2) Situational Low Self-Esteem 3) Disturbed Personal Identity 4) Disturbed Body Image

. ANS: 2 Situational Low Self-Esteem occurs when a person exhibits self-disapproval and negative self-evaluations as a specific reaction to loss or change (in this case of a job and a marriage). There are no data to indicate long-standing (chronic) Low Self-Esteem. This client has no defining characteristics for Disturbed Personal Identity, which is an inability to determine boundaries between self and others, nor of Disturbed Body Image. He does mention his appearance but does not focus on it in particular; it is only part of his overall dissatisfaction with himself

Which of the following is considered a strength of the nursing profession? 1) Biomedical focus 2) Psychosocial focus 3) Biopsychosocial focus 4) Physical focus

. ANS: 3 A strength of the nursing profession is the ability to go beyond the biomedical, psychosocial, or physical focus to care for the entire person. This approach focuses on the overall biopsychosocial well-being of the patient.

Which statement by the nurse is best when communicating with a patient with clinical depression? 1) Its a beautiful day today; youll feel better if you look out the window. 2) Youre having a bad day; Im sure youll feel better soon. 3) Life seems overwhelming at times; would you like to discuss how youre feeling? 4) You are very lucky to have such a supportive family.

. ANS: 3 When caring for a patient with depression, the nurse should encourage the patient to discuss his feelings. Its a beautiful day . . . and Youre having a bad day . . . offer false reassurance. It would not be therapeutic to say, You are very lucky . . .; that is offering a judgment.

A frail, elderly patient admitted with dehydration to a medical-surgical unit is exhibiting confusion, distractibility, memory loss, and irritability. What is most important for the nurse do? 1) Recognize these symptoms as signs of normal, physiologic aging. 2) Obtain a urine specimen before notifying the primary care provider. 3) Be sure she is placed in room occupied with another patient. 4) Interview the patient to screen for clinical depression.

. ANS: 4 Depression is often masked in older adults and expressed as physical and personality changes. Memory loss and confusion are also common symptoms of depression in older adults. Any one of the symptoms might occur as a result of physical illness, but the combination should prompt the nurse to suspect and screen for depression before exploring physiological causes for the symptom (as with a urine specimen). Placing the patient with another patient would be indicated for social isolation, which can be associated with depression; however, the nurse needs to screen for depression before looking for causes.

The nurse manager observes the response of several staff to change that occurs on the medical surgical unit. The staff are classified as "early adopters" if which behavioral patterns are observed? 1. the staff are open and receptive to new ideas and are sought out by others 2. the staff thrive on change and see it as an exciting adventure which can advance their careers 3. the staff are not the first to accept a change, nor are they the last 4. the staff are openly negative about changes and adopt them only after most others have done so

1 "Early adopters" are open and receptive to new ideas Peers tend to respect them and seek them out for information and advice about changes that are proposed or implemented

The nurse manager is hired for the pediatric unit. The nurse manager states in a meeting, "I run my unit w/a lot of control and make all the decisions." The staff recognizes the nurse manager is practicing which leadership style? 1. autocratic 2. laissez-faire 3. democratic 4. situational

1 An autocratic leader gives orders and makes decisions This style may be an efficient way to run things and works best in times of crisis Autocratic leadership usually stifles creativity and decreases motivation or staff

The nurse manager wants to implement a three-day workweek on the nursing schedule. The nurse manager states, "anyone who agrees to a three day schedule will receive an increase in the hourly rate of pay." The nurse manager is demonstrating which type of power? 1. reward power 2. coercive power 3. expert power 4. referent power

1 Basis of reward power is the ability to provide favors or promise money or other benefits A manager would reward staff w/promotions or extra time off

Which change strategy is *most* likely to be used by nurse managers who believe that people are rational and will act in ways that will be in their own best interest? 1. empirical-rational strategies 2. power-coercive strategies 3. normative-reeducative strategies 4. historical-futuristic strategies

1 Empirical-rational strategies are based on the idea that people are rational and will act in ways that they intellectually understand will be in their own best interest

The charge nurse is having difficulty making an appropriate assignment for the nursing team. Which assessment by the supervisor helps the charge nurse make the assignment for the day shift? 1. described the knowledge and skill level of each member of your team 2. do you know which assignment each staff member prefers? 3. how long has each staff member been employed on the unit? 4. do you know if any staff members are working overtime today?

1 Five rights of delegation include: right task, right circumstances, right person, right direction, right supervision It is important to consider the knowledge and abilities of each staff member Supervision consists of the initial direction (delegation) and periodic inspection (reassessment and evaluation) --> both elements must be present to ensure effectiveness in entrusting an element of client care to another staff member

The nursing team consists of two nurses, two LPN/LVNs, and two UAP. The nurse considers the assignment appropriate if an LPN/LVN is required to complete which task? 1. perform a sterile dressing change 2. obtain VS 3. stock supplies of syringes and dressings 4. transfer a stable client to x-ray

1 LPN/LVNs assist w/implementation of care, perform procedures, differentiate normal from abnormal, care for stable clients w/predictable outcomes, have knowledge of asepsis and dressing changes, and may administer medications, which varies w/educational background and state nurse practice acts

The nurse cares for a client w/a managed care plan of insurance. Which is correct about managed care? 1. the client may only use certain approved providers 2. the client has unlimited coverage for all services prescribed 3. the client may have services w/o prior authorization 4. the client may select any provider the client desires to provide care

1 Use of health care providers and services is limited to those w/in the group who agree to certain fee schedules Monitoring medical usage is critical in managed care

The psychiatric nurse is presented w/a group of clients in the ED. Which client requires *immediate* attention? 1. a young adult client who failed medical school and says, "my pain will be over soon" 2. an adult client who is unable to talk in front of other people due to symptoms of anxiety 3. a middle-age client who hears voices saying to harm others 4. a middle-age client who is anxious after witnessing a murder

1 This client is indicating thoughts of suicide and is at immediate risk for self-harm Nurse must determine if the client has the means to inflict self-harm Nurse should place the client on one-to-one observation and stay w/client to help control self-destructive impulses

The nurse plans assignments for the day after receiving the night shift's report. Which client does the nurse see *first*? 1. an adolescent client who took 100 mg methylphenidate and has a BP of 160/100 2. a young adult client who requires a metered-dose inhaler 3. a young adult client w/a short arm cast on the left arm 4. a middle-age client dx w/hypothyroidism requiring a TSH level

1 This is the most unstable client Methylphenidate is a CNS stimulant used for ADHD BP is elevated Assess the client for restlessness, dilated pupils, tremors, and possible tonic-clonic seizures

Elder Abuse

1-10% of the elder population is abused (often by the caregiver) Assessment: - Bruises on upper arms, broken bones (from being pushed and falling) - Dehydration, malnourishment - Overmedication - Poor hygiene, improper medical care - Feelings of hopelessness/helplessness - Injuries do not correlate with stated cause - Misuse of money by children or legal guardians Interventions: - Establish trust - Meet physical needs and treat wounds - Document factual, objective info about the condition of the client, injuries, and interaction with partner or family - Arrange for "respite care" for the caregiver - Arrange for visiting nurses, nutrition services, or adult day care if possible - Report suspected abuse to the authorities - Older adults often do not report abuse due to fear of abandonment and being put into a nursing home.

Which therapeutic communication technique should the nurse use when communicating with a client who is experiencing auditory hallucinations?

I understand that the voices seem real to you, but I do not hear any voices.

The nurse provides care for clients in a gynecological clinic. Which client does the nurse see *first*? 1. a middle-age client reporting a dry vaginal wall and painful intercourse 2. an adult client who had a hysterosalpingogram and is experiencing tachycardia and has a generalized rash 3. an adult client preparing for a cervical biopsy who reports feeling highly anxious 4. a young adult client scheduled for a Pap smear who reports heavy bleeding w/menstruation

2 A hysterosalpingogram is an x-ray of the cervix, uterus, and fallopian tubes performed after the injection of a contrast medium Tachycardia and rash indicates the client is having an allergic reaction and needs immediate attention Prior to any diagnostic test in which contrast medium is used, the nurse should assess for allergy to shellfish or iodine

A client in an outpatient psychiatric clinic tells the nurse, "I'm going to kill my spouse. That will make my life much better." Which mandate must the nurse follow to meet legal nursing responsibility in this scenario? 1. good samaritan 2. duty to warn 3. confidentiality 4. advocacy

2 Nurse must warn third parties of potential violence to them by a client This mandate supersedes client confidentiality and ethics

The nurse reviews a client's medical record. The nurse understands which document entry is an example of objective data? 1. client reports, "I feel so anxious" 2. skin of left leg is pale, cool, and dry 3. client reports pain in left leg which worsens w/movement 4. client is anxious and defensive

2 Objective data is information that is observable and measurable by HCPs It may add to or validate subjective data that has been given

The RN assess clients for the day shift. The nursing team includes one RN, two LPN/LVNs, and four UAP. The nurse determines assignments are appropriate if which client is assigned to the UAP? 1. the client dx w/Crohn disease requiring a sterile dressing change 2. the client dx w/chronic kidney failure requiring I&O 3. the client dx w/full thickness burns requiring IV morphine prior to a dressing change 4. the client dx w/cancer of the lung reporting a HA

2 UAPs assist w/direct client care activities (bathing, transferring, ambulating, feeding, toileting, obtaining VS, height and weight, I&O, housekeeping, transporting stable clients, and stocking supplies) Assign standard, unchanging procedures Delegation is the reassignment of responsibility for the performance of a job from one person to another --> responsibility for the task is transferred, but the accountability for the process or outcome of the task remains w/the delegator

The home health nurse is concerned about the large number of clients who are receiving home visits even though they no longer need skilled nursing or therapy services. The nurse discusses these concerns w/the management of the agency, but no changes are made. Which would protect the nurse from termination if the nurse reports the agency to regulatory authorities? 1. title VIII of the civil rights act of 1964 2. whistleblower laws 3. occupational health and safety act 4. family and medical leave act

2 Whistleblower laws are intended to prevent employers from taking retaliatory action against nurses such as suspension, demotion, harassment or discharge for reporting improper pt care or business practices Whistleblower laws vary from state to state and according to the subject matter

The nursing team consists of two nurses, two LPN/LVNs, and two UAP. Which client assigned to the LPN.LVN does the LPN/LVN question? 1. the infant client requiring a bath prior to discharge 2. the infant client w/a palpable olive-shaped mass in the epigastrium and frequent vomiting 3. the infant client requiring eye drops instilled 4. the infant client requiring a rectal temperature taken every 6 hrs

2 This indicates pyloric stenosis Client is not stable and requires further assessment by the nurse Assessment cannot be delegated to the LPN/LVN

The nurse team caring for client on the pediatric unit consists of two nurses, two LPN/LVNs, and two UAP. Which client does the nurse assign to the LPN/LVN? 1. the client admitted w/asthma 2. the client in a balanced suspension traction for a fractured left femur 3. the client admitted for intestinal bleeding 4. the client admitted for ingesting a grandparent's anti-hypertensive medication

2 This is a stable client w/a predictable outcome Caring for a client w/balanced suspension traction is a skill that may be performed by the LPN/LVN Nurse will assure that weights hang freely and frequently inspect the skin for areas of breakdown LPN/LVN assists w/the implementation of care, performs procedures, differentiates normal from abnormal, cares for stable clients w/predictable outcomes, has knowledge of asepsis and dressing changes, and may administer medications Medication administration varies w/educational background and state nurse practice acts

The nurse must use critical thinking skills while providing care for a client receiving a blood transfusion. Which is the *best* example of critical thinking in the care of this client? 1. the nurse reviews the procedure for blood administration 2. the nurse checks a unit of blood w/another nurse 3. the nurse evaluates changes in the client's VS during the procedure 4. the nurse documents the procedure in the medical record

3 Critical thinking skills involve complex thinking processes and cognitive activities used in problem solving and decision making Nurses who use the nursing process engage in critical thinking Nurse assesses the client's VS during a blood transfusion to determine normal vs. abnormal, expected vs. unexpected, to the plan of care, implement actions, and evaluate the client's response to those actions

The nurse moves to a different state and reviews the Nurse Practice Act (NPA) for the state. The nurse understands which governing body enforces the rules of the NPA? 1. american nursing association (ANA) 2. national council of state boards of nursing (NCSBN) 3. state board of nursing (BON) 4. state nursing association

3 Each state has a Board of Nursing (BON) that interprets and enforces the rules of the NPA An NPA is enacted by state legislation and its purpose is to govern and guide nursing practice w/in that state An NPA is actually a law and must be adhered to as law

The nurse overhears the unit secretary tell a joke w/sexual overtones to another staff member. The nurse is offended by the sexual nature and language of the joke. Which action does the nurse take *first*? 1. immediately document the incident 2. immediately report the incident to the supervisior 3. immediately inform the unit secretary that the nurse is offended 4. immediately file a sexual harassment complaint w/human resources

3 Nurse should inform the unit secretary clearly that the sexual nature of the joke is offensive Sexual harassment is prohibited by Title VII of the Civil Rights Act of 1964 and includes unwelcome sexual conduct such as pressure to participate in sexual activities, making sexual jokes, or sexual gestures

A nurse manager terminates a nurse for falsifying an entry in a medical record. Several months later, the manager receives a call from an agency asking for a reference for the terminated employee. Which is the nurse manager's *most* appropriate response to the potential employee? 1. I will have to consult an attorney before speaking w/you 2. the nurse did a good job and then resigned 3. that employee did work for me and was terminated for poor performance 4. I can't speak w/you about that employee

3 There are no federal laws that address what an employer can or can't say about a worker Many states have enacted legislation that gives employers a qualified immunity when providing information for a reference check These statues generally provide that an employer is immune from civil liability when it responds to a reference check in good faith Immunity is lost if it can be shown that the employer knowingly or recklessly provided false or misleading information or acted w/malicious intent Manager can freely acknowledge the employee's dates of employment, and that the employee was terminated Manager should reference the potential employer to the HR department for further information

Ten clients from a motor vehicle accident are transferred to the hospital. The nurse triages in the ED. Which client does the nurse see *first*? 1. a client w/exxhymosis and lacerations to the facial area 2. a client who reports chest tightness and pressure 3. a client w/a BP of 90/60 and apical P of 120 bpm 4. a client who reports dizziness and nervousness

3 This client is experiencing an actual problem VS indicate shock This is the most unstable client

The nurse sees clients in the adolescent psychiatric clinic. Which client does the nurse see *first*? 1. the school age client who reports impulsivity and poor attention span 2. the adolescent client who displays frequent loss of temper and argues w/teachers 3. the adolescent client who wants to be a model and only drinks water and eats vegetables 4. the adolescent client who bullies, threatens, and intimidates others and frequently initiates physical fights

3 This client is the most unstable w/actual behaviors that could cause physical harm Nurse should assess nutritional status and monitor for an eating disorder

The nurse from the pediatric unit is assigned to the psychiatric unit. Which client assignment is the most appropriate for the reassigned nurse? 1. a psychotic client who hears the devil saying "kill yourself" 2. a suicidal client who voices a plan to jump off a bridge today 3. a client dx w/depression w/a decreased appetite 4. an adolescent who has vomited every day for three months

3 This is the most stable client exhibiting predictable behavior r/t dx Nurse reassigned to an unfamiliar unit should be assigned stable clients w/predictable outcomes

The nurse understands which is primary focus of the NCLEX-RN? *select all that apply* 1. performance appraisal 2. evaluation of nursing schools 3. assessment of minimum competency 4. determination of ability of entry level nurse to perform safely 5. measurement of quality client care

3, 4 NCLEX-RN examination measures minimal competency to perform safe client care as a new nursing graduate at the generalist level --> it is a national US examination, adopted by all 50 states and the territories (except for Puerto Rico), and Canada Passing of the NCLEX-RN examination indicates that a new graduate nurse has shown the ability to provide safe client care at the generalist level

The nursing team consists of two nurses, one LPN/LVN, and two UAP. The nurse considers which assignment appropriate for the LPN/LVN to complete? *select all that apply* 1. obtain VS for the client immediately after ECT 2. assist the client w/bathing and feeding 3. administer a tube feeding for the client w/dysphagia 4. discharge the client dx w/multiple sclerosis 5. teach the client how to administer a subcutaneous injection 6. change the clean dressing for a client w/a venous ulcer

3, 6 The nurse will assign stable clients w/expected outcomes to the LPN/LVN

The nurse collapses minutes after putting on latex gloves, and the cardiac team successfully resuscitates the nurse. The nurse manager discusses that certain workers may be at greater risk for latex allergies than others. After reviewing the medical hx of the nurses on the unit, the nurse manager determines which nurse can safely use latex products? 1. the nurse w/a hx of allergies to pollen and grass 2. the nurse w/a hx of allergies to banana and kiwi 3. the nurse w/a hx of multiple surgeries 4. the nurse w/a hx of GI upset

4 A hx of GI upset is not an indication that a nurse is at risk for latex allergies

The nurse notices the client dx w/major depression crying in the day room. The nurse puts a hand over the client's shoulder and states, "let's talk about it." Which ethical principle describes the nurse's action? 1. autonomy 2. veracity 3. nonmaleficence 4. beneficence

4 Beneficence is an action that promotes good will This ethical principle requires the nurse to help clients meet all of their needs

The nurse evaluates clients in the GI clinic. Which client does the nurse see *first*? 1. a middle-age client dx w/irritable bowel syndrome reports cramping and loose stools 2. a young adult client reports not having a bowel movement in 2 days 3. a school-age client diagnosed w/gastroenteritis who had five diarrheal stools in the last 3 days 4. a newborn client experiencing projectile vomiting and irritability

4 Client's symptoms indicates pyloric stenosis Infant is at risk for fluid and electrolyte imbalance and requires immediate intervention

The nurse delegates tasks to staff on the night shift. Which task is appropriate for the nurse to delegate to the UAP? 1. begin a blood transfusion on a postoperative client 2. change the colostomy bag on a client w/an ileostomy 3. administer morning insulin to a client on a sliding scale 4. obtain a client's daily weight and shift I&O

4 This is a standard, unchanging procedure UAPs assist w/direct client care activities such as bathing, transferring, ambulating, feeding, toileting, obtaining VS, height and weight, I&O, housekeeping, transporting stable clients, and stocking supplies

The nurse begins employment in a hospital using the SOAP system of documentation. The nurse understands that SOAP is an acronym for which information? 1. strategic, operational, accountability, protocol 2. status, orders, actions, precautions 3. secure, orderly, appropriate, precise 4. subjective, objective, assessment, plan

4 SOAP documentation is problem-oriented documentation There is an interdisciplinary master problem list and all notes reflect back to an item on that list Notes are organized in a standard format: S (subjective data: what the client or family tells the nurse or other HCP); O (objective data: objective findings of the nurse or other HCP, obtained from observation, assessment including lab findings and examination); A (assessment data: assessment of the meaning of the subjective and objective data by the nurse or other HCP); P (plan of action: planned interventions to address the problem) Critical thinking is inherent in this documentation

A pt has just been admitted to the psych unit with a dx of MDD. Which behavioral manifestations would the nurse expect to assess (select all that apply) a. slumped posture b. delusional thinking c. feelings of despair d. feels best early in the morning and worse as the day progresses e. anorexia

A, B, C, E

The goal for cognitive therapy with depressed clients is to: a. identify and change dysfunctional patterns of thinking b. resolve the symptoms and initiate or restore adaptive family functioning c. alter NTs that are creating a depressed mood d. provide feedback from peers who are having similar experiences

A

a client with depression has just been px phenalzine (Nardil) she says to the nurse, "the dr says I need to watch my diet on this medication, what foods should I avoid?" What is the correct response by the nurse a. blue cheese, red wine, raisins b. black beans, garlic, pears c. pork, shellfish, egg yolks d. milk, peanuts, tomatoes

A

The nurse is caring for a newborn who was delivered from a woman who took benzodiazepines for anxiety during the last 2 months of her pregnancy after her husband was killed in war. What will the nurse assess for in this newborn? A) Newborn withdrawal syndrome B) Hepatic dysfunction C) Failure to thrive D) Learning deficiencies

A Feedback: Neonatal withdrawal syndrome may result in a baby born to a mother who was taking benzodiazepines in the final weeks of pregnancy. The neonate may be given very small doses of benzodiazepines that are withdrawn gradually to prevent symptoms. Hepatic dysfunction in the neonate is not associated with use of benzodiazepines. Failure to thrive and learning deficiencies would be long-term problems and are not assessed during the neonatal period.

The nurse is caring for a patient in a state of hypnosis, which means the patient is in what state? A) A state of extreme sedation in which the person no longer senses or reacts to incoming stimuli. B) A state of tranquility in which the person can be made to do whatever is suggested by others. C) A feeling of tension, nervousness, apprehension, or fear with high levels of awareness. D) A state in which the brain is no longer sending out signals to the body.

A Feedback: Hypnosis is an extreme state of sedation in which the person no longer senses or reacts to incoming stimuli. A state of tranquility is produced through minor tranquilizers by decreasing anxiety. Anxiety is a feeling of tension, nervousness, apprehension, or fear. Sedation is the loss of awareness and reaction to environmental stimuli, which may lead to drowsiness. The state of suggestibility often seen in television programs is not an appropriate definition of hypnosis. If the brain stopped sending signals, the patient would stop breathing and death would follow.

AIDS Dementia Complex

A 33-year-old man with HIV infection is brought in by his partner for evaluation of altered mental status. The partner has noticed waxing and waning periods of confusion throughout the day, difficulty in performing tasks such as balancing a checkbook, and deterioration of handwriting. The patient reports no fever or headache. What is the most likely diagnosis?

Hallucination

A profound distortion in a person's perception of reality, typically accompanied by a powerful sense of reality. Turn off TV, elevate HOB, invove in activity, offer reassurance

Passive Aggressive Behaviors

A psychological defense mechanism in which an individual expresses his aggression toward another person with repeated, passive failures to meet the other person's need.

Attention Deficit Hyperactivity Disorder (ADHD)

A psychological disorder marked by the appearance by age 7 of one or more of three key symptoms: extreme inattention, hyperactivity, and impulsivity.

Dissociative Identity Disorder (DID)

A rare dissociative disorder in which a person exhibits two or more distinct and alternating personalities. Also called multiple personality disorder.

The client is told by the HCP that the client's cancer is inoperable. The nurse enters the room a short time later and finds the client crying. Which action should the nurse take first? A. Acknowledge this is a sad time. B. Quietly leave the room. C. Call the chaplain or spiritual leader at the hospital. D. Stress what can be done in the time remaining.

A. Acknowledge this is a sad time Demonstrates the nurse's understanding and acceptance of grieving.

The patient is brought to the emergency department in respiratory arrest after overdosing on heroin. The person accompanying the patient says he has been using heroin for years. After being administered one dose of a narcotic antagonist, the patient begins to breathe spontaneously but remains nonresponsive to stimuli so another dose of narcotic antagonist is ordered. What symptoms would indicate the patient is experiencing acute narcotic abstinence syndrome? (Select all that apply.) A) Tachycardia B) Hypertension C) Vomiting D) Confusion E) Sedation

A, B, C Feedback: The most common adverse effect is an acute narcotic abstinence syndrome that is characterized by nausea, vomiting, sweating, tachycardia, hypertension, tremulousness, and feelings of anxiety. Confusion and sedation are not associated with acute narcotic abstinence syndrome.

The client who had a hysterectomy six months ago suddenly develops an intense fear of elevators. When the client approaches the building elevator, the client becomes panicky and cannot get on. The nurse knows this client's fear of elevators is caused by which occurrence? A. A projection of anxiety onto a neutral object. B. A common post-op phenomenon in females. C. An attempt to undo the traumatic hospital experience. D. A conversion reaction to emotional stress.

A. A projection of anxiety onto a neutral object. The client has developed a phobia in which anxiety is projected onto a neutral object, the elevator; the client has not worked through the response to surgery; avoidance behavior symbolizes the anxiety.

Which signs and symptoms does the nurse observe in the client who has recently taken heroin? A. Constricted pupil, depressed respirations B. Dilated pupil, increased respirations C. Vomiting and hypotension D. Agitation and tachycardia

A. Constricted pupil, depressed respirations Client who has recently taken heroin will have CNS-depressed effects of constricted pupils and slow, shallow breathing, drowsiness, slurred speech, initial euphoria followed by dysphoria.

The HCP prescribes lithium carbonate for a client. The nurse understands which medication is contraindicated for this client? A. Diuretics B. Monoamine oxidase inhibitors C. Tricyclic antidepressants D. Antibiotics

A. Diuretics Lithium causes sodium depletion; diuretics are contraindicated for clients on lithium.

The nurse admits the client with a diagnosis of schizophrenia to the unit. The client needs are best met by which action? A. Give the client a brief orientation and stay with the client for awhile. B. Offer the client a description of ward activities and introduce the client to other clients. C. Introduce the client to another client and ask the other client to give a short unit tour. D. Sit with the client in a quiet room and wait until the hallucinations stop.

A. Give the client a brief orientation and stay with the client for awhile. Since client has a reduced attention span and an inability to concentrate, a brief orientation is best; by staying with the client the nurse conveys an attitude of caring and protection.

The nurse cares for the client diagnosed with antisocial personality disorder. Which client statement best indicates improvement in the client's condition? A. I get into trouble because I don't think before I act. B. My parents have difficulty accepting my independence. C. I've spent very little time actually enjoying life. D. It's sad that others don't recognize my potential.

A. I get into trouble because I don't think before I act. Introspective remark that shows the client is beginning to recognize acting out of anxiety or tension without realizing the consequences of the actions.

The home care nurse makes an initial visit to the client diagnosed with a myocardial infarction. The client's spouse states the family is having difficulty coping with the client's "obsessive-compulsive" tendencies. Which client statement is consistent with obsessive-compulsive disorder? A. I have difficulty making decisions and adjusting to change. B. I am sure I am being followed by someone from work. C. All of my life I've had problems with being unkempt. D. I spend money excessively, which upsets my spouse.

A. I have difficulty making decisions and adjusting to change. Clients with obsessive-compulsive disorder have an extreme need to control and predict outcomes making decisions and change is anxiety-producing for these clients.

The nurse interacts with the client diagnosed with depression. The nurse expects the client to express which thought? A. I'm embarrassed that everyone has to take care of me. B. Once my depression is over, I'll be able to get on with life. C. I like being taken care of from time to time. D. I'm glad that I came for help in time.

A. I'm embarrassed that everyone has to take care of me. Clients diagnosed with depression usually have feelings of guilt and unworthiness, and have difficulty accepting help from others because of these feelings; while depressed clients may be dependent and demanding, they often feel unworthy of the attention they receive, and are embarrassed by their feelings of helplessness.

When caring for a person diagnosed with a peptic ulcer, which of the following nursing measure is indicated? A. Identify stress factors in the person's environment. B. Avoid giving the person choices to make. C. Encourage the person to become angry. D. Avoid discussing the person's symptoms.

A. Identify stress factors in the person's environment. Important to identify elements in the patient's environment that are contributing to stress when caring for a patient with a psychophysiological disorder; work habits and personal habits, such as smoking and drinking, must be evaluated to encourage the patient adopt less stressful lifestyle.

The nurse cares for the client diagnosed with a terminal illness. It is most important for the nurse to take which action? A. Let the client know the client is not alone. B. Attempt to promote hope in the client. C. Be helpful to the client at all times. D. Discourage denial in the client.

A. Let the client know the client is not alone. This is very important in the care of the dying client; fear of the unknown is frightening to dying clients; may become frightened of procedures and anything that reminds them that they will eventually die; need the continual reassurance that staff and family are available at all times and will be there for them.

The terminal client dies quietly while sleeping. The nurse takes which action? A. Provides a private place for family members. B. Explains that the client is in heaven now. C. Notifies the family members individually. D. Shields the family from viewing the client.

A. Provides a private place for family members. Providing a private place for family members shows compassion and understanding by the nurse.

The nurse observes the client develop a strong attachment to another client who repeatedly insults the client. The nurse understands this is an example of which behavior? A. Reaction formation B. Undoing C. Displacement D. Introjection

A. Reaction formation Expressing an attitude that is directly opposite to an unconscious wish or fear.

The 29-year-old client is told by the HCP that she cannot have children. The client subsequently forms a close attachment to the niece and nephew. The nurse understands this is an example of which defense mechanism? A. Sublimation B. Projection C. Undoing D. Rationalization

A. Sublimation This client is sublimating the desire to be a mother through a close attachment to the niece and nephew; by using sublimation, the client can satisfy some unmet maternal instincts; the client will still have to work through the issues of loss and the mourning process that usually accompanies infertility.

Which nursing intervention specifically helps reduce a patients anxiety? 1) Teaching the importance of adequate nutrition and hydration 2) Giving clear fact pertaining to the patients circumstances 3) Promoting small group activities to improve self-esteem 4) Monitoring the patient for the risk of suicide

ANS: 2 Using clear and factual knowledge that is tailored to the patients circumstances helps reduce anxiety. Teaching the importance of adequate hydration, promoting small group activities to improve self-esteem, and monitoring the patient for suicide risk are interventions designed to help the patient with depression.

The client diagnosed with paranoid schizophrenia tells the nurse, "I have a feeling of numbness in my legs. They feel like they don't belong to me, and I think someone on TV is controlling my walking." Which response by the nurse is best? A. That must be an unpleasant experience for you. Have you had these feelings before? B. I know you are frightened now, but soon the medication will ease your symptoms. C. Part of your sickness is an imaginary world. In reality, television doe snot control people. D. Tell me more about these feelings.

A. That must be an unpleasant experience for you. Have you had these feelings before? By saying this, the nurse acknowledges the client is feeling uncomfortable, and attempts to have the client explain more about what is being felt; notice the nurse does not discuss the delusional content, but tries to focus on the feeling.

The adolescent is admitted to the psychiatric hospital. The adolescent reports hitting a sibling during an argument the previous weekend. After the argument, the client's arm became paralyzed. The nurse anticipates the client will react in which way about the paralysis? A. The client appears calm about the paralysis. B. The client expresses anxiety about permanent damage. C. The client improves with passive arm exercises. D. The client recognizes the symptoms are not real.

A. The client appears calm about the paralysis. Clients with conversion reactions usually appear calm and unconcerned with their physical manifestations; the anxiety is repressed and is converted into a physical symptom.

18. Samuel, a 19-year-old high school student, has been admitted to the psychiatric unit with a diagnosis of adjustment disorder with disturbance of conduct. He assaulted a teacher when he was told he was receiving detentions for a pattern of tardiness. The nurse, while completing rounds, finds the patient in his room crying, and one of his wrists is bleeding from a self-inflicted cut made by a piece of metal from an unknown source. Prioritize each of the following nursing interventions from 1 to 5, with 1 being the highest priority. ___ A. Check the patients vital signs. ___ B. Assess the wound site. ___ C. Contact the parents. ___ D. Discuss with Samuel what precipitated this event. ___ E. Cleanse and treat the wound site to prevent infection.

ANS: A: 3 B: 1 C: 5 D: 4 E: 2 The first priority is assessment (Item B), followed by providing care to meet physical and safety needs (Items E and A). The next priority is responding to the patients emotional needs (Item D), and finally, contacting the patients parents (Item C) in accordance with standards for confidentiality of medical information.

A homeless patient is admitted with an infected leg wound. According to Maslows Hierarchy of Needs, which nursing intervention meets one of his basic physiological needs? 1) Providing the patient with a dinner tray 2) Administering antibiotics as prescribed 3) Irrigating a wound with normal saline solution 4) Encouraging the patient to express his feelings

ANS: 1 According to Abraham Maslow and his Hierarchy of Needs, basic physiological needs, such as food, should be addressed first. After the patients basic needs are met, the nurse can provide wound care, administer antibiotics as prescribed (safety needs), and encourage the patient to express his feelings (love and belonging or self-actualization, depending on what feelings he expresses.)

The nurse caring for a patient admitted with severe depression identifies a nursing diagnosis of Hopelessness on the care plan. Which outcome is appropriate for this diagnosis? 1) Displays stabilization and control of mood 2) Sleeps 6 to 8 hours per night with report of feeling rested 3) Does not engage in risky, self-injurious behavior 4) Eats a well-balanced diet to prevent weight change

ANS: 1 An outcome for the nursing diagnosis Hopelessness is displays stabilization and control of mood. Sleeps 6 to 8 hours per night and reports feeling rested and eats a well-balanced diet to prevent weight change are example of outcomes for the diagnosis Depressed Mood. Does not engage in risky, self-injurious behavior is an outcome for the nursing diagnosis Risk for Suicide.

An adult patient is diagnosed with lung cancer, and surgery to remove the right lung is recommended. The patient is uncertain about whether he should consent to the surgery because of the risks involved. Which nursing diagnosis is most appropriate for this patient? 1) Decisional Conflict 2) Death Anxiety 3) Powerlessness 4) Ineffective Denial

ANS: 1 Decisional Conflict is the most appropriate nursing diagnosis for this patient because he is uncertain about whether he should take the surgical risk. Death Anxiety is apprehension, worry, or fear related to death or dying; there is nothing to suggest that this patient is suffering from Death Anxiety at this time. Powerlessness is a perceived lack of control over a current situation; this patient is trying to exert some control over his care. Ineffective Denial is appropriate when the patient consciously or unconsciously rejects knowledge; there is nothing in this scenario to suggest that the patient is rejecting knowledge.

An elderly patient admitted from a skilled nursing residence to a medical-surgical unit is exhibiting confusion, distractibility, memory loss, and irritability. She has a medical diagnosis of dehydration. Which of the following should lead the nurse to suspect that dementia, rather than depression or dehydration, is the source of the symptoms: The history and nursing observations indicate that the patient 1) Rambles, speaks incoherently, answers questions inappropriately. 2) Speaks slowly with delayed response to questions, but responds appropriately. 3) Awakens early in the day yet sleeps almost constantly during the day. 4) Sometimes has difficulty concentrating on details of the present situation

ANS: 1 In dementia, a patients language is disoriented, rambling, and incoherent; and the patient responds to questions inappropriately or with near misses. Speaking slowly and being slow to respond to verbal stimuli are signs of depression; and in depression, the patient usually answers questions appropriately. Awakening early and sleeping constantly during the day are signs of depression; in dementia, sleep is fragment and the person awakens often during the night. Difficulty concentrating on details is a thinking pattern seen more in depression; in dementia, there is difficulty finding words, difficulty calculating, and decreased judgment.

Which nursing diagnosis is categorized as a psychosocial, rather than a self-concept, diagnosis? 1) Ineffective Individual Coping 2) Situational Low Self-Esteem 3) Disturbed Personal Identity 4) Disturbed Body Image

ANS: 1 Ineffective Individual Coping is considered a psychosocial nursing diagnosis. It implies poor life choices, inability to use available resources, and other interactional and relationship symptoms. The term psychosocial encompasses both psychological and social factors. The other diagnoses represent primarily individual, psychological factors. They are examples of self-concept nursing diagnoses.

A 13-year-old patient is admitted to the hospital. There is no medical restriction on visitation. To help maintain the patients social identity while hospitalized, it is most important for the nurse to encourage visits by: 1) Peers. 2) Grandparents. 3) Siblings. 4) Parents.

ANS: 1 Peers are more important than family in maintaining social identity in this age group.

A 17-year-old patient sustained facial fractures and a 6-inch laceration on the left side of her face in a motor vehicle accident. The patient tells the nurse that she does not want anyone to see her looking this way. Which statement by the nurse is most appropriate? 1) Tell me what you mean by looking this way. 2) OK, Ill restrict your visitors until your face heals. 3) Your friends and family love you no matter what. 4) Youre young, your face will heal quickly.

ANS: 1 Tell me what you mean . . . encourages the patient to clarify her statement so that the nurse knows exactly what the patient means. The nurse cannot assume that the patient is talking about her facial wounds. Ill restrict your visitors . . . assumes that the patient is speaking about her facial wounds when she might not be. The other options are examples of false reassurance and do not address the patients concerns.

The nurse is caring for a group of patients on the medical-surgical unit. Which patient is most likely to experience the most difficulty in adapting to a change in body image? The patient: 1) Who suffered a traumatic amputation of the left leg in an industrial accident. 2) With hypothyroidism who has coarse, dry, thinning hair, and weight gain. 3) Who is obese and who underwent gastric bypass surgery. 4) With peripheral vascular disease who required a wound graft.

ANS: 1 Theoretically, the patient who suffered a traumatic amputation in an industrial accident will most likely have more difficultly adjusting to his change in body image because the change occurred abruptly. The patients described in the other options will naturally have some difficulty adjusting to their body image change, but it should not be as great because the physical changes are more gradual, which allows for adaptation over time.

Identify one or more choices that best complete the statement or answer the question. ____ Which assessment finding(s) might suggest that the patient has low self-esteem and requires more in-depth assessment? Choose all that apply. 1) Infrequent eye contact 2) Straight posture 3) Overly critical of others 4) Careful grooming

ANS: 1, 3 Assessment findings that suggest low self-esteem include avoiding eye contact and being overly critical of others. You would not need to follow up if the person displayed straight posture and careful grooming.

A 73-year-old patient was admitted with a perforated bowel. Following surgical repair, he developed complications and required an extensive stay in the hospital. How can the medical-surgical nurse best promote self-esteem in this patient? 1) Assist the patient to ambulate in the hallway once daily. 2) Encourage the patient to participate in self-care. 3) Introduce herself to the patient if he does not know her. 4) Listen attentively when the patient speaks.

ANS: 2 Encouraging the patient to his own self-care, such as bathing and brushing his teeth, encourage independence and promote self-esteem. Assisting the patient to ambulate in the hallway prevents complications of immobility. Introducing yourself and listening attentively to the patient prevents depersonalization.

A patient has recently had a change in a family relationship that is greatly affecting his health. Which nursing diagnosis could you probably make for this patient? 1) Parental Role Conflict 2) Interrupted Family Processes 3) Compromised Family Coping 4) Ineffective Individual Coping

ANS: 2 Interrupted Family Processes is defined as a change in a family relationship significantly impacting upon a patients health. Parental Role Conflict occurs when significant role confusion by a parent results in response to crises. Compromised Family Coping occurs when support from a usual family member is compromised or disabled, causing a significant health challenge. Ineffective Individual Coping occurs when the patient is unable to comprehend and effectively judge stressors.

A patient undergoing fertility treatments for the past 9 months learns that despite in vitro fertilization she still is not pregnant. This patient is at risk for experiencing a crisis in which component of self-concept? 1) Body image 2) Self-esteem 3) Personal identity 4) Role performance

ANS: 2 Setbacks such as not becoming pregnant after months of fertility treatment can cause the patient to question her self-worth. This might provoke a crisis in self-esteem. The patient is not at risk for experiencing a crisis in body image, personal identity, or role performance.

Which of the following can the nurse assess using Erik Eriksons theory? 1) Moral development 2) Developmental tasks 3) Social identity 4) Self-esteem

ANS: 2 Using Eriksons theory, the nurse can assess for successful completion of development tasks. The theory does not help the nurse assess social identity or self-esteem. However, these factors are components of developmental tasks that Eriksons theory explores. Moral development was addressed in the Kohlbergs theory.

The nurse is updating a care plan for a patient who has a nursing diagnosis of Anxiety. Which patient behavior might suggest that the problem is resolving? 1) Pacing in the hallway at intervals 2) Using relaxation techniques 3) Speaking rapidly when spoken to 4) Avoiding eye contact

ANS: 2 Using relaxation techniques might suggest that the patients anxiety is resolving. Pacing, speaking rapidly, and avoiding eye contact suggest that anxiety is still a problem for the patient. The patients use of relaxation techniques indicates problem solving by the patient.

Identify one or more choices that best complete the statement or answer the question. Which intervention(s) by the nurse might help the patient maintain a sense of personhood during hospitalization? Assume that all are culturally appropriate. Choose all that apply. 1) Addressing the patient by his first name 2) Making eye contact if it is comfortable for the patient 3) Always offering an explanation before beginning a procedure 4) Speaking to others about the patient so that the patient can hear you

ANS: 2, 3 The nurse can help the patient maintain a sense of personhood by addressing the patient by his preferred name, which might be his first name or might be surname with title. Using eye contact, always offering an explanation before beginning a procedure, and not talking about the patient to others in the room are additional ways for the nurse to offer care that respects patient rights.

The nurse is developing a plan of care for a mother of three small children who has been admitted with a serious acute illness, which is likely to continue long term. The nurse writes the following intervention: Facilitate communication between patient and significant other regarding the sharing of responsibilities to accommodate changes brought on by illness. The purpose of this intervention is to help: 1) Promote self-esteem. 2) Promote positive body image. 3) Facilitate role enhancement. 4) Prevent depersonalization.

ANS: 3 Facilitating communication between the patient and significant other regarding sharing of responsibilities to accommodate changes brought on by the illness can help facilitate role enhancement in the patient. The intervention is not designed to promote self-esteem or positive body image or to prevent depersonalization.

Which statement best describes self-concept? An individuals: 1) Understanding of how others perceive him. 2) Evaluation of himself. 3) Overall view of himself. 4) Perspective of his role in society.

ANS: 3 Self-concept is an individuals overall view of himself. The overall view includes his evaluation of himself and how he thinks others evaluate him.

The nurse is teaching a group of parents about growth and development. Which of the following statements by a parent would indicate correct understanding of self-concept? Self-concept stabilizes during: 1) Childhood. 2) Preadolescence. 3) Midadolescence. 4) Adulthood.

ANS: 3 Self-concept is present by ages 6 or 7 years; fluctuations peak during preadolescence, and it stabilizes during midadolescence. Self-concept remains stable throughout adulthood.

Which individual is most likely to have a positive body image? 1) Child who has been deaf since birth 2) Child who was born with cystic fibrosis 3) Adolescent of average appearance who had an appendectomy 4) Adult born with a spinal defect and associated paralysis of the lower body

ANS: 3 The adolescent with average appearance who had an appendectomy is likely to have a positive body image because the adolescent suffered an acute, reversible illness. Those born with physical handicaps are less likely to have a positive body image because many times the handicap leaves them socially isolated. This is, of course, not to imply that no one born with a physical handicap has a positive body image; and, of course, a particular adolescents body image might suffer after an appendectomy. However, the question asks which is most likely based on theoretical knowledge of body image.

A patient comes to the emergency department complaining of headache, palpitations, nausea, and dizziness. After determining that the patient is anxious, the nurse notes tachycardia and trembling. Which level of anxiety is this patient exhibiting? 1) Mild anxiety 2) Moderate anxiety 3) Severe anxiety 4) Panic anxiety

ANS: 3 The patient experiencing severe anxiety may experience physical symptoms including headache, palpitations, tachycardia, insomnia, dizziness, nausea, trembling, hyperventilation, urinary frequency, and diarrhea. Symptoms associated with mild anxiety include muscle tension, restlessness, irritability, and a sense of unease. The patient experiencing moderate anxiety might experience a rise in heart rate and respiratory rate, increased perspiration, gastric discomfort, and increased muscle tension. The patient suffering from panic anxiety might believe he has a life-threatening illness. Physical symptoms include dilated pupils, labored breathing, severe trembling, sleeplessness, palpitations, diaphoresis, pallor, and uncoordinated muscle movements.

A patient admitted with depression has a nursing diagnosis of Chronic Low Self-Esteem. Which NOC outcome is essential for this nursing diagnosis? 1) Decision Making 2) Distorted Thought Content 3) Role Performance 4) Depression Level

ANS: 4 Depression Level is the appropriate NOC outcome for the patient admitted with depression who has the nursing diagnosis, Chronic Low Self-Esteem. Decision Making is associated with the nursing diagnosis Situational Low Self-Esteem; Role performance with Ineffective role performance; and Distorted Thought Content with Disturbed Personal Identity. Although the other options might contribute to the patients low self-esteem, the nurse must write one goal (outcome) that, if achieved, would demonstrate resolution of the nursing diagnosis. Decision Making is the only outcome that does that.

The nursing diagnosis Disturbed Personal Identity is identified for a newly admitted patient. Which of the following is an example of an individualized goal for that patient? 1) Distorted Thought Control 2) Anxiety Level 3) Self-Mutilation Restraint 4) No self-injury, consistently demonstrated

ANS: 4 Does not injure self, consistently demonstrated is an example of using NOC indicators and outcomes to write an individualized goal. The other options are examples of NOC outcomes; they are not written as goals.

The nurse is updating the care plan of a patient who must undergo a right mastectomy for breast cancer. Which nursing diagnosis should the nurse anticipate in expectation of the body changes associated with the upcoming surgery? 1) Deficient Knowledge 2) Impaired Adjustment 3) Hopelessness 4) Grieving

ANS: 4 Grieving may occur as a result of body changes associated with mastectomy. Deficient Knowledge, Impaired Adjustment, and Hopelessness are not associated with the expected body changes associated with the upcoming surgery, although they could certainly occur.

Which response by the patient demonstrates an internal locus of control? 1) My blood sugar wouldnt be out of control if my wife prepared better foods. 2) I knew I shouldnt have come to this hospital; Id be better if I hadnt. 3) God must be getting even with me for my past behavior. 4) Im just glad to be alive; the accident couldve been a lot worse.

ANS: 4 People who demonstrate an internal locus of control take responsibility for their life experiences and their response to them. This allows them to interpret unexpected events in a positive light, as the response . . . the accident couldve been a lot worse illustrates. The other options demonstrate an external locus of control; control of the situation is attributed to external factors.

Which statement best describes self-esteem? 1) View of oneself as a unique human being 2) Ones mental image of ones physical self 3) Ones overall view of oneself 4) How well one likes oneself

ANS: 4 Personal identity is ones view of oneself as a unique human being. Body image is described as ones mental image of ones physical self. Self-concept is defined as ones overall view of oneself. Self-esteem is a favorable impression of oneself or self-respect.

The nurse is assessing a patient for depression. Which of the following sets of behavioral symptoms may indicate depression? 1) Preoccupation with loss, self-blame, and ambivalence 2) Anger, helplessness, guilt, and sadness 3) Anorexia, insomnia, headache, and constipation 4) Tearfulness, withdrawal, and present substance abuse

ANS: 4 Tearfulness, regression, restlessness, agitation, withdrawal, past or present substance abuse, and a past history of suicide attempts are all behavioral symptoms of depression. Denial of feelings, anger, anxiety, guilt, helplessness, hopelessness, and sadness are affective findings associated with depression. Cognitive findings in depression include preoccupation with loss, self-blame, ambivalence, and blaming others. Physiological findings of depression include anorexia, overeating, insomnia, hypersomnia, headache, backache, chest pain, and constipation.

bleeding duodenal ulcer. He is exhibiting physiological signs of anxiety and seems to have difficulty concentrating. During the interview, the patient tells the nurse that he is often short of breath and says, I lie awake nights worrying about everything. He has been unable to work or care for his family for the past 6 months. What is the nurses priority after documenting this information in the nurses notes? 1) Provide emotional support for the patient using reflective listening technique. 2) Do nothing; people with duodenal ulcers typically cannot work. 3) Question the patients family about the information received from the patient. 4) Notify the primary care provider and ask for a referral to a mental health professional.

ANS: 4 The nurse should involve a mental health professional immediately, because the patient is exhibiting signs of a disabling anxiety disorder. Although it is important for the nurse to provide emotional support for the patient, a mental health professional is needed for this patient. Doing nothing is neglectful. Questioning the patients family about the information violates the patients right to privacy, unless the nurse obtains the patients permission to do so.

A 35-year-old patient diagnosed with testicular cancer is undergoing chemotherapy, which leaves him unable to help care for his young children. As a result, his wife misses work whenever the children are ill. She has become increasingly distressed over her situation. Her experience best demonstrates which of the following? 1) Role strain 2) Interpersonal role conflict 3) Role performance 4) Interrole conflict

ANS: 4 The patients wife is most likely experiencing interrole conflict, in which her role as a mother and worker are making competing demands on her. Role strain is a mismatch between role expectations and role performance. Interpersonal role conflict results when another persons idea about how a role should be performed differs from that of the person who is performing the role. Role performance is defined as the actions a person takes and the behaviors he demonstrates in performing a role.

24. A client diagnosed with major depressive disorder was raised in a strongly religious family where bad behavior was equated with sins against God. Which nursing intervention would be most appropriate to help the client address spirituality as it relates to his illness? A. Encourage the client to bring into awareness underlying sources of guilt. B. Teach the client that religious beliefs should be put into perspective throughout the life span. C. Confront the client with the irrational nature of the belief system. D. Assist the client to modify his or her belief system in order to improve coping skills

ANS: A A client raised in an environment that reinforces ones inadequacy may be at risk for experiencing guilt, shame, low self-esteem, and hopelessness, which can contribute to depression. Assisting the client to bring these feelings into awareness allows the client to realistically appraise distorted responsibility and dysfunctional guilt.

13. A client is admitted to the psychiatric unit with a diagnosis of major depressive disorder. The client is unable to concentrate, has no appetite, and is experiencing insomnia. Which should be included in this clients plan of care? A. A simple, structured daily schedule with limited choices of activities B. A daily schedule filled with activities to promote socialization C. A flexible schedule that allows the client opportunities for decision making D. A schedule that includes mandatory activities to decrease social isolation

ANS: A A client with depression has difficulty concentrating and may be overwhelmed by activity overload or the expectation of independent decision making. A simple, structured daily schedule with limited choices of activities is more appropriate.

4. A nurse reviews the laboratory data of a 29-year-old client suspected of having major depressive disorder. Which laboratory value would potentially rule out this diagnosis? A. Thyroid-stimulating hormone (TSH) level of 6.2 U/mL B. Potassium (K+) level of 4.2 mEq/L C. Sodium (Na+) level of 140 mEq/L D. Calcium (Ca2+) level of 9.5 mg/dL

ANS: A According to the DSM-5, symptoms of major depressive disorder cannot be due to the direct physiological effects of a general medical condition (e.g., hypothyroidism). The diagnosis of major depressive disorder may be ruled out if the clients laboratory results indicate a high TSH level (normal range for this age group is 0.4 to 4.2 U/mL), which results from a low thyroid function, or hypothyroidism. In hypothyroidism metabolic processes are slowed, leading to depressive symptoms.

18. A client who is admitted to the inpatient psychiatric unit and is taking Thorazine presents to the nurse with severe muscle rigidity, tachycardia, and a temperature of 105F (40.5C). The nurse identifies these symptoms as which of the following conditions? A. Neuroleptic malignant syndrome B. Tardive dyskinesia C. Acute dystonia D. Agranulocytosis

ANS: A Neuroleptic malignant syndrome is a potentially fatal condition characterized by muscle rigidity, fever, altered consciousness, and autonomic instability.

23. A client is admitted with a diagnosis of persistent depressive disorder. Which client statement would describe a symptom consistent with this diagnosis? A. I am sad most of the time and Ive felt this way for the last several years. B. I find myself preoccupied with death. C. Sometimes I hear voices telling me to kill myself. D. Im afraid to leave the house.

ANS: A Persistent depressive disorder is characterized by depressed mood for most of day, for more days than not, for at least 2 years. Thoughts of death would be more consistent with major depressive disorder; hearing voices is more consistent with a psychotic disorder; and fear of leaving the house is more consistent with a phobia.

22. If clozapine (Clozaril) therapy is being considered, the nurse should evaluate which laboratory test to establish a baseline for comparison in order to recognize a potentially life-threatening side effect? A. White blood cell count B. Liver function studies C. Creatinine clearance D. Blood urea nitrogen

ANS: A The nurse should establish a baseline white blood cell count to evaluate a potentially life-threatening side effect if clozapine (Clozaril) is being considered as a treatment option. Clozapine can have a serious side effect of agranulocytosis, in which a potentially fatal drop in white blood cells can occur.

3. A 16-year-old client diagnosed with schizophrenia experiences command hallucinations to harm others. The clients parents ask a nurse, Where do the voices come from? Which is the appropriate nursing reply? A. Your child has a chemical imbalance of the brain, which leads to altered thoughts. B. Your childs hallucinations are caused by medication interactions. C. Your child has too little serotonin in the brain, causing delusions and hallucinations. D. Your childs abnormal hormonal changes have precipitated auditory hallucinations.

ANS: A The nurse should explain that a chemical imbalance of the brain leads to altered thought processes. Hallucinations, or false sensory perceptions, may occur in all five senses. The client who hears voices is experiencing an auditory hallucination.

11. A psychiatrist prescribes a monoamine oxidase inhibitor (MAOI) for a client. Which foods should the nurse teach the client to avoid? A. Pepperoni pizza and red wine B. Bagels with cream cheese and tea C. Apple pie and coffee D. Potato chips and diet cola

ANS: A The nurse should instruct the client to avoid pepperoni pizza and red wine. Foods with high tyramine content can induce hypertensive crisis within 2 hours of ingestion. Symptoms of hypertensive crisis include severe occipital and/or temporal pounding headaches with occasional photophobia, sensations of choking, palpitations, and a feeling of dread.

21. A client diagnosed with schizophrenia is prescribed clozapine (Clozaril). Which client symptoms related to the side effects of this medication should prompt a nurse to intervene immediately? A. Sore throat, fever, and malaise B. Akathisia and hypersalivation C. Akinesia and insomnia D. Dry mouth and urinary retention

ANS: A The nurse should intervene immediately if the client experiences a sore throat, fever, and malaise when taking the atypical antipsychotic drug clozapine (Clozaril). Clozapine can have a serious side effect of agranulocytosis, in which a potentially fatal drop in white blood cells can occur. Symptoms of infectious processes would alert the nurse to this potential.

13. . Which statement should indicate to a nurse that an individual is experiencing a delusion? A. Theres an alien growing in my liver. B. I see my dead husband everywhere I go. C. The IRS may audit my taxes. D. Im not going to eat my food. It smells like brimstone.

ANS: A The nurse should recognize that a client who claims that an alien is inside his or her body is experiencing a delusion. Delusions are false personal beliefs that are inconsistent with the persons intelligence or cultural background.

A psychiatrist who embraces the Psychological Recovery Model tells the nurse that a client is in the Growth stage. What should the nurse expect to find when assessing this client? A. A client feeling confident about achieving goals in life. B. A client who is aware of the need to set goals in life. C. A client who has mobilized personal and external resources. D. A client who begins to actively take control of his or her life.

ANS: A Andresen and associates have conceptualized a five-stage model of recovery called the Psychological Recovery Model. The stages include Stage 1, Moratorium; Stage 2, Awareness; Stage 3, Preparation; Stage 4, Rebuilding; and Stage 5, Growth. In the growth stage, the individual feels a sense of optimism and hope of a rewarding future. Skills that have been nurtured in the previous stages are applied with confidence, and the individual strives for higher levels of well-being. KEY: Cognitive Level: Analysis | Integrated Processes: Nursing Process: Assessment | Client Need: Psychosocial Integrity

A client states, "My illness is so devastating, I feel like my life is on hold." The nurse recognizes that this client is in which stage of the Psychological Recovery Model as described by Andersen and associates? A. Moratorium B. Awareness C. Preparation D. Rebuilding

ANS: A Andresen and associates have conceptualized a five-stage model of recovery called the Psychological Recovery Model. The stages include Stage 1, Moratorium; Stage 2, Awareness; Stage 3, Preparation; Stage 4, Rebuilding; and Stage 5, Growth. The moratorium stage is identified by dark despair and confusion. It is called moratorium, because it seems that "life is on hold." KEY: Cognitive Level: Application | Integrated Processes: Nursing Process: Evaluation | Client Need: Psychosocial Integrity

A client is questioning the nurse about a newly prescribed medication, acamprosate calcium (Campral). Which is the most appropriate reply by the nurse? A. "This medication will help you maintain your abstinence." B. "This medication will cause uncomfortable symptoms if you combine it with alcohol." C. "This medication will decrease the effect alcohol has on your body." D. "This medication will lower your risk of experiencing a complicated withdrawal."

ANS: A Campral has been approved by the U.S. Food and Drug Administration (FDA) for the maintenance of abstinence from alcohol in clients diagnosed with alcohol dependence who are abstinent at treatment initiation.

13. A military vet who recently returned from active duty in a Middle Eastern country and suffers from PTSD states he will not allow the lab tech, who is Iranian, to draw his blood. The patient states Hell probably use a contaminated needle on me. Which of these is the most appropriate response by the nurse? A. Let me see if I can arrange for a different technician to draw your blood. B. Let me help you overcome your cultural bias by letting him draw your blood. C. There is no other technician, so youre just going to have to let him draw your blood. D. I dont think the technician is really Middle Eastern.

ANS: A Item A demonstrates acceptance of the patient and attempts to create a less threatening situation for the patient. Item B makes an unsubstantiated assumption about the patients biases. Item C will not contribute to the patients sense of control, and sense of comfort and control is important in managing symptoms of PTSD. Item D minimizes the patients concerns rather than responding empathically to them.

Providing nursing education on drug abuse to a high school class is an example of which level of preventive care? A. Primary prevention B. Secondary prevention C. Tertiary prevention D. Primary intervention

ANS: A Providing nursing education on drug abuse to a high school class is an example of primary prevention. Primary prevention services are aimed at reducing the incidence of mental health disorders within the population. KEY: Cognitive Level: Application | Integrated Processes: Nursing Process: Implementation | Client Need: Health Promotion and Maintenance

A client diagnosed with chronic alcohol dependency is being discharged from an inpatient treatment facility after detoxification. Which client outcome related to Alcoholics Anonymous (AA) would be most appropriate for a nurse to discuss with the client during discharge teaching? A. After discharge, the client will immediately attend 90 AA meetings in 90 days. B. After discharge, the client will rely on an AA sponsor to help control alcohol cravings. C. After discharge, the client will incorporate family in AA attendance. D. After discharge, the client will seek appropriate deterrent medications through AA.

ANS: A The most appropriate client outcome for the nurse to discuss during discharge teaching is attending 90 AA meetings in 90 days after discharge. AA is a major self-help organization for the treatment of alcoholism. It accepts alcoholism as an illness and promotes total abstinence as the only cure.

A client diagnosed with neurocognitive disorder exhibits progressive memory loss, diminished cognitive functioning, and verbal aggression upon experiencing frustration. Which nursing intervention is most appropriate? A. Schedule structured daily routines. B. Minimize environmental lighting. C. Organize a group activity to present reality. D. Explain the consequences for aggressive behaviors.

ANS: A The most appropriate nursing intervention for this client is to schedule structured daily routines. A structured routine will reduce frustration and thereby reduce verbal aggression. KEY: Cognitive Level: Application | Integrated Processes: Nursing Process: Implementation | Client Need: Psychosocial Integrity

A nursing instructor is teaching about donepezil (Aricept). A student asks, "How does this work? Will this cure Alzheimer's disease (AD)?" Which is the appropriate instructor reply? A. "This medication delays the destruction of acetylcholine, a chemical in the brain necessary for memory processes. Although most effective in the early stages, it serves to delay, but not stop, the progression of the AD." B. "This medication encourages production of acetylcholine, a chemical in the brain necessary for memory processes. It delays the progression of the disease." C. "This medication delays the destruction of dopamine, a chemical in the brain necessary for memory processes. Although most effective in the early stages, it serves to delay, but not stop, the progression of the AD." D. "This medication encourages production of dopamine, a chemical in the brain necessary for memory processes. It delays the progression of the disease."

ANS: A The most appropriate response by the instructor is to explain that donepezil (Aricept) delays the destruction of acetylcholine, a chemical in the brain necessary for memory processes. Although most effective in the early stages, it serves to delay, but not stop, the progression of AD. KEY: Cognitive Level: Analysis | Integrated Processes: Teaching/Learning | Client Need: Physiological Integrity: Pharmacological and Parenteral Therapies

A client on the inpatient unit tells a student nurse, "My life has no purpose. I can't think about living another day, but please don't tell anyone about the way I feel. I know you are obligated to protect my confidentiality." Which is the most appropriate reply by the student nurse? A. "The treatment team is composed of many specialists who are working to improve your ability to function. Sharing this information with the team is critical to your care." B. "Let's discuss steps that will resolve negative lifestyle choices that may increase your suicidal risk." C. "You seem to be preoccupied with self. You should concentrate on hope for the future." D. "This information is secure with me because of client confidentiality."

ANS: A The most appropriate response by the student nurse is to explain that sharing the information with the treatment team is critical to the client's care. The nurse's priority is to ensure client safety and to inform others of the client's suicidal ideation. KEY: Cognitive Level: Application | Integrated Processes: Nursing Process: Implementation | Client Need: Safe and Effective Care Environment

A client has a history of daily bourbon drinking for the past 6 months. He is brought to an emergency department by family who report that his last drink was 1 hour ago. It is now 12 midnight. When should a nurse expect this client to exhibit withdrawal symptoms? A. Between 3 a.m. and 11 a.m. B. Shortly after a 24-hour period C. At the beginning of the third day D. Withdrawal is individualized and cannot be predicted.

ANS: A The nurse should expect that this client will begin experiencing withdrawal symptoms from alcohol between 3 a.m. and 11 a.m. Symptoms of alcohol withdrawal usually occur within 4 to 12 hours of cessation or reduction in heavy and prolonged alcohol use.

A geriatric nurse is teaching student nurses about the risk factors for development of delirium in older adults. Which student statement indicates that learning has occurred? A. "Taking multiple medications may lead to adverse interactions or toxicity." B. "Age-related cognitive changes may lead to alterations in mental status." C. "Lack of rigorous exercise may lead to decreased cerebral blood flow." D. "Decreased social interaction may lead to profound isolation and psychosis."

ANS: A The nurse should identify that taking multiple medications may lead to adverse reactions or toxicity and put an older adult at risk for the development of delirium. Symptoms of delirium include difficulty sustaining and shifting attention. The client with delirium is disoriented to time and place and may also have impaired memory. KEY: Cognitive Level: Application | Integrated Processes: Nursing Process: Evaluation | Client Need: Physiological Integrity

A client diagnosed with depression and substance abuse has an altered sleep pattern and demands that a psychiatrist prescribe a sedative. Which rationale explains why a nurse should encourage the client to first try nonpharmacological interventions? A. Sedative-hypnotics are potentially addictive and will lose their effectiveness due to tolerance. B. Sedative-hypnotics are expensive and have numerous side effects. C. Sedative-hypnotics interfere with necessary REM (rapid eye movement) sleep. D. Sedative-hypnotics are not as effective to promote sleep as antidepressant medications.

ANS: A The nurse should recommend nonpharmacological interventions to this client because sedative-hypnotics are potentially addictive and will lose their effectiveness due to tolerance. The effects of central nervous system depressants are additive with one another and are capable of producing physiological and psychological dependence.

A lonely, depressed divorcée has been self-medicating with cocaine for the past year. Which term should a nurse use to best describe this individual's situation? A. The individual is experiencing psychological dependency. B. The individual is experiencing physical dependency. C. The individual is experiencing substance dependency. D. The individual is experiencing social dependency.

ANS: A The nurse should use the term "psychological dependency" to best describe this client's situation. A client is considered to be psychologically dependent on a substance when there is an overwhelming desire to use a substance in order to produce pleasure or avoid discomfort.

What should be the priority nursing diagnosis for a client experiencing alcohol withdrawal? A. Risk for injury R/T central nervous system stimulation B. Disturbed thought processes R/T tactile hallucinations C. Ineffective coping R/T powerlessness over alcohol use D. Ineffective denial R/T continued alcohol use despite negative consequences

ANS: A The priority nursing diagnosis for a client experiencing alcohol withdrawal should be risk for injury R/T central nervous system stimulation. Alcohol withdrawal may include the following symptoms: course tremors of hands, tongue, or eyelids; seizures; nausea or vomiting; malaise or weakness; tachycardia; sweating; elevated blood pressure; anxiety; depressed mood; hallucinations; headache; and insomnia.

A client is admitted for alcohol detoxification. During detoxification, which symptoms should the nurse expect to assess? A. Gross tremors, delirium, hyperactivity, and hypertension B. Disorientation, peripheral neuropathy, and hypotension C. Oculogyric crisis, amnesia, ataxia, and hypertension D. Hallucinations, fine tremors, confabulation, and orthostatic hypotension

ANS: A Withdrawal is defined as the physiological and mental readjustment that accompanies the discontinuation of an addictive substance. Symptoms can include gross tremors, delirium, hyperactivity, hypertension, nausea, vomiting, tachycardia, hallucinations, and seizures.

6. A patient admitted to the hospital with PTSD is ordered the following medications. Which of these medications has a direct use in treating symptoms that are common in PTSD? Select all that apply. A. Alprazolam B. Propanolol C. Colace D. Dulcolax

ANS: A, B Alprazolam is an antianxiety agent and anxiety symptoms are common in PTSD. Propanolol is an antihypertensive medication and evidence has demonstrated its effectiveness in treating symptoms of PTSD, including nightmares, intrusive recollections, and insomnia. The last two medications are used to treat constipation, and this symptom is not directly related to PTSD.

4. A patients wife reports to the nurse that she was told her husbands PTSD may be related to cognitive problems. She is asking the nurse to explain what that means. Which of the following are accurate statements about the cognitive theory as it applies to PTSD? Select all that apply. A. People are vulnerable to trauma-related disorders when their fundamental beliefs are invalidated. B. Cognitive theory addresses the importance of how people think (or cognitively appraise) events. C. Dementia is a common symptom of PTSD. D. Amnesia is the biggest cognitive problem in PTSD and is the primary cause of trauma-related disorders.

ANS: A, B Both A and B address aspects of cognitive theory and its relevance in PTSD. Dementia includes cognitive symptoms but is not a symptom of PTSD. Amnesia does not cause PTSD but is a symptom of PTSD.

8. Joe, a patient being treated for PTSD, tells the nurse that his therapist is recommending cognitive therapy. He asks the nurse how thats supposed to help his nightmares. Which of these responses by the nurse provides accurate information about the benefits of this type of therapy? Select all that apply. A. The nightmares may be related to troubling thoughts and feelings; cognitive therapy will help you explore and modify those thoughts and feelings. B. It is designed to help you cope with anxiety, anger, and other feelings that may be related to your symptoms. C. It is designed to repeatedly expose you to the trauma you experienced so you can regain a sense of safety. D. Once you learn to repress these troubling feelings, the nightmares should cease

ANS: A, B Both A and B are desired outcomes in cognitive therapy. Item C more aptly describes prolonged exposure therapy. D is incorrect because exploration and awareness (rather than repression) are fundamental to cognitive therapy.

3. A military veteran is being assessed for outpatient therapy after he reports having problems at home and at work. Which of the symptoms that he describes are commonly associated with PTSD? Select all that apply. A. Ive been drinking and smoking pot daily. B. Ive been having trouble sleeping and I think Ive been having nightmares but I cant remember them. C. I slapped my wife when she was trying to hug me. D. Ive been having intense pain in the leg where I sustained a combat wound.

ANS: A, B, C Common symptoms associated with PTSD include substance abuse, sleep disturbances, nightmares, and aggression. Whereas the combat exposure and wounding could be described as traumas, the patients complaint of pain requires further physical assessment rather than assuming this symptom is related to PTSD.

25. After a teenager reveals that he is gay, the father responds by beating him. The next morning, the teenager is found hanging in his closet. Which paternal grief responses should a nurse anticipate? Select all that apply.

ANS: A, B, C I cant believe this is happening. If only I had been more understanding How dare he do this to me! Suicide of a family member can induce a whole gamut of feelings in the survivors. Shock, disbelief, guilt, remorse, anger, and resentment are all feelings that may be experienced by this father. The last two possible responses suggest acceptance and understanding. It is far more common for survivors of suicide to have a sense of feeling wounded and as if they will never get over it.

9. Joshua recently moved into a dormitory to begin his freshman year in college. He was reprimanded by the dormitory supervisor for not properly disposing of food items and responded by throwing all of his belongings from a second story window while shouting obscenities. The campus police escorted him to campus health services, where he was diagnosed with an Adjustment Disorder with Disturbance of Conduct. Which of the following items in Joshuas history predispose him to this disorder? Select all that apply. A. Joshua reports that he doesnt have any friends in the dormitory. B. Joshuas family currently lives out of the country and are often difficult to reach. C. Joshua was notified the same day that he would have to withdraw from one of his classes because he didnt have the prerequisite credits needed to register for the class. D. Joshua has a higher than average GPA and is a member of The National Honor Society.

ANS: A, B, C Items A and B may suggest lack of available support systems, which is identified as a predisposing factor for Adjustment Disorders. Item C presents evidence of another stressor occurring in proximity to the reprimand from the dormitory supervisor, which may also predispose to the development of an Adjustment Disorder. KEY: Cognitive Level: Application | Integrated Processes: Nursing Pro

30. A client is prescribed phenelzine (Nardil). Which of the following client statements should indicate to a nurse that discharge teaching about this medication has been successful? Select all that apply. A. Ill have to let my surgeon know about this medication before I have my cholecystectomy. B. Guess I will have to give up my glass of red wine with dinner. C. Ill have to be very careful about reading food and medication labels. D. Im going to miss my caffeinated coffee in the morning. E. Ill be sure not to stop this medication abruptly.

ANS: A, B, C, E The nurse should evaluate that teaching has been successful when the client states that phenelzine (Nardil) should not be taken in conjunction with the use of alcohol or foods high in tyramine and should not be stopped abruptly. Phenelzine is a monoamine oxidase inhibitor (MAOI) that can have negative interactions with other medications. The client needs to tell other physicians about taking MAOIs because of the risk of drug interactions.

Which of the following has the SAMHSA described, as major dimensions of support for a life of recovery? Select all that apply. A. Health B. Community C. Home D. Religious affiliation E. Purpose

ANS: A, B, C, E SAMHSA suggests that a life in recovery is supported by four major dimensions: health, home, purpose, and community. Religious affiliation is not included in the listed dimensions. KEY: Cognitive Level: Analysis | Integrated Processes: Nursing Process: Assessment | Client Need: Psychosocial Integrity

A nurse uses the commitments of the Tidal Model of Recovery in psychiatric nursing practice. Which of the following nursing actions reflect the use of the Develop Genuine Curiosity commitment? Select all that apply. A. The nurse expresses interest in the client's story. B. The nurse asks for clarification of certain points. C. The nurse encourages the client to speak his own words in his own unique way. D. The nurse assists the client to unfold the story at his or her own rate. E. The nurse provides the clients with copies of all documents relevant to care.

ANS: A, B, D Barker & Buchanan-Barker developed a set of essential values termed The 10 Tidal Commitments, upon which the Tidal Model is based. They include Value the Voice, Respect the Language, Develop Genuine Curiosity, Become the Apprentice, Use the Available Toolkit, Craft the Step Beyond, Give the Gift of Time, Reveal Personal Wisdom, Know that Change Is Constant, and Be Transparent. This nurse is employing the Develop Genuine Curiosity commitment, by expressing interest, asking for clarification, and assisting the client to unfold the story at his or her own rate. KEY: Cognitive Level: Application | Integrated Processes: Nursing Process: Implementation | Client Need: Psychosocial Integrity

Which of the following nursing statements exemplify the cognitive process that must be completed by a nurse prior to caring for clients diagnosed with substance-abuse disorders? (Select all that apply.) A. "I am easily manipulated and need to work on this prior to caring for these clients." B. "Because of my father's alcoholism, I need to examine my attitude toward these clients." C. "I need to review the side effects of the medications used in the withdrawal process." D. "I'll need to set boundaries to maintain a therapeutic relationship." E. "I need to take charge when dealing with clients diagnosed with substance disorders."

ANS: A, B, D The nurse should examine personal bias and preconceived negative attitudes prior to caring for clients diagnosed with substance-abuse disorders. A deficit in this area may affect the nurse's ability to establish therapeutic relationships with these clients.

28. Which of the following components should a nurse recognize as an integral part of a rehabilitative program when planning care for clients diagnosed with schizophrenia? Select all that apply. A. Group therapy B. Medication management C. Deterrent therapy D. Supportive family therapy E. Social skills training

ANS: A, B, D, E The nurse should recognize that group therapy, medication management, supportive family therapy, and social skills training all play an integral part in rehabilitative programs for clients diagnosed with schizophrenia. Schizophrenia results from various combinations of genetic predispositions, biochemical dysfunctions, physiological factors, and psychological stress. Effective treatment requires a comprehensive, multidisciplinary effort.

A nursing instructor is teaching students about cirrhosis of the liver. Which of the following student statements about the complications of hepatic encephalopathy should indicate that further student teaching is needed? (Select all that apply.) A. "A diet rich in protein will promote hepatic healing." B. "This condition leads to a rise in serum ammonia resulting in impaired mental functioning." C. "In this condition, blood accumulates in the abdominal cavity." D. "Neomycin and lactulose are used in the treatment of this condition." E. "This condition is caused by the inability of the liver to convert ammonia to urea."

ANS: A, C The nursing instructor should understand that further teaching is needed if the nursing student states that a diet rich in protein will promote hepatic healing and that this condition causes blood to accumulate in the abdominal cavity (ascites), because these are incorrect statements. The treatment of hepatic encephalopathy requires abstention from alcohol, temporary elimination of protein from the diet, and reduction of intestinal ammonia using neomycin or lactulose. This condition occurs in response to the inability of the liver to convert ammonia to urea for excretion.

28. A 20-year-old female has a diagnosis of premenstrual dysphoric disorder. Which of the following should a nurse identify as consistent with this diagnosis? Select all that apply. A. Symptoms are causing significant interference with work, school, and social relationships. B. Patient-rated mood is 2/10 for the past 6 months C. Mood swings occur the week before onset of menses D. Patient reports subjective difficulty concentrating E. Patient manifests pressured speech when communicating

ANS: A, C, D Diagnostic criteria for a premenstrual dysphoric disorder include that symptoms must be associated with significant distress, occur in the week before onset of menses, and improve or disappear in the week post-menses

2. A patient is admitted to the community mental health center for outpatient therapy with a diagnosis of Adjustment Disorder. Which of the following subjective statements by the patient support this diagnosis? Select all that apply. A. I was divorced 3 months ago and I cant seem to cope. B. I was a victim of date rape 15 years ago when I was in college. C. My partner came home last week and told me he just didnt love me anymore. D. I failed one of my classes last month and I cant get motivated to register for my next semester.

ANS: A, C, D A diagnosis of Adjustment Disorder is appropriate when the stressors are related to relational conflict, where there are significant emotional or behavioral symptoms, and when the response occurs within 3 months after the onset of the stressor (and persists no longer than 6 months). Item B would be more aptly described as a traumatic event.

26. A nursing student is developing a study guide related to historical facts about suicide. Which of the following facts should the student include? Select all that apply

ANS: A, C, D In the Middle Ages, suicide was viewed as a selfish and criminal act. Suicide was an offense in ancient Greece, and a common-site burial was denied. During the Renaissance, suicide was discussed and viewed more philosophically.

7. A patient who is being seen in the community mental health center for PTSD is being considered for EMDR (Eye Movement Desensitization and Reprocessing) therapy. The nurse is being asked to conduct an assessment to validate the patients appropriateness for this treatment. Which of the following pieces of data, collected by the nurse, are most important to document when determining appropriateness for treatment with EMDR? Select all that apply. A. The patient has a history of a seizure disorder. B. The patient has a history of ECT. C. The patient reports suicidal ideation with a plan. D. The patient has been using alcohol in increasing quantities over the last 3 months.

ANS: A, C, D Items A, C, and D are all factors that would contraindicate the use of EMDR. A history of ECT is not directly relevant in determining appropriateness for EMDR.

29. A nurse is administering risperidone (Risperdal) to a client diagnosed with schizophrenia. The therapeutic effect of this medication would most effectively address which of the following symptoms? Select all that apply. A. Somatic delusions B. Social isolation C. Gustatory hallucinations D. Flat affect E. Clang associations

ANS: A, C, E The nurse should expect that risperidone (Risperdal) would be effective treatment for somatic delusions, gustatory hallucinations, and clang associations. Risperidone is an atypical antipsychotic that has been effective in the treatment of the positive symptoms of schizophrenia and in maintenance therapy to prevent exacerbation of schizophrenic symptoms.

6. What is the priority reason for a nurse to perform a full physical health assessment on a client admitted with a diagnosis of major depressive disorder? A. The attention during the assessment is beneficial in decreasing social isolation. B. Depression is a symptom of several medical conditions. C. Physical health complications are likely to arise from antidepressant therapy. D. Depressed clients avoid addressing physical health and ignore medical problems.

ANS: B Medical conditions such as hormone disturbances, electrolyte disturbances, and nutritional deficiencies may produce symptoms of depression. These are a priority to identify and treat, since they may be the cause of the depressive symptoms and represent physiological needs.

5. A nurse who works on an inpatient psychiatric unit is working on developing a treatment plan for a patient admitted with PTSD. The patient, a military veteran, reports that sometimes he thinks he sees bombs exploding and the enemy rushing toward him. He has had aggressive outbursts and was hospitalized after assaulting a coworker during one of these episodes. Which of these interventions by the nurse are evidence-based responses? Select all that apply. A. Collaborate with the patient about how he would like staff to respond when he has episodes of re-experiencing traumatic events. B. Tell the patient it is not appropriate to hit other patients or staff and if that occurs he will have to be discharged from the hospital. C. Contact the doctor and recommend that the patient be ordered an antipsychotic medication. D. Refer the patient to a support group with other military veterans.

ANS: A, D Collaborating with the patient demonstrates an environment of mutual respect and is helpful in establishing a trusting relationship. Both of these are identified as essential in effective treatment of PTSD. Evidence also supports that a group with other people who have experienced similar traumas is helpful in reducing the sense of isolation that some people with PTSD experience. Items B and C are incorrect since they both reflect an inaccurate understanding of the dynamics of PTSD.

A client diagnosed with obsessive-compulsive disorder states, "I really think my future will improve because of my successful treatment choices. I'm going to make my life better." Which guiding principle of recovery has assisted this client? A. Recovery emerges from hope. B. Recovery is person-driven. C. Recovery occurs via many pathways. D. Recovery is holistic.

ANS: A. The SAMHSA lists the following as guiding principles for the recovery model: recovery emerges from hope; recovery is person-driven; recovery occurs via many pathways; recovery is holistic; recovery is supported by peers and allies; recovery is supported through relationship and social networks; recovery is culturally based and influenced; recovery is supported by addressing trauma; recovery involves individual, family, and community strengths and responsibility; and recovery is based on respect. This client has internalized hope. This hope is the catalyst of the recovery process. KEY: Cognitive Level: Application | Integrated Processes: Nursing Process: Evaluation | Client Need: Psychosocial Integrity

20. Joe, who recently lost both parents in a tragic automobile accident, has been diagnosed with an adjustment disorder after he struck a friend who told him he needed to get his feelings out. The stage of grieving that Joe is struggling with is ___________________.

ANS: Anger The stages of grieving include denial, anger, bargaining, depression, and resolution. Joe is expressing anger but in a way that is impairing his relationships with others.

14. An isolative client was admitted 4 days ago with a diagnosis of major depressive disorder. Which nursing statement would best motivate this client to attend a therapeutic group being held in the milieu? A. Well go to the day room when you are ready for group. B. Ill walk with you to the day room. Group is about to start. C. It must be difficult for you to attend group when you feel so bad. D. Let me tell you about the benefits of attending this group.

ANS: B A client diagnosed with major depressive disorder exhibits little to no motivation and must be actively directed by staff to participate in therapy. It is difficult for a severely depressed client to make decisions, and this function must be temporarily assumed by the staff.

2. A client is diagnosed with major depressive disorder. Which nursing diagnosis should a nurse assign to this client to address a behavioral symptom of this disorder? A. Altered communication R/T feelings of worthlessness AEB anhedonia B. Social isolation R/T poor self-esteem AEB secluding self in room C. Altered thought processes R/T hopelessness AEB persecutory delusions D. Altered nutrition: less than body requirements R/T high anxiety AEB anorexia

ANS: B A nursing diagnosis of social isolation R/T poor self-esteem AEB secluding self in room addresses a behavioral symptom of major depressive disorder. Other behavioral symptoms include psychomotor retardation, virtually nonexistent communication, maintaining a fetal position, and no personal hygiene and/or grooming.

12. A client who has been taking buspirone (BuSpar) as prescribed for 2 days is close to discharge. Which statement indicates to the nurse that the client has an understanding of important discharge teaching? A. I cannot drink any alcohol with this medication. B. It is going to take 2 to 3 weeks in order for me to begin to feel better. C. This drug causes physical dependence, and I need to strictly follow doctors orders. D. I cant take this medication with food. It needs to be taken on an empty stomach.

ANS: B BuSpar takes at least 2 to 3 weeks to be effective in controlling symptoms of anxiety. This is important to teach clients in order to prevent potential noncompliance due to the perception that the medication is ineffective.

27. A newly admitted client diagnosed with major depressive disorder states, I have never considered suicide. Later the client confides to the nurse about plans to end it all by medication overdose. What is the most helpful nursing reply? A. There is nothing to worry about. We will handle it together. B. Bringing this up is a very positive action on your part. C. We need to talk about the things you have to live for. D. I think you should consider all your options prior to taking this action.

ANS: B By admitting to the staff a suicide plan, this client has taken responsibility for possible personal actions and expresses trust in the nurse. Therefore, the client may be receptive to continuing a safety plan. Recognition of this achievement reinforces this adaptive behavior.

15. A client who is diagnosed with major depressive disorder asks the nurse what causes depression. Which of these is the most accurate response? A. Depression is caused by a deficiency in neurotransmitters, including serotonin and norepinephrine. B. The exact cause of depressive disorders is unknown. A number of things, including genetic, biochemical, and environmental influences, likely play a role. C. Depression is a learned state of helplessness cause by ineffective parenting. D. Depression is caused by intrapersonal conflict between the id and the ego.

ANS: B Depression is likely an illness that has varied and multiple causative factors, but at present the exact cause of depressive disorders is not entirely understood.

26. A client admitted to the psychiatric unit following a suicide attempt is diagnosed with major depressive disorder. Which behavioral symptoms should the nurse expect to assess? A. Anxiety and unconscious anger B. Lack of attention to grooming and hygiene C. Guilt and indecisiveness D. Low self-esteem

ANS: B Lack of attention to grooming and hygiene is the only behavioral symptom presented. Lack of energy, low self-esteem, and feelings of helplessness and hopelessness (all common symptoms of depression) contribute to lack of attention to activities of daily living, including grooming and hygiene.

18. A nurse is implementing a one-on-one suicide observation level with a client diagnosed with major depressive disorder. The client states, Im feeling a lot better, so you can stop watching me. I have taken up too much of your time already. Which is the best nursing reply? A. I really appreciate your concern but I have been ordered to continue to watch you. B. Because we are concerned about your safety, we will continue to observe you. C. I am glad you are feeling better. The treatment team will consider your request. D. I will forward you request to your psychiatrist because it is his decision.

ANS: B Often suicidal clients resist personal monitoring, which impedes the implementation of a suicide plan. A nurse should continually observe a client when risk for suicide is suspected.

19. A newly admitted client is diagnosed with major depressive disorder with suicidal ideations. Which would be the priority nursing intervention for this client? A. Teach about the effect of suicide on family dynamics. B. Carefully and unobtrusively observe on the basis of assessed data, at varied intervals around the clock. C. Encourage the client to spend a portion of each day interacting within the milieu. D. Set realistic achievable goals to increase self-esteem.

ANS: B The most effective way to interrupt a suicide attempt is to carefully, unobtrusively observe on the basis of assessed data at varied intervals around the clock. If a nurse observes behavior that indicates self-harm, the nurse can intervene to stop the behavior and keep the client safe.

11. Which nursing behavior will enhance the establishment of a trusting relationship with a client diagnosed with schizophrenia? A. Establishing personal contact with family members. B. Being reliable, honest, and consistent during interactions. C. Sharing limited personal information. D. Sitting close to the client to establish rapport.

ANS: B The nurse can enhance the establishment of a trusting relationship with a client diagnosed with schizophrenia by being reliable, honest, and consistent during interactions. The nurse should also convey acceptance of the clients needs and maintain a calm attitude when dealing with agitated behavior.

7. During an admission assessment, a nurse asks a client diagnosed with schizophrenia, Have you ever felt that certain objects or persons have control over your behavior? The nurse is assessing for which type of thought disruption? A. Delusions of persecution B. Delusions of influence C. Delusions of reference D. Delusions of grandeur

ANS: B The nurse is assessing the client for delusions of influence when asking if the client has ever felt that objects or persons have control of the clients behavior. Delusions of control or influence are manifested when the client believes that his or her behavior is being influenced. An example would be if a client believes that a hearing aid receives transmissions that control personal thoughts and behaviors.

6. A client diagnosed with schizophrenia tells a nurse, The Shopatouliens took my shoes out of my room last night. Which is an appropriate charting entry to describe this clients statement? A. The client is experiencing command hallucinations. B. The client is expressing a neologism. C. The client is experiencing a paranoia. D. The client is verbalizing a word salad.

ANS: B The nurse should describe the clients statement as experiencing a neologism. A neologism is when a client invents a new word that is meaningless to others but may have symbolic meaning to the client. Word salad refers to a group of words that are put together randomly.

10. A client who has been taking fluvoxamine (Luvox) without significant improvement asks a nurse, I heard about something called a monoamine oxidase inhibitor (MAOI). Cant my doctor add that to my medications? Which is an appropriate nursing reply? A. This combination of drugs can lead to delirium tremens. B. A combination of an MAOI and Luvox can lead to a life-threatening hypertensive crisis. C. Thats a good idea. There have been good results with the combination of these two drugs. D. The only disadvantage would be the exorbitant cost of the MAOI.

ANS: B The nurse should explain to the client that combining an MAOI and Luvox can lead to a life-threatening hypertensive crisis. Symptoms of hypertensive crisis include severe occipital and/or temporal pounding headaches with occasional photophobia, sensations of choking, palpitations, and a feeling of dread.

25. A client has been recently admitted to an inpatient psychiatric unit. Which intervention should the nurse plan to use to reduce the clients focus on delusional thinking? A. Present evidence that supports the reality of the situation B. Focus on feelings suggested by the delusion C. Address the delusion with logical explanations D. Explore reasons why the client has the delusion

ANS: B The nurse should focus on the clients feelings rather than attempt to change the clients delusional thinking by the use of evidence or logical explanations. Delusional thinking is usually fixed, and clients will continue to have the belief in spite of obvious proof that the belief is false or irrational.

1. A paranoid client presents with bizarre behaviors, neologisms, and thought insertion. Which nursing action should be prioritized to maintain this clients safety? A. Assess for medication noncompliance B. Note escalating behaviors and intervene immediately C. Interpret attempts at communication D. Assess triggers for bizarre, inappropriate behaviors

ANS: B The nurse should note escalating behaviors and intervene immediately to maintain this clients safety.

16. A nurse is caring for a client who is experiencing a flat affect, paranoia, anhedonia, anergia, neologisms, and echolalia. Which statement correctly differentiates the clients positive and negative symptoms of schizophrenia? A. Paranoia, anhedonia, and anergia are positive symptoms of schizophrenia. B. Paranoia, neologisms, and echolalia are positive symptoms of schizophrenia. C. Paranoia, anergia, and echolalia are negative symptoms of schizophrenia. D. Paranoia, flat affect, and anhedonia are negative symptoms of schizophrenia.

ANS: B The nurse should recognize that positive symptoms of schizophrenia include paranoid delusions, neologisms, and echolalia. The negative symptoms of schizophrenia include flat affect, anhedonia, and anergia. Positive symptoms reflect an excess or distortion of normal functions. Negative symptoms reflect a decrease or loss of normal functions.

24. A college student has quit attending classes, isolates self because of hearing voices, and yells accusations at fellow students. Based on this information, which nursing diagnosis should the nurse prioritize? A. Altered thought processes R/T hearing voices AEB increased anxiety B. Risk for other-directed violence R/T yelling accusations C. Social isolation R/T paranoia AEB absence from classes D. Risk for self-directed violence R/T depressed mood

ANS: B The nursing diagnosis that must be prioritized in this situation is risk for other-directed violence R/T yelling accusations. Hearing voices and yelling accusations indicate a potential for violence, and this potential safety issue should be prioritized.

A mother who has a history of chronic heroin use has lost custody of her children due to abuse and neglect. She has been admitted to an inpatient substance-abuse program. Which client statement should a nurse associate with a positive prognosis for this client? A. "I'm not going to use heroin ever again. I know I've got the willpower to do it this time." B. "I cannot control my use of heroin. It's stronger than I am." C. "I'm going to get all my children back. They need their mother." D. "Once I deal with my childhood physical abuse, recovery should be easy."

ANS: B A positive prognosis is more likely when a client admits that he or she is addicted to a substance and has a loss of control. One of the first steps in accepting treatment is for the client to admit powerlessness over the substance.

A client states, "I have come to the conclusion that this disease has not paralyzed me." The nurse recognizes that this client is in which stage of the Psychological Recovery Model as described by Andersen and associates? A. Moratorium B. Awareness C. Preparation D. Rebuilding

ANS: B Andresen and associates have conceptualized a five-stage model of recovery called the Psychological Recovery Model. The stages include Stage 1, Moratorium; Stage 2, Awareness; Stage 3, Preparation; Stage 4, Rebuilding; and Stage 5, Growth. In the awareness stage, the individual comes to a realization that a possibility for recovery exists. Andresen and associates state, "It involves an awareness of a possible self other than that of 'sick person': a self that is capable of recovery." KEY: Cognitive Level: Analysis | Integrated Processes: Nursing Process: Evaluation | Client Need: Psychosocial Integrity

8. A client diagnosed with schizophrenia states, Cant you hear him? Its the devil. Hes telling me Im going to hell. Which is the most appropriate nursing reply? A. Did you take your medicine this morning? B. You are not going to hell. You are a good person. C. Im sure the voices sound scary. I dont hear any voices speaking. D. The devil only talks to people who are receptive to his influence.

ANS: C The most appropriate reply by the nurse is to reassure the client with an accepting attitude while not reinforcing the hallucination.

A 27-year-old client was diagnosed 5 years ago with schizophrenia. What course of treatment should the nurse expect to be implemented? A. Eventual admission for long-term care in a psychiatric facility B. Community-based care with numerous brief hospitalizations C. Case management in the community with few relapses D. Occasional contact with outpatient counselors and psychiatrists

ANS: B Community-based care is the standard of treatment that followed the deinstitutionalization movement. Schizophrenia is a chronic disease that includes both exacerbations and remissions in the course of the illness, leading to numerous brief hospitalizations. KEY: Cognitive Level: Application | Integrated Processes: Nursing Process: Implementation | Client Need: Psychosocial Integrity

10. Jane presents in the Emergency Department with a friend, who reports that Jane has been sitting in her apartment staring off into space and doesnt seem interested in doing anything. During the assessment Jane reveals, with little emotion, that she was raped 4 months ago. Which of these is the most appropriate interpretation of Janes lack of emotion? A. Jane is probably hearing voices telling her to be emotionless. B. Jane is experiencing numbing of emotional response, which is a common symptom of PTSD. C. Jane is trying to be secretive, and lying is a common symptom in PTSD. D. Jane is currently re-experiencing the traumatic event and is having a dissociative episode.

ANS: B General numbing of emotional response is a common symptom of PTSD. Items A and D are not the most appropriate interpretations because the data are inadequate to make that inference. Item C is incorrect; lying is not a common symptom in PTSD.

In assessing a client diagnosed with polysubstance abuse, the nurse should recognize that withdrawal from which substance may require a life-saving emergency intervention? A. Dextroamphetamine (Dexedrine) B. Diazepam (Valium) C. Morphine (Astramorph) D. Phencyclidine (PCP)

ANS: B If large doses of central nervous system (CNS) depressants (like Valium) are repeatedly administered over a prolonged duration, a period of CNS hyperexcitability occurs on withdrawal of the drug. The response can be quite severe, even leading to convulsions and death.

A nursing instructor is teaching about components present in the recovery process, as described by Andresen and associates, which led to the development of the Psychological Recovery Model. Which student statement indicates that further teaching is needed? A. "A client has a better chance of recovery if he or she truly believes that recovery can occur." B. "If a client is willing to give the responsibility of treatment to the health-care team, he or she is likely to recover." C. "A client who has a positive sense of self and a positive identity is likely to recover." D. "A client has a better chance of recovery if he or she has purpose and meaning in life."

ANS: B In examining a number of studies, Andresen and associates identified four components that were consistently evident in the recovery process. These components are hope, responsibility, self and identity, and meaning and purpose. Under responsibility, this model tasks the client, not the health-care team, with taking responsibility for his or her life and well-being. KEY: Cognitive Level: Analysis | Integrated Processes: Nursing Process: Evaluation | Client Need: Psychosocial Integrity

17. Brandy is an 18-year-old being treated in the Community Mental Health Clinic for an adjustment disorder after receiving news of her parents impending divorce. While talking about her feelings she becomes angry and starts shouting and crying. She screams, I wish they would both die! Which of these is the most appropriate response by the nurse at this point? A. Contact the parents and the police to report that Brandy is expressing homicidal ideation. B. Encourage Brandy to talk more about her anger. C. Instruct Brandy that its okay to cry but that it is not acceptable to talk that way about her parents. D. Assess Brandy for suicidal ideation.

ANS: B It is important in treating patients with adjustment disorders to allow them to express anger. Item C discourages the patient from expressing anger. Items A and D would be premature, since there is inadequate evidence to warrant those responses.

A community health nurse is teaching a class to expectant parents. All participants lack infant care knowledge. A student nurse asks, "If you had to assign a nursing diagnosis to this group, what would it be?" What is the best nursing reply? A. "I would assign the nursing diagnosis of cognitive deficit." B. "I would assign the nursing diagnosis of knowledge deficit." C. "I would assign the nursing diagnosis of altered family processes." D. "I would assign the nursing diagnosis of risk for caregiver role strain."

ANS: B Knowledge deficit is defined as the absence or deficiency of cognitive information related to a specific topic. Cognitive deficit would indicate an alteration in the ability to process information, and this evidence is not provided in the question. KEY: Cognitive Level: Analysis | Integrated Processes: Nursing Process: Analysis | Client Need: Health Promotion and Maintenance

Which symptom should a nurse identify that would differentiate clients diagnosed with neurocognitive disorders from clients diagnosed with amnesic disorders? A. Neurocognitive disorders involve disorientation that develops suddenly, whereas amnestic disorders develop more slowly. B. Neurocognitive disorders involve impairment of abstract thinking and judgment, whereas amnestic disorders do not. C. Neurocognitive disorders include the symptom of confabulation, whereas amnestic disorders do not. D. Both neurocognitive disorders and profound amnesia typically share the symptom of disorientation to place, time, and self.

ANS: B Neurocognitive disorders involve impairment of abstract thinking and judgment. Amnestic disorders are characterized by an inability to learn new information and to recall previously learned information, with no impairment in higher cortical functioning or personality change. KEY: Cognitive Level: Analysis | Integrated Processes: Nursing Process: Assessment | Client Need: Physiological Integrity

A client diagnosed with alcohol abuse disorder is referred to a residential care facility after discharge. According to the SAMHSA, which dimension of recovery is supporting this client? A. Health B. Home C. Purpose D. Community

ANS: B SAMHSA describes the dimension of Home as a stable and safe place to live. KEY: Cognitive Level: Application | Integrated Processes: Nursing Process: Assessment | Client Need: Psychosocial Integrity

A nursing instructor is teaching about the guiding principles of the recovery model, as described by the SAMHSA. Which student statement indicates that further teaching is needed? A. "Recovery occurs via many pathways." B. "Recovery emerges from strong religious affiliations." C. "Recovery is supported by peers and allies." D. "Recovery is culturally based and influenced."

ANS: B SAMHSA lists the following as guiding principles for the recovery model: recovery emerges from hope; recovery is person-driven; recovery occurs via many pathways; recovery is holistic; recovery is supported by peers and allies; recovery is supported through relationship and social networks; recovery is culturally based and influenced; recovery is supported by addressing trauma; recovery involves individual, family, and community strengths and responsibility; and recovery is based on respect. Recovery emerges from hope, but affiliation with any particular religion would have little bearing on the recovery process. KEY: Cognitive Level: Application | Integrated Processes: Nursing Process: Evaluation | Client Need: Psychosocial Integrity

Which is the priority nursing intervention for a client admitted for acute alcohol intoxication? A. Darken the room to reduce stimuli in order to prevent seizures. B. Assess aggressive behaviors in order to intervene to prevent injury to self or others. C. Administer lorazepam (Ativan) to reduce the rebound effects on the central nervous system. D. Teach the negative effects of alcohol on the body.

ANS: B Symptoms associated with the syndrome of alcohol intoxication include but are not limited to aggressiveness, impaired judgment, impaired attention, and irritability. Safety is a nursing priority in this situation.

A nurse maintains a client's confidentiality, addresses the client appropriately, and does not discriminate on the basis of gender, age, race, or religion. Which guiding principle of recovery has this nurse employed? A. Recovery is culturally based and influenced. B. Recovery is based on respect. C. Recovery involves individual, family, and community strengths and responsibility. D. Recovery is person-driven.

ANS: B The SAMHSA lists the following as guiding principles for the recovery model: recovery emerges from hope; recovery is person-driven; recovery occurs via many pathways; recovery is holistic; recovery is supported by peers and allies; recovery is supported through relationship and social networks; recovery is culturally based and influenced; recovery is supported by addressing trauma; recovery involves individual, family, and community strengths and responsibility; and recovery is based on respect. This nurse accepts and appreciates clients who are affected by mental health and substance use problems. This nurse protects the rights of clients and does not discriminate against them. KEY: Cognitive Level: Application | Integrated Processes: Nursing Process: Implementation | Client Need: Psychosocial Integrity

A nursing instructor is teaching about recovery as it applies to mental illness. Which student statement indicates that further teaching is needed? A. "The goal of recovery is improved health and wellness." B. "The goal of recovery is expedient, comprehensive behavioral change." C. "The goal of recovery is the ability to live a self-directed life." D. "The goal of recovery is the ability to reach full potential."

ANS: B The Substance Abuse and Mental Health Services Administration (SAMHSA) defines recovery from mental health disorders and substance use disorders as a process of change through which individuals improve their health and wellness, live a self-directed life, and strive to reach their full potential. Change in recovery is not an expedient process. It occurs incrementally over time. KEY: Cognitive Level: Application | Integrated Processes: Nursing Process: Evaluation | Client Need: Psychosocial Integrity

Which is the priority focus of recovery models? A. Empowerment of the health-care team to bring their expertise to decision-making B. Empowerment of the client to make decisions related to individual health care C. Empowerment of the family system to provide supportive care D. Empowerment of the physician to provide appropriate treatments

ANS: B The basic concept of a recovery model is empowerment of the client. The recovery model is designed to allow clients primary control over decisions about their own care. KEY: Cognitive Level: Analysis | Integrated Processes: Nursing Process: Assessment | Client Need: Psychosocial Integrity

A client diagnosed with schizophrenia is hospitalized because of an exacerbation of psychosis related to antipsychotic medication nonadherence. Which level of care does the client's hospitalization reflect? A. Primary prevention level of care B. Secondary prevention level of care C. Tertiary prevention level of care D. Case management level of care

ANS: B The client's hospitalization reflects the secondary prevention level of care. Secondary prevention aims at minimizing symptoms and is accomplished through early identification of problems and prompt initiation of effective treatment. KEY: Cognitive Level: Application | Integrated Processes: Nursing Process: Implementation | Client Need: Safe and Effective Care Environment

A nursing instructor is teaching about case management. What student statement indicates that learning has occurred? A. "Case management is a method used to achieve independent client care." B. "Case management provides coordination of services required to meet client needs." C. "Case management exists to facilitate client admission to needed inpatient services." D. "Case management is a method to facilitate physician reimbursement."

ANS: B The instructor evaluates that learning has occurred when a student defines case management as providing coordination of services required to meet client needs. Case management strives to organize client care so that specific outcomes are achieved within allotted time frames. KEY: Cognitive Level: Application | Integrated Processes: Nursing Process: Evaluation | Client Need: Psychosocial Integrity

4. Parents ask a nurse how they should reply when their child, diagnosed with schizophrenia, tells them that voices command him to harm others. Which is the appropriate nursing reply? A. Tell him to stop discussing the voices. B. Ignore what he is saying, while attempting to discover the underlying cause. C. Focus on the feelings generated by the hallucinations and present reality. D. Present objective evidence that the voices are not real.

ANS: C The most appropriate response by the nurse is to instruct the parents to focus on the feelings generated by the hallucinations and present reality. The parents should maintain an attitude of acceptance to encourage communication but should not reinforce the hallucinations by exploring details of content. It is inappropriate to present logical arguments to persuade the client to accept the hallucinations as not real.

A nurse evaluates a client's patient-controlled analgesia (PCA) pump and notices 100 attempts within a 30-minute period. Which is the best rationale for assessing this client for substance dependence? A. Narcotic pain medication is contraindicated for all clients with active substance-abuse problems. B. Clients who are dependent on alcohol or benzodiazepines may have developed cross-tolerance to analgesics and require increased doses to achieve effective pain control. C. There is no need to assess the client for substance dependence. There is an obvious PCA malfunction. D. The client is experiencing symptoms of withdrawal and needs to be accurately assessed for lorazepam (Ativan) dosage.

ANS: B The nurse should assess the client for substance dependence because clients who are dependent on alcohol or benzodiazepines may have developed cross-tolerance to analgesics, and require increased doses to achieve effective pain control. Cross-tolerance occurs when one drug lessened the client's response to another drug.

A nurse is reviewing STAT laboratory data of a client presenting in the emergency department. At what minimum blood alcohol level should a nurse expect intoxication to occur? A. 50 mg/dL B. 100 mg/dL C. 250 mg/dL D. 300 mg/dL

ANS: B The nurse should expect that 100 mg/dL is the minimum blood alcohol level at which intoxication occurs. Intoxication usually occurs between 100 and 200 mg/dL. Death has been reported at levels ranging from 400 to 700 mg/dL.

A client with a history of cerebrovascular accident (CVA) is brought to an emergency department experiencing memory problems, confusion, and disorientation. On the basis of this client's assessment data, which diagnosis would the nurse expect the physician to assign? A. Medication-induced delirium B. Vascular neurocognitive disorder C. Altered thought processes D. Alzheimer's disease

ANS: B The nurse should expect that this client would be diagnosed with vascular neurocognitive disorder (NCD), which is due to significant cerebrovascular disease. Vascular NCD often has an abrupt onset. This disease often occurs in a fluctuating pattern of progression. KEY: Cognitive Level: Application | Integrated Processes: Nursing Process: Assessment | Client Need: Physiological Integrity

A client with a history of heavy alcohol use is brought to an emergency department (ED) by family members who state that the client has had nothing to drink in the last 24 hours. Which client symptom should the nurse immediate report to the ED physician? A. Tactile hallucinations B. Blood pressure of 180/100 mm Hg C. Mood rating of 2/10 on numeric scale D. Dehydration

ANS: B The nurse should recognize that high blood pressure is a symptom of alcohol withdrawal and should promptly report this finding to the physician. Complications associated with alcohol withdrawal may progress to alcohol withdrawal delirium and possible seizure activity on about the second or third day following cessation of prolonged alcohol consumption.

A homeless client comes to an emergency department reporting cough, night sweats, weight loss, and blood-tinged sputum. What disease that has recently become more prevalent among the homeless community should a nurse suspect? A. Meningitis B. Tuberculosis C. Encephalopathy D. Mononucleosis

ANS: B The nurse should suspect that the homeless client has contracted tuberculosis. Tuberculosis is a growing problem among homeless individuals because of being in crowded shelters, which are ideal conditions for the spread of respiratory tuberculosis. Alcoholism, drug addiction, HIV infection, and poor nutrition also contribute to the increase in cases of tuberculosis. KEY: Cognitive Level: Application | Integrated Processes: Nursing Process: Assessment | Client Need: Physiological Integrity

16. Major Smith, who is being treated for PTSD symptoms following a course of military duty, reports, I think I was in denial about even having PTSD. I thought I was just having trouble sleeping. Which of these is an accurate evaluation of the patients comments? A. The patient is still in denial and unable to recognize that he is having flashbacks rather than insomnia. B. The patient is beginning to recognize stages of grieving and reevaluating his symptoms. C. The patient is beginning to recognize that he may be at risk for suicide. D. The patient is trying to avoid discussing symptoms of PTSD.

ANS: B The patient is expressing recognition that he was in denial, which is a stage of grieving. It is not uncommon for people to recognize that they are having troubling symptoms but not immediately recognize this as PTSD.

15. A patient being treated for symptoms of PTSD following a shooting incident at a local elementary school reports I feel like theres no reason to go on living when so many others died. Which of these is the most appropriate response by the nurse at this juncture? A. Youve got lots of reasons to go on living B. Are you having thoughts of hurting or killing yourself? C. Youre just experiencing survivor guilt. D. There must be something that gives you hope.

ANS: B This patient is expressing hopelessness, and it is a priority to assess for suicide ideation in these circumstances. Items A and D minimize the patients experience of feeling hopelessness. Item C may be a useful strategy to encourage the patient that this is a common experience of trauma survivors, but the immediate priority is determining patient safety.

29. An individual experiences sadness and melancholia in September continuing through November. Which of the following factors should a nurse identify as most likely to contribute to the etiology of these symptoms? Select all that apply. A. Gender differences in social opportunities that occur with age B. Drastic temperature and barometric pressure changes C. Increased levels of melatonin D. Variations in serotonergic functioning E. Inaccessibility of resources for dealing with life stressors

ANS: B, C, D The nurse should identify drastic temperature and barometric pressure changes, increased levels of melatonin, and/or variations in serotonergic functioning as contributing to the etiology of the clients symptoms. A number of studies have examined seasonal patterns associated with mood disorders and have revealed two prevalent periods of seasonal involvement: spring (March, April, May) and fall (September, October, November).

20. An elderly client diagnosed with schizophrenia takes an antipsychotic and a beta-adrenergic blocking agent (propranolol) for hypertension. Understanding the combined side effects of these drugs, the nurse would most appropriately make which statement? A. Make sure you concentrate on taking slow, deep, cleansing breaths. B. Watch your diet and try to engage in some regular physical activity. C. Rise slowly when you change position from lying to sitting or sitting to standing. D. Wear sunscreen and try to avoid midday sun exposure.

ANS: C The most appropriate statement by the nurse is to instruct the client to rise slowly when changing positions. Antipsychotic medications and beta blockers cause a decrease in blood pressure. When given in combination, this side effect places the client at risk for developing orthostatic hypotension.

1. A mother brings her son to the Emergency Department and tells the nurse that her son must have PTSD, because 2 days ago he witnessed a car accident in which there were fatalities. She is convinced that her son has PTSD because he has been crying when he talks about the incident. She believes that boys are at greater risk for PTSD because they don't typically cry. She read on the internet that PTSD can have dangerous consequences, so she wants her son to get some medication to cure the PTSD before it gets too bad. Which of these statements by the nurse would accurately correct this mothers misunderstanding about PTSD? Select all that apply. A. There are no long-term or dangerous consequences from PTSD. B. Women appear to be at greater risk of this disorder than men. C. Medications have been found to be effective in treating symptoms of depression or anxiety but do not represent a cure for the disorder. D. Fewer than 10% of trauma victims develop PTSD.

ANS: B, C, D Items B, C, and D are evidence-based pieces of information. Item A is incorrect since, in fact, dangerous consequences of unmanaged PTSD may include depression and/or suicide.

Which of the following clients should a nurse recommend for a structured day program? Select all that apply. A. An acutely suicidal teenager who has had three previous suicide attempts B. A chronically mentally ill woman who has a history of medication noncompliance C. An elderly individual with end-stage Alzheimer's disease D. A depressed individual who is able to participate in a safety plan E. A client who is hearing voices that tell him or her to harm others

ANS: B, D The nurse should recommend a structured day program for a chronically mental ill woman who has a history of medication noncompliance and for a depressed individual who is able to participate in a safety plan. Day programs (also called partial hospitalizations) are designed to prevent institutionalization or to ease the transition from inpatient hospitalization to community living. KEY: Cognitive Level: Application | Integrated Processes: Nursing Process: Evaluation | Client Need: Safe and Effective Care Environment: Management of Care

8. A nurse admits an older client who is experiencing memory loss, confused thinking, and apathy. A psychiatrist suspects depression. What is the rationale for performing a mini-mental status exam? A. To rule out bipolar disorder B. To rule out schizophrenia C. To rule out neurocognitive disorder D. To rule out a personality disorder

ANS: C A mini-mental status exam should be performed to rule out neurocognitive disorder. The elderly are often misdiagnosed with neurocognitive disorder such as Alzheimers disease, when depression is their actual diagnosis. Memory loss, confused thinking, and apathy are common symptoms of depression in the elderly.

22. A 75-year-old client with a long history of depression is currently on doxepin (Sinequan), 100 mg daily. The client takes a daily diuretic for hypertension and is recovering from the flu. Which nursing diagnosis should the nurse assign highest priority? A. Risk for ineffective thermoregulation R/T anhidrosis B. Risk for constipation R/T excessive fluid loss C. Risk for injury R/T orthostatic hypotension D. Risk for infection R/T suppressed white blood cell count

ANS: C A side effect of Sinequan is orthostatic hypotension. Dehydration due to fluid loss from a combination of diuretic medication and flu symptoms can also contribute to this problem, putting this client at risk for injury R/T orthostatic hypotension.

27. A newly admitted client has taken thioridazine (Mellaril) for 2 years, with good symptom control. Symptoms exhibited on admission included paranoia and hallucinations. The nurse should recognize which potential cause for the return of these symptoms? A. The client has developed tolerance to the antipsychotic medication. B. The client has not taken the medication with food. C. The client has not taken the medication as prescribed. D. The client has combined alcohol with the medication.

ANS: C Altered thinking can affect a clients insight into the necessity for taking antipsychotic medications consistently. When symptoms are no longer bothersome, clients may stop taking medications that cause disturbing side effects. Clients may miss the connection between taking the medications and an improved symptom profile.

20. The nurse is providing counseling to clients diagnosed with major depressive disorder. The nurse chooses to help the clients alter their mood by learning how to change the way they think. The nurse is functioning under which theoretical framework? A. Psychoanalytic theory B. Interpersonal theory C. Cognitive theory D. Behavioral theory

ANS: C Cognitive theory suggests that depression is a product of negative thinking. Helping the individual change the way they think is believed to have a positive impact on mood and self-esteem.

26. A client states, I hear voices that tell me that I am evil. Which outcome related to these symptoms should the nurse expect this client to accomplish by discharge? A. The client will verbalize the reason the voices make derogatory statements. B. The client will not hear auditory hallucinations. C. The client will identify events that increase anxiety and illicit hallucinations. D. The client will positively integrate the voices into the clients personality structure.

ANS: C It is unrealistic to expect the client to completely stop hearing voices. Even when compliant with antipsychotic medications, clients may still hear voices. It would be realistic to expect the client to associate stressful events with an increase in auditory hallucinations. By this recognition the client can anticipate symptoms and initiate appropriate coping skills.

A client is in the late stage of Alzheimer's disease. To address the client's symptoms, which nursing intervention should take priority? A. Improve cognitive status by encouraging involvement in social activities. B. Decrease social isolation by providing group therapies. C. Promote dignity by providing comfort, safety, and self-care measures. D. Facilitate communication by providing assistive devices.

ANS: C KEY: Cognitive Level: Analysis | Integrated Processes: Nursing Process: Implementation | Client Need: Physiological Integrity: Basic Care and Comfort

5. A depressed client reports to a nurse a history of divorce, job loss, family estrangement, and cocaine abuse. According to learning theory, what is the cause of this clients symptoms? A. Depression is a result of anger turned inward. B. Depression is a result of abandonment. C. Depression is a result of repeated failures. D. Depression is a result of negative thinking.

ANS: C Learning theory describes a model of learned helplessness in which multiple life failures cause the client to abandon future attempts to succeed.

12. A client diagnosed with schizophrenia states, My psychiatrist is out to get me. Im sad that the voice is telling me to stop him. What symptom is the client exhibiting, and what is the nurses legal responsibility related to this symptom? A. Magical thinking; administer an antipsychotic medication B. Persecutory delusions; orient the client to reality C. Command hallucinations; warn the psychiatrist D. Altered thought processes; call an emergency treatment team meeting

ANS: C The nurse should determine that the client is exhibiting command hallucinations. The nurses legal responsibility is to warn the psychiatrist of the potential for harm. A client who is demonstrating a risk for violence could potentially become physically, emotionally, and/or sexually harmful to others or to self.

14. A client diagnosed with schizophrenia is slow to respond and appears to be listening to unseen others. Which medication should a nurse expect a physician to order to address this type of symptom? A. Haloperidol (Haldol) to address the negative symptom B. Clonazepam (Klonopin) to address the positive symptom C. Risperidone (Risperdal) to address the positive symptom D. Clozapine (Clozaril) to address the negative symptom

ANS: C The nurse should expect the physician to order risperidone (Risperdal) to address the positive symptoms of schizophrenia. Risperidone (Risperdal) is an atypical antipsychotic used to reduce positive symptoms, including disturbances in content of thought (delusions), form of thought (neologisms), or sensory perception (hallucinations).

2. A client diagnosed with schizoaffective disorder is admitted for social skills training. Which information should be taught by the nurse? A. The side effects of medications B. Deep breathing techniques to decrease stress C. How to make eye contact when communicating D. How to be a leader

ANS: C The nurse should plan to teach the client how to make eye contact when communicating. Social skills, such as making eye contact, can assist clients in communicating needs and maintaining

9. A client diagnosed with brief psychotic disorder tells a nurse about voices telling him to kill the president. Which nursing diagnosis should the nurse prioritize for this client? A. Disturbed sensory perception B. Altered thought processes C. Risk for violence: directed toward others D. Risk for injury

ANS: C The nurse should prioritize the diagnosis risk for violence: directed toward others. A client who hears voices telling him to kill someone is at risk for responding and reacting to the command hallucination. Other risk factors for violence include aggressive body language, verbal aggression, catatonic excitement, and rage reactions.

15. A client is diagnosed with schizophrenia. A physician orders haloperidol (Haldol), 50 mg bid; benztropine (Cogentin), 1 mg prn; and zolpidem (Ambien), 10 mg HS. Which client behavior would warrant the nurse to administer benztropine? A. Tactile hallucinations B. Tardive dyskinesia C. Restlessness and muscle rigidity D. Reports of hearing disturbing voices

ANS: C The symptom of tactile hallucinations and reports of hearing disturbing voices would be addressed by an antipsychotic medication such as haloperidol. Tardive dyskinesia, a potentially irreversible condition, would warrant the discontinuation of an antipsychotic medication such as haloperidol. An anticholinergic medication such as benztropine would be used to treat the extrapyramidal symptoms of restlessness and muscle rigidity.

16. What client information does a nurse need to assess prior to initiating medication therapy with phenelzine (Nardil)? A. The clients understanding of the need for regular bloodwork B. The clients mood and affect score, according to the facilitys mood scale C. The clients cognitive ability to understand information about the medication D. The clients access to a support network willing to participate in treatment

ANS: C There are many dietary and medication restrictions when taking Nardil. A client must have the cognitive ability to understand information about the medication and which foods, beverages, and medications to eliminate when taking Nardil.

19. A client diagnosed with schizophrenia takes an antipsychotic agent daily. Which assessment finding should a nurse immediately report to the clients attending psychiatrist? A. Respirations of 22 beats/minute B. Weight gain of 8 pounds in 2 months C. Temperature of 104F (40C) D. Excessive salivation

ANS: C When assessing a client diagnosed with schizophrenia who takes an antipsychotic agent daily, the nurse should immediately address a temperature of 104F (40C). A temperature this high can be a symptom of the rare but life-threatening neuroleptic malignant syndrome.

A nursing instructor is teaching students about the Community Health Centers Act of 1963. What was a deterring factor to the proper implementation of this act? A. Many perspective clients did not meet criteria for mental illness diagnostic-related groups. B. Zoning laws discouraged the development of community mental health centers. C. States could not match federal funds to establish community mental health centers. D. There was not a sufficient employment pool to staff community mental health centers.

ANS: C A deterring factor to the proper implementation of the Community Mental Health Centers Act of 1963 was that states could not match federal funds to establish community mental health centers. This act called for the construction of comprehensive community mental health centers to offset the effects of deinstitutionalization caused by the closing of state mental health hospitals. KEY: Cognitive Level: Application | Integrated Processes: Nursing Process: Evaluation | Client Need: Safe and Effective Care Environment

A nurse on an inpatient unit helps a client understand the significance of treatments and provides the client with copies of all documents related to the plan of care. This nurse is employing which commitment in the "Tidal Model of Recovery?" A. Know that Change Is Constant B. Reveal Personal Wisdom C. Be Transparent D. Give the Gift of Time

ANS: C Barker & Buchanan-Barker developed a set of essential values termed The 10 Tidal Commitments, upon which the Tidal Model is based. They include Value the Voice, Respect the Language, Develop Genuine Curiosity, Become the Apprentice, Use the Available Toolkit, Craft the Step Beyond, Give the Gift of Time, Reveal Personal Wisdom, Know that Change Is Constant, and Be Transparent. This nurse is employing the Be Transparent commitment. KEY: Cognitive Level: Application | Integrated Processes: Nursing Process: Implementation | Client Need: Psychosocial Integrity

11. Studies have suggested that re-experiencing a traumatic event can become an addiction of sorts. The evidence suggests that the reason for this is: A. People with PTSD often have addictive personalities. B. Perpetuating the traumatic experience yields secondary gains. C. The re-experiencing of trauma enhances production of endogenous opioid peptides. D. People with PTSD often have concurrent substance abuse issues.

ANS: C Hollander and Simeon (2008) report on studies suggesting that the release of endogenous opioid peptides can produce an addiction to the trauma. There is no evidence suggesting that addictive personality traits are responsible for chronicity in PTSD symptoms. Items B and D are possible outcomes in any individual with PTSD, but neither has been correlated to an addiction to re-experiencing trauma

A recovering alcoholic relapses and drinks a glass of wine. The client presents in the emergency department (ED) experiencing severe throbbing headache, tachycardia, flushed face, dyspnea, and continuous vomiting. What may these symptoms indicate to the ED nurse? A. Alcohol poisoning B. Cardiovascular accident (CVA) C. A reaction to disulfiram (Antabuse) D. A reaction to tannins in the red wine

ANS: C Ingestion of alcohol while disulfiram is in the body results in a syndrome of symptoms that can produce a good deal of discomfort for the individual. Symptoms may include but are not limited to flushed skin, throbbing in the head and neck, respiratory difficulty, dizziness, nausea and vomiting, confusion, hypotension, and tachycardia.

12. Sandy, a rape survivor, is being treated for PTSD. Which of these statements are good indications that Sally is beginning to recover from PTSD? A. I still have nightmares every night, but I dont always remember them anymore. B. Im not drinking as much alcohol as I had been over the last several months. C. This traumatic event immobilized me for awhile, but I have found imagery helpful in reducing my anxiety. D. All of the above.

ANS: C Item C demonstrates evidence of awareness of the impact the trauma had on Sandys life and demonstrates evidence of effective coping skills. Item A indicates continued presence of symptoms and possibly amnesia. Although item B may be evidence of a positive coping strategy, evaluation of recovery from PTSD must also include assessment for less symptoms such as nightmares and flashbacks.

SSRIs, SNRIs, TCAs and heterocyclics work by..

Block reuptake of norepinephrine, serotonin, and/or dopamine

The nurse believes that a client being admitted for a surgical procedure may have a drinking problem. How should the nurse further evaluate this possibility? A. By asking directly if the client has ever had a problem with alcohol B. By holistically assessing the client using the CIWA scale C. By using a screening tool such as the CAGE questionnaire D. By referring the client for physician evaluation

ANS: C The CAGE questionnaire is a screening tool used to determine the diagnosis of alcoholism. This questionnaire is composed of four simple questions. Scoring two or three "yes" answers strongly suggests a problem with alcohol.

A client's wife has been making excuses for her alcoholic husband's work absences. In family therapy, she states, "His problems at work are my fault." Which is the appropriate nursing response? A. "Why do you assume responsibility for his behaviors?" B. "Codependency is a typical behavior of spouses of alcoholics." C. "Your husband needs to deal with the consequences of his drinking." D. "Do you understand what the term 'enabler' means?"

ANS: C The appropriate nursing response is to use confrontation with caring. The nurse should understand that the client's wife may be in denial and enabling the husband's behavior. Partners of clients with substance abuse must come to realize that the only behavior they can control is their own.

A client diagnosed with alcohol abuse joins a community 12-step program and states, "My life is unmanageable." How should the nurse interpret this client's statement? A. The client is using minimization as an ego defense. B. The client is ready to sign an Alcoholics Anonymous contract for sobriety. C. The client has accomplished the first of 12 steps advocated by Alcoholics Anonymous. D. The client has met the requirements to be designated as an Alcoholics Anonymous sponsor.

ANS: C The first step of the 12-step program advocated by Alcoholics Anonymous is that clients must admit powerlessness over alcohol and that their lives have become unmanageable.

A client diagnosed with a neurocognitive disorder is exhibiting behavioral problems on a daily basis. At change of shift, the client's behavior escalates from pacing to screaming and flailing. Initially, which action should a nurse implement in this situation? A. Consult the psychologist regarding behavior-modification techniques. B. Medicate the client with prn antianxiety medications. C. Assess environmental triggers and potential unmet needs. D. Anticipate the behavior and restrain when pacing begins.

ANS: C The initial nursing action is to assess environmental triggers and potential unmet needs. Due to the cognitive decline experienced in a client diagnosed with neurocognitive disorder, communication skills may be limited. The client may become disoriented and frustrated. KEY: Cognitive Level: Application | Integrated Processes: Nursing Process: Implementation | Client Need: Psychosocial Integrity

A newly admitted homeless client diagnosed with schizophrenia states, "I have been living in a cardboard box for 2 weeks. Why did the government let me down?" Which is an appropriate nursing reply? A. "Your discharge from the state hospital was done prematurely. Had you remained in the state hospital longer, you would not be homeless." B. "Your premature discharge from the state hospital was not intended for patients diagnosed with chronic schizophrenia." C. "Your discharge from the state hospital was based on firm principles; however, the resources were not available to make the transition a success." D. "Your discharge from the state hospital was based on presumed family support, and this was not forthcoming."

ANS: C The most accurate nursing reply is to explain to the client that the resources were not available for successful transitioning out of a state hospital to the community. There are several factors that are thought to contribute to homelessness among the mentally ill: deinstitutionalization, poverty, lack of affordable housing, lack of affordable health care, domestic violence, and addiction disorders. KEY: Cognitive Level: Application | Integrated Processes: Nursing Process: Implementation | Client Need: Psychosocial Integrity

Upon admission for symptoms of alcohol withdrawal a client states, "I haven't eaten in 3 days." Assessment reveals BP 170/100 mm Hg, P 110, R 28, and T 97F (36C) with dry skin, dry mucous membranes, and poor skin turgor. What should be the priority nursing diagnosis? A. Knowledge deficit B. Fluid volume excess C. Imbalanced nutrition: less than body requirements D. Ineffective individual coping

ANS: C The nurse should assess that the priority nursing diagnosis is imbalanced nutrition: less than body requirements. The client is exhibiting signs and symptoms of malnutrition as well as alcohol withdrawal. The nurse should consult a dietitian, restrict sodium intake to minimize fluid retention, and provide small, frequent feedings of nonirritating foods.

At what time during a 24-hour period should a nurse expect clients with Alzheimer's disease to exhibit more pronounced symptoms? A. When they first awaken B. In the middle of the night C. At twilight D. After taking medications

ANS: C The nurse should determine that clients with Alzheimer's disease exhibit more pronounced symptoms at twilight. Sundowning is the term used to describe the worsening of symptoms in the late afternoon and evening. KEY: Cognitive Level: Comprehension| Integrated Processes: Nursing Process: Assessment | Client Need: Physiological Integrity

Which symptom should a nurse identify that would differentiate clients diagnosed with neurocognitive disorders from clients with pseudodementia (depression)? A. Altered sleep B. Impaired attention and concentration C. Altered task performance D. Impaired psychomotor activity

ANS: C The nurse should identify that attention and concentration are impaired in neurocognitive disorder and not in pseudodementia (depression). KEY: Cognitive Level: Analysis | Integrated Processes: Nursing Process: Assessment | Client Need: Physiological Integrity

During group therapy, a client diagnosed with chronic alcohol dependence states, "I would not have boozed it up if my wife hadn't been nagging me all the time to get a job. She never did think that I was good enough for her." How should a nurse interpret this statement? A. The client is using denial by avoiding responsibility. B. The client is using displacement by blaming his wife. C. The client is using rationalization to excuse his alcohol dependence. D. The client is using reaction formation by appealing to the group for sympathy.

ANS: C The nurse should interpret that the client is using rationalization to excuse his alcohol dependence. Rationalization is the defense mechanism by which people avoid taking responsibility for their actions by making excuses for the behavior.

After 1 week of continuous mental confusion, an elderly African American client is admitted with a preliminary diagnosis of major neurocognitive disorder due to Alzheimer's disease. What should cause the nurse to question this diagnosis? A. Neurocognitive disorder does not typically occur in African American clients. B. The symptoms presented are more indicative of Parkinsonism. C. Neurocognitive disorder does not develop suddenly. D. There has been no T3 or T4 level evaluation ordered.

ANS: C The nurse should know that neurocognitive disorder (NCD) does not develop suddenly and should question this diagnosis. The onset of NCD symptoms is slow and insidious and is unrelated to race, culture, or creed. The disease is generally progressive and debilitating. KEY: Cognitive Level: Analysis | Integrated Processes: Nursing Process: Assessment | Client Need: Physiological Integrity

A client is experiencing progressive changes in memory that have interfered with personal, social, and occupational functioning. The client exhibits poor judgment and has a short attention span. A nurse should recognize these as classic signs of which condition? A. Mania B. Delirium C. Neurocognitive disorder D. Parkinsonism

ANS: C The nurse should recognize that the client is exhibiting signs of neurocognitive disorder (NCD). In NCD, impairment is evident in abstract thinking, judgment, and impulse control. Behavior may be uninhibited and inappropriate. KEY: Cognitive Level: Application | Integrated Processes: Nursing Process: Assessment | Client Need: Physiological Integrity

A nurse is implementing care within the parameters of tertiary prevention. Which nursing action is an example of this type of care? A. Teaching an adolescent about pregnancy prevention B. Teaching an elderly client the reportable side effects of a newly prescribed neuroleptic medication C. Teaching a client with schizophrenia to cook meals, make a grocery list, and establish a budget D. Teaching a client about his or her new diagnosis of bipolar disorder

ANS: C The nurse who teaches a client to cook meals, make a grocery list, and establish a budget is implementing care within the parameters of tertiary prevention. Tertiary prevention is services aimed at reducing the residual effects that are associated with severe and persistent mental illness. It is accomplished by promoting rehabilitation that is directed toward achievement of maximum functioning. KEY: Cognitive Level: Application | Integrated Processes: Nursing Process: Implementation | Client Need: Psychosocial Integrity

A client diagnosed with neurocognitive disorder due to Alzheimer's disease is disoriented and ataxic, and he wanders. Which is the priority nursing diagnosis? A. Disturbed thought processes B. Self-care deficit C. Risk for injury D. Altered health-care maintenance

ANS: C The priority nursing diagnosis for this client is risk for injury. Both ataxia (muscular incoordination) and purposeless wandering place the client at an increased risk for injury. KEY: Cognitive Level: Analysis | Integrated Processes: Nursing Process: Analysis | Client Need: Physiological Integrity: Reduction of Risk Potential

On the first day of a client's alcohol detoxification, which nursing intervention should take priority? A. Strongly encourage the client to attend 90 Alcoholics Anonymous meetings in 90 days. B. Educate the client about the biopsychosocial consequences of alcohol abuse. C. Administer ordered chlordiazepoxide (Librium) in a dosage according to protocol. D. Administer vitamin B1 to prevent Wernicke-Korsakoff syndrome.

ANS: C The priority nursing intervention for this client should be to administer ordered chlordiazepoxide (Librium) in a dosage according to protocol. Chlordiazepoxide (Librium) is a benzodiazepine and is often used for substitution therapy in alcohol withdrawal. Substitution therapy may be required to reduce life-threatening effects of the rebound stimulation of the central nervous system that occurs during withdrawal.

A client with a history of insomnia has been taking chlordiazepoxide (Librium) 15 mg at night for the past year. The client currently reports getting to sleep. Which nursing diagnosis appropriately documents this problem? A. Ineffective coping R/T unresolved anxiety AEB substance abuse B. Anxiety R/T poor sleep AEB difficulty falling asleep C. Disturbed sleep pattern R/T Librium tolerance AEB difficulty falling asleep D. Risk for injury R/T addiction to Librium

ANS: C Tolerance is defined as the need for increasingly larger or more frequent doses of a substance in order to obtain the desired effects originally produced by a lower dose.

When a home health nurse administers an outpatient's injection of haloperidol decanoate (Haldol decanoate), which level of care is the nurse providing? A. Primary prevention level of care B. Secondary prevention level of care C. Tertiary prevention level of care D. Case management level of care

ANS: C When administering this long-acting antipsychotic medication, the nurse is providing a tertiary prevention level of care. Tertiary prevention services are aimed at reducing the residual effects associated with severe and persistent mental illness. It is accomplished by preventing complications of the illness and promoting rehabilitation directed toward achievement of maximum functioning. KEY: Cognitive Level: Application | Integrated Processes: Nursing Process: Implementation | Client Need: Psychosocial Integrity

Which intervention should the nurse consider as primary prevention for an individual who is on the verge of being homeless because of a job layoff? A. Referral to primary care provider to improve general health status B. Encouraging client to recognize reasons for job layoff C. Job training to increase employment options D. Encouraging the use of prn medications to control symptoms

ANS: C When the nurse implements primary prevention interventions, the nurse is providing services aimed at reducing the incidences of mental disorders within the population. In this situation, there is emphasis on providing education and support to unemployed or homeless individuals. KEY: Cognitive Level: Application | Integrated Processes: Nursing Process: Implementation | Client Need: Psychosocial Integrity

19. According to NANDA (2012), a disorder that occurs after the death of a significant other or any loss perceived as significant to the individual, in which the experience of distress accompanying bereavement fails to follow normative expectations and manifests in functional impairment, is referred to as________________________.

ANS: Complicated grieving A grieving process that does not follow normative expectations may include fixation at a particular stage of grieving, psychosomatic symptoms, and/or impairment in occupational, social, intellectual, or emotional function.

7. A nurse is planning care for a child who is experiencing depression. Which medication is approved by the U.S. Food and Drug Administration (FDA) for the treatment of depression in children and adolescents? A. Paroxetine (Paxil) B. Sertraline (Zoloft) C. Citalopram (Celexa) D. Fluoxetine (Prozac)

ANS: D Fluoxetine (Prozac) is FDA approved for the treatment of depression in children and adolescents. Fluoxetine is a selective serotonin reuptake inhibitor (SSRI) used in the treatment of depression. All antidepressants carry an FDA warning for increased risk of suicide in children and adolescents.

21. Which client statement expresses a typical underlying feeling of clients diagnosed with major depressive disorder? A. Its just a matter of time and I will be well. B. If I ignore these feelings, they will go away. C. I can fight these feelings and overcome this disorder. D. Nothing will help me feel better.

ANS: D Hopelessness and helplessness are typical symptoms of clients diagnosed with major depressive disorder.

25. A nurse is caring for four clients taking various medications, including imipramine (Tofranil), doxepine (Sinequan), ziprasidone (Geodon), and tranylcypromine (Parnate). The nurse orders a special diet for the client receiving which medication? A. Tofranil B. Senequan C. Geodon D. Parnate

ANS: D Hypertensive crisis occurs in clients receiving a monoamine oxidase inhibitor (MAOI) who consume foods or drugs with a high tyramine content.

10. Which nursing intervention would be most appropriate when caring for an acutely agitated client with paranoia? A. Provide neon lights and soft music. B. Maintain continual eye contact throughout the interview. C. Use therapeutic touch to increase trust and rapport. D. Provide personal space to respect the clients boundaries.

ANS: D The most appropriate nursing intervention is to provide personal space to respect the clients boundaries. Providing personal space may serve to reduce anxiety and thus reduce the clients risk for violence.

5. A nurse is assessing a client diagnosed with schizophrenia. The nurse asks the client, Do you receive special messages from certain sources, such as the television or radio? Which potential symptom of this disorder is the nurse assessing? A. Thought insertion B. Paranoia C. Magical thinking D. Delusions of reference

ANS: D The nurse is assessing for the potential symptom of delusions of reference. A client who believes that he or she receives messages through the radio is experiencing delusions of reference. When a client experiences these delusions, he or she interprets all events within the environment as personal references.

17. A client diagnosed with major depressive disorder states, Ive been feeling down for 3 months. Will I ever feel like myself again? Which reply by the nurse will best assess this clients affective symptoms? A. Have you been diagnosed with any physical disorder within the last 3 months? B. Have you ever felt this way before? C. People who have mood changes often feel better when spring comes. D. Help me understand what you mean when you say, feeling down?

ANS: D The nurse is using a clarifying statement in order to gather more details related to this clients mood.

3. A nurse assesses a client suspected of having major depressive disorder. Which client symptom would eliminate this diagnosis? A. The client is disheveled and malodorous. B. The client refuses to interact with others. C. The client is unable to feel any pleasure. D. The client has maxed-out charge cards and exhibits promiscuous behaviors.

ANS: D The nurse should assess that a client who has maxed-out credit cards and exhibits promiscuous behavior would be exhibiting manic symptoms. According to the DSM-5, these symptoms would rule out the diagnosis of major depressive disorder.

1. A client is diagnosed with persistent depressive (dysthymia) disorder. Which should a nurse classify as an affective symptom of this disorder? A. Social isolation with a focus on self B. Low energy level C. Difficulty concentrating D. Gloomy and pessimistic outlook on life

ANS: D The nurse should classify a gloomy and pessimistic outlook on life as an affective symptom of dysthymia. Symptoms of depression can be described as alterations in four areas of human functions: affective, behavioral, cognitive, and physiological. Affective symptoms are those that relate to the mood.

17. A client diagnosed with schizophrenia, who has been taking antipsychotic medication for the last 5 months, presents in an emergency department (ED) with uncontrollable tongue movements, stiff neck, and difficulty swallowing. The nurse would expect the physician to recognize which condition and implement which treatment? A. Neuroleptic malignant syndrome, treated by discontinuing antipsychotic medications B. Agranulocytosis, treated by administration of clozapine (Clozaril) C. Extrapyramidal symptoms, treated by administration of benztropine (Cogentin) D. Tardive dyskinesia, treated by discontinuing antipsychotic medications

ANS: D The nurse should expect that an ED physician would diagnose the client with tardive dyskinesia and discontinue antipsychotic medications. Tardive dyskinesia is a condition of abnormal involuntary movements of the mouth, tongue, trunk, and extremities that can be an irreversible side effect of typical antipsychotic medications.

23. During an admission assessment, a nurse notes that a client diagnosed with schizophrenia has allergies to penicillin, prochlorperazine (Compazine), and bee stings. On the basis of this assessment data, which antipsychotic medication would be contraindicated? A. Haloperidol (Haldol), because it is used only in elderly patients B. Clozapine (Clozaril), because of a cross-sensitivity to penicillin C. Risperidone (Risperdal), because it exacerbates symptoms of depression D. Thioridazine (Mellaril), because of cross-sensitivity among phenothiazines

ANS: D The nurse should know that thioridazine (Mellaril) would be contraindicated because of cross-sensitivity among phenothiazines. Prochlorperazine (Compazine) and thioridazine are both classified as phenothiazines.

9. A confused client has recently been prescribed sertraline (Zoloft). The clients spouse is taking paroxetine (Paxil). The client presents with restlessness, tachycardia, diaphoresis, and tremors. What complication does a nurse suspect, and what could be its possible cause? A. Neuroleptic malignant syndrome caused by ingestion of two different serotonin reuptake inhibitors (SSRIs) B. Neuroleptic malignant syndrome caused by ingestion of an SSRI and a monoamine oxidase inhibitor (MAOI) C. Serotonin syndrome caused by ingestion of an SSRI and an MAOI D. Serotonin syndrome caused by ingestion of two different SSRIs

ANS: D The nurse should suspect that the client is suffering from serotonin syndrome possibly caused by ingesting two different SSRIs (Zoloft and Paxil). Symptoms of serotonin syndrome include confusion, agitation, tachycardia, hypertension, nausea, abdominal pain, myoclonus, muscle rigidity, fever, sweating, and tremor.

Which term should a nurse use to describe the administration of a central nervous system (CNS) depressant during alcohol withdrawal? A. Antagonist therapy B. Deterrent therapy C. Codependency therapy D. Substitution therapy

ANS: D A CNS depressant such as Ativan is used during alcohol withdrawal as substitution therapy to prevent life-threatening symptoms that occur because of the rebound reaction of the central nervous system.

Which client statement demonstrates positive progress toward recovery from substance abuse? A. "I have completed detox and therefore am in control of my drug use." B. "I will faithfully attend Narcotic Anonymous (NA) when I can't control my carvings." C. "As a church deacon, my focus will now be on spiritual renewal." D. "Taking those pills got out of control. It cost me my job, marriage, and children."

ANS: D A client who takes responsibility for the consequences of substance abuse/dependence is making positive progress toward recovery. This client would most likely be in the working phase of the counseling process in which acceptance of the fact that substance abuse causes problems occurs.

When a nurse attempts to provide health-care services to the homeless, what should be a realistic concern? A. Most individuals who are homeless reject help. B. Most individuals who are homeless are suspicious of anyone who offers help. C. Most individuals who are homeless are proud and will often refuse charity. D. Most individuals who are homeless relocate frequently.

ANS: D A realistic concern in the provision of health-care services to the homeless is that individuals who are homeless relocate frequently. Frequent relocation confounds service delivery and interferes with providers' efforts to ensure appropriate care. KEY: Cognitive Level: Application | Integrated Processes: Nursing Process: Evaluation | Client Need: Safe and Effective Care Environment

A client diagnosed with schizophrenia was released from a state mental hospital after 20 years of institutionalization. A nurse should recognize which characteristic that is likely to be exhibited by this client? A. The client is likely to be compliant with treatment because of institutional dependency. B. The client is likely to find a variety of community support services to aid in the transition. C. The client is likely to adjust to the community environment if given sufficient support. D. The client is likely to be admitted at some time to an acute care unit for psychiatric treatment.

ANS: D Because of the chronic nature of this client's diagnosis and commonly occurring medication noncompliance, the nurse would expect recidivism during the course of the illness. KEY: Cognitive Level: Application | Integrated Processes: Nursing Process: Evaluation | Client Need: Psychosocial Integrity

When intervening with a married couple experiencing relationship discord, which reflects a nursing intervention at the secondary level of prevention? A. Assessing how the children are coping with the parents' relationship issues B. Supplying the couple with guidelines related to marital seminar leadership C. Teaching the couple about various methods of birth control D. Counseling the couple in relation to open and honest communication skills

ANS: D Counseling the couple in relation to open and honest communication skills is reflective of a nursing intervention at the secondary level of prevention. Secondary prevention aims at minimizing symptoms and is accomplished through early identification of problems and prompt initiation of effective treatment. KEY: Cognitive Level: Analysis | Integrated Processes: Nursing Process: Implementation | Client Need: Psychosocial Integrity

A client diagnosed with vascular dementia is discharged to home under the care of his wife. Which information should cause the nurse to question the client's safety? A. His wife works from home in telecommunication. B. The client has worked the night shift his entire career. C. His wife has minimal family support. D. The client smokes one pack of cigarettes per day.

ANS: D Forgetfulness is an early symptom of dementia that would alert the nurse to question the client's safety at home if the client smokes cigarettes. Vascular dementia is a clinical syndrome of dementia due to significant cerebrovascular disease. The cause of vascular dementia is related to an interruption of blood flow to the brain. High blood pressure and hypertension are significant factors in the etiology. KEY: Cognitive Level: Analysis | Integrated Processes: Nursing Process: Assessment | Client Need: Safe and Effective Care Environment

A nurse is caring for a client who has been prescribed disulfiram (Antabuse) as a deterrent to alcohol relapse. Which information should the nurse include when teaching the client about this medication? A. "Only oral ingestion of alcohol will cause a reaction when taking this drug." B. "It is safe to drink beverages that have only 12% alcohol content." C. "This medication will decrease your cravings for alcohol." D. "Reactions to combining Antabuse with alcohol can occur 2 weeks after stopping the drug."

ANS: D If Antabuse is discontinued, it is important for the client to understand that the sensitivity to alcohol may last for as long as 2 weeks.

14. Which of these statements by the patient are indications of complicated grieving? A. I feel like I should have been the one to die in that hurricane. B. Last year, several of my coworkers died in a hurricane and I still cant go back to work. C. Ive been having incapacitating migraines ever since the memorial services. D. All of the above

ANS: D Item A indicates survivor guilt, and items B and C are both indications that the trauma has contributed to functional impairment. All three are symptoms of complicated grieving.

A client experiences an exacerbation of psychiatric symptoms to the point of threatening self-harm. Which action step of the Wellness Recovery Action Plan (WRAP) model should be employed, and what action reflects this step? A. Step 3: Triggers that cause distress or discomfort are listed. B. Step 4: Signs indicating relapse are identified and plans for responding are developed. C. Step 5: A specific plan to help with symptoms is formulated. D. Step 6: Following client-designed plan, caregivers now become decision-makers.

ANS: D The WRAP recovery model is a step-wise process through which an individual is able to monitor and manage distressing symptoms that occur in daily life. The six steps include Step 1, Develop a Wellness Toolbox; Step 2, Daily Maintenance List; Step 3, Triggers; Step 4, Early Warning Signs; Step 5, Things Are Breaking Down or Getting Worse; and Step 6, Crisis Planning. In Step 6 (Crisis Planning), clients can no longer care for themselves, make independent decisions, or keep themselves safe. Caregivers take an active role in this step on behalf of the client and implement the plan that the client has previously developed. All other actions presented require the client to be functionally capable. KEY: Cognitive Level: Analysis | Integrated Processes: Nursing Process: Evaluation | Client Need: Safe and Effective Care Environment

A nursing instructor is teaching students about the differences between partial and inpatient hospitalization. In what way does partial hospitalization differ from traditional inpatient hospitalization? A. Partial hospitalization does not provide medication administration and monitoring. B. Partial hospitalization does not use an interdisciplinary team. C. Partial hospitalization does not offer a comprehensive treatment plan. D. Partial hospitalization does not provide supervision 24 hours a day.

ANS: D The instructor should explain that partial hospitalization does not provide supervision 24 hours a day. Partial hospitalization programs generally offer a comprehensive treatment plan formulated by an interdisciplinary team. This has proved to be an effective method of preventing hospitalization. KEY: Cognitive Level: Application | Integrated Processes: Nursing Process: Planning | Client Need: Psychosocial Integrity

Which nursing statement best describes the current nature of mental health care in the community? A. "All homeless people have a history of institutionalization and are frequently admitted to acute care settings." B. "In the United States, the rate of serious mental illness in the prison population is the same as the general population." C. "The deinstitutionalization movement in the United States was successful in transitioning clients into the community." D. "Today, the majority of clients admitted to psychiatric hospitals are in a crisis stage, and the treatment goal is stabilization."

ANS: D The majority of clients admitted to psychiatric hospitals are in a crisis stage, and the treatment goal is stabilization and reintroduction into the community. Crisis situations can occur because of treatment noncompliance and exacerbations of the chronic mental illness. KEY: Cognitive Level: Application | Integrated Processes: Nursing Process: Evaluation | Client Need: Psychosocial Integrity

A nurse holds the hand of a client who is withdrawing from alcohol. What is the nurse's rationale for this intervention? A. To assess for emotional strength B. To assess for Wernicke-Korsakoff syndrome C. To assess for tachycardia D. To assess for fine tremors

ANS: D The nurse is most likely assessing the client for fine tremors secondary to alcohol withdrawal. Withdrawal from alcohol can also cause headache, insomnia, transient hallucinations, depression, irritability, anxiety, elevated blood pressure, sweating, tachycardia, malaise, coarse tremors, and seizure activity.

Which medication orders should a nurse anticipate for a client who has a history of complicated withdrawal from benzodiazepines? A. Haloperidol (Haldol) and fluoxetine (Prozac) B. Carbamazepine (Tegretol) and donepezil (Aricept) C. Disulfiram (Antabuse) and lorazepan (Ativan) D. Chlordiazepoxide (Librium) and phenytoin (Dilantin)

ANS: D The nurse should anticipate that a physician would order chlordiazepoxide (Librium) and phenytoin (Dilantin) for a client who has a history of complicated withdrawal from benzodiazepines. It is common for long-lasting benzodiazepines to be prescribed for substitution therapy. Phenytoin (Dilantin) is an anticonvulsant that would be indicated for a client who has experienced a complicated withdrawal. Complicated withdrawals may progress to seizure activity.

Which client statement indicates a knowledge deficit related to substance abuse? A. "Although it's legal, alcohol is one of the most widely abused drugs in our society." B. "Tolerance to heroin develops quickly." C. "Flashbacks from LSD use may reoccur spontaneously." D. "Marijuana is like smoking cigarettes. Everyone does it. It's essentially harmless."

ANS: D The nurse should determine that the client has a knowledge deficit related to substance abuse when the client compares marijuana to smoking cigarettes and claims it to be harmless. Cannabis is the second most widely abused drug in the United States.

An older client has recently moved to a nursing home. The client has trouble concentrating and socially isolates. A physician believes the client would benefit from medication therapy. Which medication should the nurse expect the physician to prescribe? A. Haloperidol (Haldol) B. Donepezil (Aricept) C. Diazepam (Valium) D. Sertraline (Zoloft)

ANS: D The nurse should expect the physician to prescribe sertraline (Zoloft) to improve the client's social functioning and concentration levels. Sertraline (Zoloft) is an SSRI (selective serotonin reuptake inhibitor) antidepressant. Depression is the most common mental illness in older adults and is often misdiagnosed as neurocognitive disorder. KEY: Cognitive Level: Application | Integrated Processes: Nursing Process: Evaluation | Client Need: Physiological Integrity: Pharmacological and Parenteral Therapies

A client diagnosed with neurocognitive disorder due to Alzheimer's disease can no longer ambulate, does not recognize family members, and communicates with agitated behaviors and incoherent verbalizations. The nurse recognizes these symptoms as indicative of which stage of the illness? A. Confabulation stage B. Early stage C. Middle stage D. Late stage

ANS: D The nurse should recognize that this client is in the late stage of Alzheimer's disease. The late stage is characterized by a severe cognitive decline. KEY: Cognitive Level: Application | Integrated Processes: Nursing Process: Assessment | Client Need: Physiological Integrity

A client diagnosed with neurocognitive disorder due to Alzheimer's disease has impairments of memory and judgment and is incapable of performing activities of daily living. Which nursing intervention should take priority? A. Present evidence of objective reality to improve cognition B. Design a bulletin board to represent the current season C. Label the client's room with name and number D. Assist with bathing and toileting

ANS: D The priority nursing intervention for this client is to assist with bathing and toileting. A client who is incapable of performing activities of daily living requires assistance in these areas to ensure health and safety. KEY: Cognitive Level: Analysis | Integrated Processes: Nursing Process: Implementation | Client Need: Physiological Integrity

11. A client diagnosed with neurocognitive disorder due to Alzheimers disease has impairments of memory and judgment and is incapable of performing activities of daily living. Which nursing intervention should take priority?

Assist with bathing and toileting The priority nursing intervention for this client is to assist with bathing and toileting. A client who is incapable of performing activities of daily living requires assistance in these areas to ensure health and safety

A nurse is interviewing a client in an outpatient substance-abuse clinic. To promote success in the recovery process, which outcome should the nurse expect the client to initially accomplish? A. The client will identify one person to turn to for support. B. The client will give up all old drinking buddies. C. The client will be able to verbalize the effects of alcohol on the body. D. The client will correlate life problems with alcohol use.

ANS: D To promote the recovery process the nurse should expect that the client would initially correlate life problems with alcohol use. Acceptance of the problem is the first step of the recovery process.

Which situation presents an example of the basic concept of a recovery model? A. The client's family is encouraged to make decisions in order to facilitate discharge. B. A social worker, discovering the client's income, changes the client's discharge placement. C. A psychiatrist prescribes an antipsychotic drug on the basis of observed symptoms. D. A client diagnosed with schizophrenia schedules follow-up appointments and group therapy.

ANS: D The basic concept of a recovery model is empowerment of the consumer. The recovery model is designed to allow consumers primary control over decisions about their own care. KEY: Cognitive Level: Analysis | Integrated Processes: Nursing Process: Evaluation | Client Need: Psychosocial Integrity

30. Laboratory results reveal elevated levels of prolactin in a client diagnosed with schizophrenia. When assessing the client, the nurse should expect to observe which symptoms? Select all that apply. A. Apathy B. Social withdrawal C. Anhedonia D. Galactorrhea E. Gynecomastia

ANS: D, E Dopamine blockage, an expected action of antipsychotic medications, also results in prolactin elevation. Galactorrhea and gynecomastia are symptoms of prolactin elevation.

_________________________ from mental health disorders and substance use disorders is a process of change through which individuals improve their health and wellness, live a self-directed life, and strive to reach their full potential.

ANS: Recovery Recovery from mental health disorders and substance use disorders is a process of change through which individuals improve their health and wellness, live a self-directed life, and strive to reach their full potential. Recovery is the restoration to a former or better state or condition. KEY: Cognitive Level: Knowledge | Integrated Processes: Nursing Process: Assessment | Client Need: Health Promotion and Maintenance

Order the six steps of The Wellness Recovery Action Plan (WRAP) Model as described by Copeland et al. A.________ Daily Maintenance List B.________ Things Are Breaking Down or Getting Worse C. ________Crisis Planning D.________ Develop a Wellness Toolbox E._________Early Warning Signs F. ________ Triggers

ANS: The correct order is 2, 5, 6, 1, 4, 3. The WRAP model is a step-wise process, through which an individual is able to monitor and manage distressing symptoms that occur in daily life. The six steps include Step 1, Develop a Wellness Toolbox; Step 2, Daily Maintenance List; Step 3, Triggers; Step 4, Early Warning Signs; Step 5, Things Are Breaking Down or Getting Worse; and Step 6, Crisis Planning. KEY: Cognitive Level: Analysis | Integrated Processes: Nursing Process: Assessment | Client Need: Psychosocial Integrity

Alzheimer's Drugs

Acetylcholinesterase Inhibitors N-Methyl D-Aspartate Agonist

Bipolar Disorder (Manic Depressive Illness)

Affective disorder characterized by euphoria, grandiosity, and inflated sense of self-worth. May or may not include sudden swings of depression. Range from mild to severe. Mild: - Feelings of being high, having a sense of well-being, and minor alterations in habits - Do not seek treatment because they like the pleasurable effect Moderate: - Grandiosity, talkativeness, pressured speech, impulsiveness, excessive spending, and bizarre dress and grooming Severe: - Extreme hyperactivity - Flight of Ideas - Sexually acting out; explicit language - Talkativeness - Over responsiveness to stimuli - Agitation and explosiveness - Delusions of Grandeur - Sleep disturbance

Side Effects of Antipsychotic Medications

Agranulocytosis Thrombocytopenia Extrapyramidal Effects: - Parkinsonism - Akathisia - Dystonia - Tardive Dyskinesia Photosensitivity Neuroleptic Malignant Syndrome (Fever) Serotonin Syndrome Anticholinergic Effects

Alcohol Drugs

Alcohol Deterrents

Dissociative Disorders

Alteration in function of consciousness, personality, memory, or identity. Handle stress by "splitting" from the situation and going into a fantasy state. This is an unconscious defense mechanisms to protect from overwhelming anxiety: Examples: - Psychogenic Amnesia - Psychogenic Amnesia with Fugue - Dissociative Identity Disorder - Depersonalization including derealization Assessment: - Depression, mood swings, insomnia, potential for suicide - Varying levels of orientation and anxiety - Impairment of social and occupational function - Self-medication with alcohol or drugs Interventions: - Reduce environmental stimulation - Stay with client during depersonalization - Accept client during various experiences and personalities - Identify stressful situations that cause transition - Identify coping measures - Note that all behaviors have meaning - With amnesia, do not provide too much info at once. The amnesia is there to protect the patient so too much can cause decompensation.

7. Which symptom should a nurse identify that would differentiate clients diagnosed with neurocognitive disorders from clients with pseudodementia (depression)?

Altered task performance The nurse should identify that attention and concentration are impaired in neurocognitive disorder and not in pseudodementia (depression).

Tricyclic Antidepressants

Amitryptyline (Elavil) Nortriptyline (Aventyl) Protriptyline (Vivactil) Maprotiline (Ludiomil) Imipramine (Tofranil) Desipramne (Norpramin)

Somatization Disorder

An emergency department nurse is assessing a 28-year-old client who complains of back pain, migraine headache, and feelings of generalized fatigue. The client's medical record indicates that she has had multiple emergency department visits with the same complaints since her abusive husband left her. Based on these findings, the client most likely has with which disorder?

The nurse is caring for a patient experiencing postoperative pain. The physician orders 2.5 mg of morphine IV every two hours. Morphine is supplied in 10 mg/mL vials. How many mL will the nurse administer? A) 0.25 mL B) 0.5 mL C) 1 mL D) 2 mL

Ans: A Feedback: 10 mg = 1 mL and a dose of 2.5 mg is ordered. 10 mg/1 mL: 2.5 mg/x Cross-multiply to yield 2.5 mg = 10x. Divide each side by 10 to learn the nurse should administer 0.25 mL

A patient has just been prescribed a phenothiazine. During patient teaching about this drug, what would be important for the nurse to tell the patient? A) The urine can turn pink or reddish. B) The urine output will be decreased. C) Diarrhea can be an adverse effect. D) Hyperexcitability can occur.

Ans: A Feedback: Phenothiazines can cause the urine to turn pink or reddish. The patient should be informed that this is a simple color change and is not caused by blood in his urine. Decreased urine output is not associated with this drug. Constipation is usually an adverse effect of the drug. Drowsiness, not hyperexcitability, can occur.

8. At what time during a 24-hour period should a nurse expect clients with Alzheimers disease to exhibit more pronounced symptoms?

At twilight The nurse should determine that clients with Alzheimers disease exhibit more pronounced symptoms at twilight. Sundowning is the term used to describe the worsening of symptoms in the late afternoon and evening

Identification

Attempt to be like someone or emulate the personality, traits, or behaviors of another person

The nurse is caring for an adolescent patient who began taking an antipsychotic drug last month to treat newly diagnosed schizophrenia. The drug has not been effective and the mother asks the nurse if this means the adolescents symptoms cannot be controlled by drugs. What is the nurses best response? A) Patients commonly have to try different drugs until the most effective drug is identified. B) Some patients do not respond to drugs and have to rely solely on behavior therapy. C) Most likely your child was not taking the medication properly as prescribed. D) He may need to take multiple drugs before effects will be seen that control his symptoms.

Ans: A Feedback: A patient who does not respond to one drug may react successfully to another agent. Responses may also vary because of cultural issues. The selection of a specific drug depends on the desired potency and patient tolerance of the associated adverse effects. It is not common to have a patient who does not demonstrate some improvement from medications so it would be incorrect to tell the mother the child wont respond to any drug after trying only one medication. There is no indication the drug was taken improperly and even properly administered drugs will not work on all patients. Multiple drug therapy is not indicated by the question.

The nurse is caring for a patient treated with flumazenil (Anexate) for benzodiazepine toxicity. After administering flumazenil what will the nurse carefully assess for? A) Agitation, confusion, and seizures B) Cerebral hemorrhage and dystonia C) Hypertension and renal insufficiency D) Hypotension, dysrhythmias, and cardiac arrest

Ans: A Feedback: Administration of flumazenil blocks the action of benzodiazepines. If the patient has been taking these medications for an extended period of time, the blockage of the drugs effects could precipitate an acute benzodiazepine withdrawal syndrome with symptoms including agitation, confusion, and seizures. Anexate does not cause cerebral hemorrhage and dystonia, hypertension, renal insufficiency, hypotension, dysrhythmias, and cardiac arrest.

The nurse administers promethazine (Phenergan) to the patient before sending the patient to the preoperative holding area. What is the rationale for administration of this drug? A) Sedation B) Oral secretions C) Hypotension and bradycardia D) Confusion

Ans: A Feedback: Antihistamines (promethazine, diphenhydramine [Benadryl]) can be very sedating in some people. They are used as preoperative medications and postoperatively to decrease the need for narcotics. Promethazine is not given for hypotension, bradycardia, confusion, or oral secretions.

Psychosis is a severe mental illness characterized by what? A) Disordered thought B) Increased social interaction C) Hypoactivity with aggressiveness D) Paranoid hallucinations

Ans: A Feedback: Antipsychotic drugs are used mainly for the treatment of psychosis, a severe mental disorder characterized by disordered thought processes; blunted or inappropriate emotional responses; bizarre behavior ranging from hypoactivity to hyperactivity with agitation, aggressiveness, hostility, and combativeness; social withdrawal in which a person pays less-than-normal attention to the environment and other people; deterioration from previous levels of occupational and social functioning (poor self-care and interpersonal skills); hallucinations; and paranoid delusions.

Parents bring a 15-year-old boy into the clinic. The parents tell the nurse that there is a family history of schizophrenia and they fear their son has developed the disease. What symptoms, if described by the family, would support their conclusion? A) He hears and interacts with voices no one else can hear. B) He is overactive and always so excitable. C) He falls asleep in the middle of a sentence. D) He cannot concentrate and his grades are suffering.

Ans: A Feedback: Characteristics of schizophrenia include hallucinations, paranoia, delusions, speech abnormalities, and affective problems. Overactivity and excitement are associated with mania. Falling asleep suddenly describes narcolepsy. Difficulty concentrating and failing grades is associated with attention deficient disorders.

A 75-year-old patient is brought to the emergency department by his family. The family relates that the patient is complaining of confusion, seizures, and abnormal perception of movement. The nurse reviews all of the medication bottles found in the house and suspects the patient overdosed on what medication? A) Benzodiazepine B) Antihypertensive C) Sedative D) Analgesic

Ans: A Feedback: Common manifestations of benzodiazepine toxicity include increased anxiety, psychomotor agitation, insomnia, irritability, headache, tremor, and palpitations. Less common but more serious manifestations include confusion, abnormal perception of movement, depersonalization, psychosis, and seizures. These symptoms are not found in association with options B, C, or D.

When evaluating the effects of narcotic agonist-antagonists on a patient, what adverse effects would the nurse monitor for? A) Hypertension B) Bleeding C) Suppressed bone marrow function D) Increased pulse pressure

Ans: A Feedback: Monitor for adverse effects (e.g., central nervous system changes, gastrointestinal (GI) depression, respiratory depression, arrhythmias, hypertension). Bleeding, bone marrow suppression, and increased pulse pressure are not normally seen with these drugs.

As the nurse settles the patient into his room after returning from the post-anesthesia care unit (PACU), the patient says he is in severe pain. The nurse checks the medical record and sees the patient has an order for morphine 4 to 8 mg every 1 to 2 hour IV as needed for pain. The nurse sees this medication has not been administered yet so the nurse administers 4 mg. After administering the drug, the PACU nurse calls to say a dose of morphine was given and not documented. What drug will the nurse be prepared to administer if the patients respiratory rate is depressed? A) Naloxone hydrochloride tartrate (Narcan) B) Butorphanol C) Buprenorphine (Buprenex) D) Nalbuphine hydrochloride (Nubain)

Ans: A Feedback: Naloxone is the drug of choice for treatment of opioid overdose. Butorphanol (INN) is amorphinan-type synthetic opioid analgesic that would not reverse the effects of an opioid. Buprenex (buprenorphine hydrochloride) is a narcotic-agonist-antagonist and would suppress respirations further. Nalbuphine is a synthetic opioid used commercially as an analgesic that would also depress respirations.

The physician has ordered olanzapine (Zyprexa) for a new patient. What laboratory test should be done before administration of olanzapine? A) Blood glucose B) Urine specific gravity C) Cholesterol D) Hemoglobin and hematocrit

Ans: A Feedback: Olanzapine has been associated with weight gain, hyperglycemia, and initiation or aggravation of diabetes mellitus. Other options are not necessary for patients taking olanzapine unless a secondary diagnosis indicates a need.

Projection

Attributing one's own thoughts or impulses to another person

NDRIs

Bupropion (Wellbutrin) Mirtazapine (Remeron)

The nurse admits a patient newly diagnosed with schizophrenia to the inpatient mental health unit. What is the priority reason for why the nurse includes the family when collecting the nursing history? A) The patient may not be able to provide a coherent history. B) The patient may not be able to speak due to reduced level of consciousness. C) The family will feel better if they are included in the process. D) The patient will be less anxious if the family listens while he answers questions.

Ans: A Feedback: Schizophrenia, the most common psychosis, is characterized by delusions, hallucinations, and inappropriate responses to stimuli. As a result, the patient may be unable to provide a coherent history and may be unaware of his behaviors considered dysfunctional. There is no reason to suspect the patient cannot speak and reducing anxiety is not the priority rationale for including family. While family is included in treatment, the goal is to treat the patient and not make the family feel better if actions were not in the patients best interests.

A patient on chlorpromazine is feeling better and decides they no longer need their medication. The nurse teaches the patient that abrupt withdrawal of a typical antipsychotic medication can result in what? A) Insomnia B) Tardive dyskinesia C) Somnolence D) Constipation

Ans: A Feedback: Sudden withdrawal can cause cholinergic effects such as diarrhea, gastritis, nausea, vomiting, dizziness, arrhythmias, drooling, and insomnia. Abrupt withdrawal of a typical antipsychotic generally does not cause tardive dyskinesia, somnolence, or constipation.

The nurse is caring for a child receiving a central nervous system (CNS) stimulant who was admitted to the pediatric intensive care unit following repeated seizures after a closed head injury. The physician orders phenytoin to control seizures and lorazepam to be administered every time the child has a seizure. What is the nurses priority action? A) Call the doctor and question the administration of phenytoin. B) Call the doctor and question the administration of lorazepam. C) Wait 24 hours before beginning to administer phenytoin. D) Wait 24 hours before beginning to administer lorazepam.

Ans: A Feedback: The combination of CNS stimulants with phenytoin leads to a risk of increased drug levels. Patients who receive such a combination should be monitored for toxicity. There is no contraindication for use of lorazepam.

A child was diagnosed with attention-deficit hyperactivity disorder and methylphenidate was prescribed for treatment to be taken once a day in a sustained release form. On future visits what is a priority nursing assessment for this child? A) Weight and height B) Breath sounds and respiratory rate C) Urine output and kidney function D) Electrocardiogram (ECG) and echocardiogram Ans:

Ans: A Feedback: The nurse needs to carefully track this childs weight and height because the drug can cause weight loss, anorexia, and nausea that could result in slowed or absent growth. There would be no need to monitor breath sounds, respiratory rate, urine output, and kidney function. Although arrhythmias may occur as an adverse effect necessitating an ECG, there is no need to perform echocardiograms.

The mother of a child diagnosed with attention-deficit syndrome receives a prescription for a central nervous system (CNS) stimulant to treat her child. The mother asks the nurse, I dont understand why were giving a stimulant to calm him down? What is the nurses best response to this mother? A) It helps the reticular activating system (RAS), a part of the brain, to be more selective in response to incoming stimuli. B) It helps energize the child so they use up all of their available energy and then they can focus on quieter stimuli. C) No one truly understands why it works but it has been demonstrated to be very effective in treating ADHD. D) The drugs work really well and you will see a tremendous change in your child within a few weeks without any other treatment.

Ans: A Feedback: The paradoxical effect of calming hyperexcitability through CNS stimulation seen in attention-deficit syndrome is believed to be related to increased stimulation of an immature RAS, which leads to the ability to be more selective in response to incoming stimuli. CNS stimulants do not cause the child to use all his energy, the effect is thought to be understood, and telling the mother the drug just works without any explanation is not appropriate and may result in noncompliance with pharmacology therapy if the mother does not understand why the drug is given.

The nurse is caring for an older adult in the long-term care facility who has begun to display signs of anxiety and insomnia. What is the priority nursing action? A) Assess the patient for physical problems. B) Call the provider and request an antianxiety drug order. C) Increase the patients social time, encouraging interaction with others. D) Suggest the family visit more often to reduce the residents stress level.

Ans: A Feedback: The patient should be screened for physical problems, neurological deterioration, or depression, which could contribute to the insomnia or anxiety. Only after physical problems are ruled out would the nurse consider nondrug measures such as increased socialization with other residents or family members. If nothing else is effective, pharmacological intervention may be necessary.

The nurse administers chlorpromazine intramuscularly to the preoperative patient who is extremely anxious about surgery in the morning. What priority teaching point will the nurse provide this patient? A) Remain recumbent for at least 30 minutes after the injection. B) Do not eat for 1 hour after the drug is administered. C) Encourage fluids with the goal of 3,000 mL/d. D) Avoid eating avocados and oranges when taking this medication.

Ans: A Feedback: When giving a parenteral form of an antipsychotic, the patient should remain recumbent to decrease the risk of injury if orthostatic hypotension occurs. Eating after drug injection should not interfere with the drugs absorption and although adequate hydration should be maintained there is no need to increase fluid intake. Avocados and oranges are not contraindicated in patients receiving this medication.

The nurse assesses the patient who had an abrupt withdrawal of benzodiazepines for withdrawal syndrome and would recognize what symptoms as part of the syndrome? (Select all that apply.) A) Headache B) Nightmares C) Malaise D) Bradycardia E) Hypotension

Ans: A, B, C Feedback: Abrupt cessation of benzodiazepines may lead to a withdrawal syndrome characterized by nausea, headache, vertigo, malaise, and nightmares. Withdrawal symptoms may be caused by the abrupt separation of benzodiazepine molecules from their receptor sites and the resulting acute decrease in gamma-aminobutyric acid (GABA) neurotransmission. Because GABA is an inhibitory neurotransmitter, less GABA may produce a less inhibited central nervous system (CNS) and therefore symptoms of hyperarousal or CNS stimulation. The nurse would not categorize hypotension or bradycardia as indicating benzodiazepine withdrawal.

The nurse is caring for a patient who received a new diagnosis of cancer. The patient exhibits signs of a sympathetic stress reaction. What signs and symptoms will the nurse assess in this patient consistent with an acute reaction to stress? (Select all that apply.) A) Profuse sweating B) Fast heart rate C) Rapid breathing D) Hypotension E) Inability to interact with others

Ans: A, B, C Feedback: Anxiety is often accompanied by signs and symptoms of the sympathetic stress reaction that may include sweating, fast heart rate, rapid breathing, and elevated blood pressure. Chronically anxious people may be afraid to interact with other people but this is not usually seen in an acute stress reaction.

What is the nurses priority assessment when administering narcotics to older adults? (Select all that apply.) A) Central nervous system (CNS) effects B) Gastrointestinal effects C) Cardiovascular effects D) Urinary effects E) Developmental effects

Ans: A, B, C Feedback: Older patients are more likely to experience the adverse effects associated with these drugs, including central nervous system, gastrointestinal (GI), and cardiovascular effects. Urinary and developmental effects are not areas of high concern.

The nurse is teaching a class for nurses working in prenatal clinics about the danger associated with use of benzodiazepines during pregnancy and explains that what fetal anomalies result from maternal use of benzodiazepines during the first trimester of pregnancy? (Select all that apply.) A) Cleft lip or palate B) Inguinal hernia C) Cardiac defects D) Microencephaly E) Gastroschises

Ans: A, B, C, D Feedback: Benzodiazepines are contraindicated in pregnancy because a predictable syndrome of cleft lip or palate, inguinal hernia, cardiac defects, microcephaly, or pyloric stenosis occurs when they are taken in the first trimester. Gastroschises, when the abdominal organs are found outside the abdominal cavity, is not associated with use of benzodiazepine use in the first trimester.

The nurse is caring for a patient newly diagnosed with schizophrenia. His parents say they have heard the term before but do not really understand exactly what schizophrenia means. How would the nurse describe the disorder? (Select all that apply.) A) Thought disorder B) Difficulty functioning in society C) Hallucinations can be auditory, visual, or sensory D) Can be cured with the correct medications E) Enter into fugue state in most cases

Ans: A, B, C, D Feedback: Mental disorders are thought process disorders that may be caused by some inherent dysfunction within the brain. A psychosis is a thought disorder, and schizophrenia is the most common psychosis in which delusions and hallucinations are hallmarks. Hallucinations can be auditory, visual, or sensory. Patients diagnosed with schizophrenia have difficulty functioning in society. Schizophrenic patients do not generally go into fugue states and it certainly is not a common disorder.

What reasons can the nurse give for why barbiturates are no longer considered the mainstay for treatment of anxiety? (Select all that apply.) A) Adverse effects are more severe. B) There is an increased risk of physical tolerance. C) There is an increased risk of psychological dependence. D) The most common adverse effects are related to cardiac arrhythmias. E) Hypersensitivity reactions can sometimes be fatal.

Ans: A, B, C, E Feedback: The adverse effects caused by barbiturates are more severe than those associated with other, newer sedatives/hypnotics. For this reason, barbiturates are no longer considered the mainstay for the treatment of anxiety. In addition, the development of physical tolerance and psychological dependence is more likely with the barbiturates than with other anxiolytics. The most common adverse effects are related to central nervous system (CNS) depression. Hypersensitivity reactions to barbiturates are sometimes fatal.

Narcotic agonists-antagonists have what function? (Select all that apply.) A) Relief of moderate-to-severe pain B) Adjunctive therapies to nonsteroidal anti-inflammatory drugs (NSAIDs) C) Relief of pain during labor and delivery D) Relief of orthopedic pain E) Adjuncts to general anesthesia

Ans: A, C, E Feedback: These drugs have three functions: (1) relief of moderate-to-severe pain, (2) adjuncts to general anesthesia, and (3) relief of pain during labor and delivery. Adjunctive therapies to NSAIDs or specificity for orthopedic pain are not functions of this classification of medication.

Narcotic agonists-antagonists have what function? (Select all that apply.) A) Relief of moderate-to-severe pain B) Adjunctive therapies to nonsteroidal anti-inflammatory drugs (NSAIDs) C) Relief of pain during labor and delivery D) Relief of orthopedic pain E) Adjuncts to general anesthesia

Ans: A, C, E Relief of moderate-to-severe pain, Relief of pain during labor and delivery &Relief of orthopedic pain Feedback: These drugs have three functions: (1) relief of moderate-to-severe pain, (2) adjuncts to general anesthesia, and (3) relief of pain during labor and delivery. Adjunctive therapies to NSAIDs or specificity for orthopedic pain are not functions of this classification of medication.

The anatomy and physiology instructor is talking about pain sensations. What produces pain sensations when stimulated by generating nerve impulses? (Select all that apply) A) A-delta fibers B) D-delta sensory nerves C) Mu receptors D) Sigma-receptors E) C fibers

Ans: A, E Feedback: Two small-diameter sensory nerves, A-delta and C fibers, respond to stimulation by generating nerve impulses that produce pain sensations. Large-diameter sensory nerves (i.e., A fibers) transmit sensations associated with touch and temperature. Mu-receptors are primarily pain-blocking receptors; sigma-receptors cause papillary dilation and may be responsible for the hallucinations, dysphoria, and psychoses that can occur with narcotic use.

When evaluating the effects of narcotic agonist-antagonists on a patient, what adverse effects would the nurse monitor for? A) Hypertension B) Bleeding C) Suppressed bone marrow function D) Increased pulse pressure

Ans: A: Hypertension Feedback: Monitor for adverse effects (e.g., central nervous system changes, gastrointestinal (GI) depression, respiratory depression, arrhythmias, hypertension). Bleeding, bone marrow suppression and increased pulse pressure are not normally seen with these drugs.

What order for naloxone would be appropriate for the nurse to administer for reversal of opioid effects? A) 1 mg IV repeat every 2 to 3 minutes B) 5 mg IV repeat every 5 minutes C) 0.1 mg IV repeat every 2 to 3 minutes D) 0.4 mg IV repeat every 3 minutes

Ans: C Feedback: 0.1 to 0.2 mg is given IV and then repeated every 2 to 3 minutes for reversal of opioid effects. If the patient has overdosed on opioids the dose would be 0.4 to 2 mg every 2 to 3 minutes. The other options are incorrect.

Nonbenzodiazepines

Buspirone (BuSpar) Zolpidem (Ambien) Ramelteon (Rozerem)

The pharmacology instructor is explaining to their class the difference between the typical and the atypical groups of antipsychotic drugs. What medication would the instructor explain to the students has fewer extrapyramidal effects and greater effectiveness than older antipsychotic drugs in relieving negative symptoms of schizophrenia? A) Chlorpromazine (Thorazine) B) Clozapine (Clozaril) C) Thiothixene (Navane) D) Haloperidol (Haldol)

Ans: B Feedback: Advantages of clozapine include improvement of negative symptoms without causing the extrapyramidal effects associated with older antipsychotic drugs. Chlorpromazine is a typical antipsychotic, one of the older drugs, which does cause the extrapyramidal effects. Navane is part of the thioxanthene group of typical antipsychotics. This group of drugs has low sedative and hypotensive effects but can cause extrapyramidal effects. Haloperidol is a butyrophenone group drug used in psychiatric disorders. Usually, it produces a relatively low incidence of hypotension and sedation and a high incidence of extrapyramidal effects.

The nurse, providing teaching about a typical antipsychotic newly prescribed for the patient, cautions against use of alcohol with the drug by explaining it will have what effect? A) Prolonged QT interval B) Increased central nervous system (CNS) depression C) Increased anticholinergic effects D) Increased gastrointestinal (GI) adverse effects

Ans: B Feedback: Antipsychoticalcohol combinations combinations result in an increased risk of CNS depression, and antipsychoticanticholinergic combinations lead to increased anticholinergic effects, so dosage adjustments are necessary. Patients should not take thioridazine or ziprasidone with any other drug associated with prolongation of the QT interval. Increase in GI adverse effects is not associated with concurrent use of alcohol.

The nurse is caring for a patient who has a sedative hypnotic ordered. The nurse would consider this drug contraindicated if the patient had what disorder? A) Neurological diseases B) Liver failure C) Endocrine disorders D) Heart disease

Ans: B Feedback: Benzodiazepines undergo extensive hepatic metabolism. In the presence of liver disease, the metabolism of most benzodiazepines is slowed, with resultant accumulation and increased risk of adverse effects. Neurological disorders, endocrine disorders, and heart disease are not contraindications for the use of benzodiazepines.

What anxiolytic drugs would be given to a premenopausal patient who is a registered nurse planning to return to work at the hospital after anxiety is controlled? A) Alprazolam (Xanax) B) Buspirone (BuSpar) C) Diazepam (Valium) D) Clorazepate (Tranxene)

Ans: B Feedback: Buspirone is a newer anxiolytic drug that does not cause sedation or muscle relaxation. It is preferred when the patient needs to be alert such as when driving or working. Alprazolam, diazepam, and clorazepate are benzodiazepines, which cause drowsiness, sedation, depression, lethargy, confusion, and decreased mental alertness. It would be unsafe for a nurse to function in her role while taking one of these drugs.

When compared with benzodiazepines, buspirone (BuSpar) stands out as unique among antianxiety drugs because of what factor? A) Increases the central nervous system (CNS) depression of alcohol and other drugs. B) Lacks muscle relaxant and anticonvulsant effects. C) Causes significant physical and psychological dependence. D) Rapidly absorbed from the gastrointestinal (GI) tract and metabolized in the liver.

Ans: B Feedback: Buspirone, a newer antianxiety agent, has no sedative, anticonvulsant, or muscle-relaxant properties, and its mechanism of action is unknown. However, it reduces the signs and symptoms of anxiety without many of the central nervous system effects and severe adverse effects associated with other anxiolytic drugs. Most of the antianxiety drugs are rapidly absorbed from the GI tract, metabolized in the liver, have a significant drugdrug interaction with alcohol and other drugs, and can result in psychological dependence.

An elderly patient has been taking zolpidem (Ambien) as a sleep aid for the past 2 months. On admission to the assisted-living facility, it is determined that the drug is no longer needed. What is an important nursing consideration concerning this drug? A) Hallucinations are common. B) The drug needs to be withdrawn gradually. C) Another anxiolytic will need to be substituted. D) Sundowning is common with withdrawal from this drug.

Ans: B Feedback: It is important for the nurse to understand that zolpidem must be withdrawn gradually over a 2-week period after prolonged use. If chloral hydrate is stopped suddenly, it will result in serious adverse effects. Hallucinations and sundowning are not common with withdrawal of the drug. The prescriber and the patient would determine the need for chloral hydrate to be substituted for another anxiolytic.

A patient, in the manic phase of bipolar disorder, is being discharged home on an antimanic drug. What antimanic drug is used for long-term maintenance of bipolar disorders? A) Aripiprazole (Abilify) B) Lamotrigine (Lamictal) C) Quetiapine (Seroquel) D) Ziprasidone (Geodon)

Ans: B Feedback: Lamotrigine is used for long-term maintenance of bipolar disorders. Aripiprazole and ziprasidone are used for acute manic and mixed episodes of bipolar disorders. Quetiapine is used as adjunct or monotherapy for the treatment of manic episodes associated with bipolar disorder.

Which drug does not have a recommended pediatric dose? A) Pimozide (Orap) B) Lithium salts (Lithotabs) C) Haloperidol (Haldol) D) Risperidone (Risperdal)

Ans: B Feedback: Lithium does not have a recommended pediatric dose; the drug should not be administered to children younger than 12 years old. Pimozide, haloperidol, and risperidone all have recommended pediatric doses.

The nurse is teaching the mother of a child diagnosed with attention-deficit hyperactivity disorder how to administer methylphenidate (Ritalin). When would the nurse instruct the mother to administer this drug? A) Administer at lunch every day. B) Administer at breakfast every day. C) Administer at dinner every day. D) Administer at bedtime.

Ans: B Feedback: Several long-acting formulations of methylphenidate have become available that allow the drug to be given only once a day. It should always be given in the morning because administration at dinnertime or bedtime could result in insomnia.

A nursing student is developing a plan of care for a suicidal client. Which documented intervention should the student implement first?

Assess suicide risk. Assessment is the first step of the nursing process to gain needed information to determine further appropriate interventions.

The nurse is caring for a resident in a long-term care facility who is African American with a history of an anxiety disorder. The patient is receiving oral lorazepam (Ativan) 2 mg t.i.d. When developing this patients plan of care, what priority assessment will the nurse include? A) Depression B) Extreme sedation C) Phlebitis D) Nightmares

Ans: B Feedback: Special care should be taken when anxiolytic or hypnotic drugs are given to African Americans. About 15% to 20% of African Americans are genetically predisposed to delayed metabolism of benzodiazepines. As a result, they may develop high serum levels of these drugs, with increased sedation and an increased incidence of adverse effects. Depression is not a common adverse effect. Phlebitis can occur at injection sites but this patient is taking the medication orally. Nightmares occur during drug withdrawal.

Why would the nurse expect the patient with liver disease to receive a smaller dose of benzodiazepines? A) Excretion of the drug relies on liver function. B) The drugs are metabolized extensively in the liver. C) They are lipid soluble and well distributed throughout the body. D) The drugs are well absorbed from the gastrointestinal tract.

Ans: B Feedback: The benzodiazepines are metabolized extensively in the liver. Patients with liver disease must receive a smaller dose and be monitored closely. Excretion is primarily through the urine. All of the answer options are true, but only the fact that the benzodiazepines are metabolized in the liver explains why a patient with liver disease would require smaller dosages.

The nurse is caring for a patient who is receiving an opioid analgesic. What are the nurses priority assessments? A) Pain intensity and blood glucose level B) Level of consciousness and respiratory rate C) Respiratory rate and electrolytes D) Urine output and pain intensity

Ans: B Feedback: The nurse should assess respiratory rate and level of consciousness because respiratory depression and sedation are adverse effects of opioid analgesics. Blood glucose levels, electrolytes, and urine output are not priority assessments with opioid ingestion.

The nurse is caring for a patient in intensive care unit receiving IV lorazepam (Ativan) to reduce anxiety related to mechanical ventilation. While injecting the medication the nurse notes a decrease in blood pressure and bradycardia. What is the nurses priority action? A) Discontinue drug administration. B) Give the IV drug more slowly. C) Notify the patients health care provider. D) Document the reaction to the drug.

Ans: B Feedback: The nurses priority action is to slow the rate of injection because rapid injection of benzodiazepines can result in hypotension and bradycardia and can lead to cardiac arrest.

A nurse is discussing the use of alprazolam (Xanax) with a 68-year-old patient. What statement indicates that the patient has an understanding of the drug? A) When I stop having panic attacks, I can stop taking the drug. B) This drug will calm me down in about 30 minutes after I take it. C) One dose will keep me calm for about 24 hours. D) I am taking an increased dose because of my age.

Ans: B Feedback: The onset of alprazolam is about 30 minutes. The drug must be tapered after long-term use and the duration is approximately 4 to 6 hours. Elderly patients usually have a reduced dosage.

The nurse evaluates teaching as effective when a patient taking a benzodiazepine states, A) I should always take the medication with meals. B) I should not stop taking this drug without talking to my health care provider first. C) I cannot take aspirin with this medication. D) I will have to take this medication for the rest of my life.

Ans: B Feedback: The patient makes a correct statement when saying the drug should not be stopped without talking to the health care provider first because withdrawal of benzodiazepines require careful monitoring and should be gradually withdrawn. Medications do not have to be taken with food, aspirin is not contraindicated, and the medication need only be taken while the condition being treated continues. Patients with anxiety may only need the medication for a few weeks whereas those with a seizure disorder may take it for longer periods of time.

A patient diagnosed with bipolar disorder is to be discharged home in 48 hours. The nurse has completed patient teaching regarding the use of lithium. What statement by the patient indicates an understanding of their responsibility? A) I will increase my salt intake. B) I will increase my fluid intake. C) I will decrease my salt intake. D) I will decrease my fluid intake.

Ans: B Feedback: To maintain a therapeutic lithium level, the patient must increase fluids. A decrease in consumption of fluids can lead to toxicity. An increase in salt intake can lead to lithium excretion and a decrease in effectiveness. A decrease in salt intake can cause retention, also leading to toxicity. Adequate salt intake is necessary to keep serum levels in therapeutic range but need not be increased or decreased.

The nurse is caring for a patient who is taking a benzodiazepine. The nurse knows that caution should be used when administering a benzodiazepine to the elderly because of what possible adverse effect? A) Acute renal failure B) Unpredictable reactions C) Paranoia D) Hallucinations

Ans: B Feedback: Use benzodiazepines with caution in elderly or debilitated patients because of the possibility of unpredictable reactions and in patients with renal or hepatic dysfunction, which may alter the metabolism and excretion of these drugs, resulting in direct toxicity. Dosage adjustments usually are needed for such patients. Acute renal failure, paranoia, and hallucinations are not commonly related to therapy with these medications in the elderly.

A patient, who is 77 years old, is admitted with a diagnosis of dementia. Haloperidol (Haldol) has been ordered for this patient. What nursing considerations would govern the nurses actions? (Select all that apply.) A) It is classed as an atypical antipsychotic. B) A lowered dosage is indicated for older adult. C) It often has a hyperactive effect on patients. D) It should not be used to control behavior with dementia. E) It should only be given every other day.

Ans: B, D Feedback: Haloperidol is classified as a typical antipsychotic with a high risk of extrapyramidal effects and lower risk for hypotension and sedation. Older patients may be more susceptible to the adverse effects of antipsychotic drugs. All dosages need to be reduced and patients monitored very closely for toxic effects and to provide safety measures if central nervous system effects do occur. They should not be used to control behavior with dementia. Haloperidol does not have a hyperactive effect on patients; it should not be given on an every-other-day schedule.

What would the nurse assess for when benzodiazepines are abruptly stopped? A) Urinary retention and change in sexual functioning B) Dry mouth, constipation, nausea, and vomiting C) Nausea, headache, vertigo, malaise, and nightmares D) In most cases nothing significant

Ans: C Feedback: Abrupt cessation of these drugs may lead to a withdrawal syndrome characterized by nausea, headache, vertigo, malaise, and nightmares. When benzodiazepines are stopped abruptly the likelihood of withdrawal symptoms increases with the length of time the patient took the medication. Urinary retention, change in sexual functioning, dry mouth, constipation, nausea, and vomiting are all common adverse effects of the medications classified as benzodiazepines.

According to the Gate Control Theory, what interventions by the nurse could help to block pain impulses? A) Administration of opioid medications B) Administration of narcotic agonist-antagonists C) Back massage D) Acupuncture

Ans: C Feedback: According to the gate control theory, the transmission of these impulses can be modulated or adjusted all along these tracts. All along the spinal cord, interneurons can act as gates by blocking the ascending transmission of pain impulses. It is thought that the gates can be closed by stimulation of the larger A fibers and by descending impulses coming down the spinal cord from higher levels in such areas as the cerebral cortex, the limbic system, and the reticular activating system. Administration of medications does not use the Gate Control Theory. Acupuncture uses the Gate Control Theory but is not performed by the nurse.

A pharmacology student asks the instructor what an accurate description of a drug agonist is. What is the instructors best response? A) A drug that reacts with a receptor site on a cell preventing a reaction with another chemical on a different receptor site B) A drug that interferes with the enzyme systems that act as catalyst for different chemical reactions C) A drug that interacts directly with receptor sites to cause the same activity that a natural chemical would cause at that site D) A drug that reacts with receptor sites to block normal stimulation, producing no effect

Ans: C Feedback: Agonists are drugs that produce effects similar to those produced by naturally occurring neurotransmitters, hormones, or other substances found in the body. Noncompetitive antagonists are drugs that react with some receptor sites preventing the reaction of another chemical with a different receptor site. Drugenzyme interactions interfere with the enzyme systems that stimulate various chemical reactions.

A patient presents at the free clinic complaining of nervousness, worrying about everything, and feeling very tense. What diagnose would the nurse suspect? A) Neurosis B) Psychosis C) Anxiety D) Depression

Ans: C Feedback: Anxiety is a common disorder that may be referred to as nervousness, tension, worry, or using other terms that denote an unpleasant feeling. The other options would not be described by these symptoms.

A patient arrives at the emergency room after attempting suicide by taking an entire bottle of diazepam. What antidote will the nurse most likely administer? A) Phenobarbital (Luminal) B) Dexmedetomidine (Precedex) C) Flumazenil (Romazicon) D) Ramelteon (Rozerem)

Ans: C Feedback: Flumazenil is an antidote to benzodiazepine overdose and is administered to reverse the effects of benzodiazepines when used for anesthesia. Phenobarbital, a barbiturate, would further depress the body functions of this patient. Dexmedetomidine is a new hypnotic drug used in the intensive care unit for mechanically ventilated patients. Ramelteon is also new; it is used as a hypnotic. Adverse effects of this drug include depression and suicidal ideation.

Haloperidol is a typical antipsychotic drug. What adverse effect is associated with this drug? A) Bradycardia B) Bradypnea C) Extrapyramidal effects D) Hypoglycemia

Ans: C Feedback: Haloperidol produces a relatively low incidence of hypotension and sedation and a high incidence of extrapyramidal effects. Haloperidol does not generally produce bradycardia, bradypnea, or hypoglycemia.

What medication would the nurse administer to the patient in severe pain? A) Codeine B) Hydrocodone C) Hydromorphone D) Opium

Ans: C Feedback: Hydromorphone is indicated for moderate-to-severe pain. Codeine is indicated for mild-to-moderate pain, hydrocodone is indicated for moderate pain, and opium is indicated for treatment of diarrhea and relief of moderate pain.

Hypnotic drugs are used to aid people in falling asleep. What physiological system does a hypnotic act on to be effective in helping a patient to sleep? A) Limbic system B) Sympathetic nervous system C) Reticular activating system D) Lymph system

Ans: C Feedback: Hypnotics are used to help people fall asleep by causing sedation. Drugs that are effective hypnotics act on the reticular activating system and block the brains response to incoming stimuli. Hypnosis, therefore, is the extreme state of sedation, in which the person no longer senses or reacts to incoming stimuli. The other options are incorrect.

What antiepileptic medication might the nurse administer to treat bipolar disorder? A) Apriprazole (Abilify) B) Cyclobenzaprine (Flexeril) C) Lamotrigine (Lamictal) D) Temazepam (Restoril)

Ans: C Feedback: Lamotrigine is an antiepileptic agent used for long-term maintenance of patients with bipolar disorders because it decreases occurrence of acute mood episodes. Apriprazole is an atypical antipsychotic and is not an antiepileptic medication. Flexeril is a muscle relaxant and Temazepam is a hypnotic agent. None of these medications are indicated for the treatment of bipolar disorder.

The nurse is presenting an in-service at a childrens unit on hyperactivity. The nurse is told that a 6-year-old on the unit is being treated with methylphenidate (Ritalin). The presenting nurse talks about discharge teaching for this patient and the importance of monitoring what? A) Long bone growth B) Visual acuity C) Weight and complete blood count D) Urea and nitrogen levels

Ans: C Feedback: Methylphenidate is associated with weight loss, bone marrow suppression, and cardiac arrhythmias. Weight, blood count, and cardiac function should be monitored regularly. The drug is not associated with renal dysfunction, visual changes, or growth retardation, so those values would not need to be regularly evaluated as part of drug therapy.

A patients medication has been changed to clozapine (Clozaril). The nurse evaluates this patient for which life-threatening adverse effect? A) Renal insufficiency B) Emphysema C) Neuroleptic malignant syndrome D) Cerebrovascular accident (CVA)

Ans: C Feedback: Neuroleptic malignant syndrome can be a life-threatening adverse effect of atypical non-phenothiazines. Renal insufficiency, emphysema, and CVA are not commonly seen adverse effects of atypical non-phenothiazines.

A 7-year-old boy is admitted to the pediatric behavioral health unit with a diagnosis of an acute psychotic episode. Aripiprazole has been ordered. Before administering the medication, what is the nurses first priority? A) Weigh the patient. B) Obtain baseline vital signs. C) Call the physician. D) Administer the medication between meals.

Ans: C Feedback: Of the antipsychotics, chlorpromazine, haloperidol, pimozide, prochlorperazine, risperidone, thioridazine, and trifluoperazine are the only ones with established pediatric regimens. Aripiprazole has dosages for children 13 to 17 years of age but would not be appropriate for a 7-year-old child. Weighing the patient and obtaining baseline vital signs is necessary assessment data but is not the first priority. There is nothing to indicate medications should be administered between meals.

What nursing intervention is appropriate for a 70-year-old female patient receiving lithium? A) Instruct the patient to use barrier contraceptives. B) Monitor blood glucose levels. C) Monitor fluid and sodium intake. D) Encourage the patient to check daily for weight loss.

Ans: C Feedback: Older patients, and especially those with renal impairment, should be encouraged to maintain adequate hydration and salt intake. Decreased dosages may also be necessary with the elderly. A 70-year-old patient would not be concerned about the use of contraceptives. These drugs alone do not affect glucose levels. Weight loss is usually not associated with lithium use.

The nurse is providing patient teaching about a prescribed opioid analgesic. What is an important teaching point related to a possible adverse effect of this drug? A) Ataxia B) Blurred vision C) Hypotension D) Dysrhythmias

Ans: C Feedback: Orthostatic hypotension is commonly seen in association with some narcotics. Ataxia, blurred vision, and dysrhythmias are not commonly seen adverse effects of an opioid analgesic.

The nurse is caring for a 36-year-old man who experienced a seizure 30 minutes before coming into the emergency room, where he begins to have another. What barbiturate has the fastest onset and would be most appropriate to give to the patient to quickly stop the seizure? A) Amobarbital (Amytal Sodium) B) Mephobarbital (Mebaral) C) Phenobarbital (Luminal) D) Secobarbital (Seconal)

Ans: C Feedback: Phenobarbitals onset is between 10 and 60 minutes, depending on the route administered, and most likely this would be given to the patient. Amobarbital is given for convulsions and the onset is between 15 and 60 minutes. Mephobarbitals onset is between 30 and 60 minutes. Secobarbital is given for convulsive seizures of tetanus and has an onset of 1 to 4 hours.

A nurse is caring for a patient who is taking lithium for mania. The nurses assessment includes a notation of a lithium serum level of 2.4 mEq/L. The nurse anticipates seeing what? A) Fine tremors of both hands B) Slurred speech C) Clonic movements D) Nausea and vomiting

Ans: C Feedback: Serum levels of 2 to 2.5 mEq/L may produce ataxia, clonic movements, possible seizures, and hypotension. Fine hand tremors, slurred speech, and nausea and vomiting are indicative of lithium levels less than 1.5 mEq/L.

A nurse is caring for a 4-year-old child who is receiving a barbiturate. What common adverse effect would the nurse assess for? A) Decrease in respirations B) Vomiting C) Excitability D) Dry mucous membranes

Ans: C Feedback: The barbiturates, being older drugs, have established pediatric dosages. These drugs must be used with caution because of the often unexpected responses. Children must be monitored very closely for central nervous system (CNS) depression and excitability. The most common adverse effects are related to general CNS depression. Other CNS effects may include drowsiness, somnolence, lethargy, ataxia, vertigo, a feeling of a hangover, thinking abnormalities, paradoxical excitement, anxiety, and hallucinations. Alteration in respirations and dried mucous membranes are adverse effects of antihistamines, which can be given to calm children or induce sleep. Vomiting could occur with the use of paraldehyde due to the unpleasant taste and odor of the drug.

The nurse is caring for a patient who experiences anxiety and insomnia and is prescribed benzodiazepines. When developing the plan of care, what would be an appropriate nursing diagnosis related to potential adverse effects of the drug? A) Provide patient teaching about drug therapy. B) Anxiety related to drug therapy. C) Risk for injury related to central nervous system (CNS) effects. D) Avoid preventable adverse effects, including abuse and dependence.

Ans: C Feedback: The most appropriate nursing diagnosis related to adverse effects of the drug is risk for injury related to CNS effects because benzodiazepines can have many CNS adverse effects. Anxiety is the condition for which drug therapy is prescribed not related to drug therapy. Patient teaching and avoiding adverse effects are interventions and not nursing diagnoses.

A psychotic patient is admitted through the emergency department. The physician has ordered chlorpromazine (Thorazine) 25 mg intramuscularly. After administration of the medication, what is the nurses priority to evaluate? A) The patients ability to ambulate B) Return of the patients appetite C) A decrease in psychotic symptoms D) Blood pressure and pulse

Ans: C Feedback: The nurse will evaluate the effectiveness of the drug in diminishing psychotic symptoms because this is the purpose of administering the drug. Monitoring blood pressure, pulse, and appetite is part of all patient care but is not the priority evaluation criterion for this patient. The ability to ambulate and maintain adequate nutrition would be assessed but is not the priority evaluation for this patient.

A nurse is about to administer a parenteral benzodiazepine to a female patient in the hospital before the performance of a procedure. What is the priority nursing action before administration of the drug? A) Make sure that the side rails are up and the bed is in the lowest position. B) Close the blinds and ensure appropriate room temperature for the patient. C) Help the patient out of bed to the bathroom and encourage her to void. D) Ask all visitors to leave the room and remain in the waiting area.

Ans: C Feedback: The priority action would be to help the patient up to void. After the medication is administered the patient should not get out of bed because of possibly injury due to drowsiness. Safety should always be the priority concern. After administration of the drug the nurse would ask visitors to leave before beginning the procedure, make the room conducive to rest and sleep, and make sure that both side rails are up and the bed is in the lowest position

A 16-year-old youth has just been diagnosed with schizophrenia. The parents ask the nurse what causes schizophrenia. What would be the nurses best response? A) Schizophrenia is caused by pain that the brain perceives. B) Schizophrenia is thought to occur due to trauma experienced in childhood. C) Schizophrenia is thought to reflect a fundamental biochemical abnormality. D) Schizophrenia is caused by seizure activity in the brain.

Ans: C Feedback: This disorder, which seems to have a very strong genetic association, may reflect a fundamental biochemical abnormality. Mental disorders are now thought to be caused by some inherent dysfunction within the brain that leads to abnormal thought processes and responses. Schizophrenia is not caused by pain, childhood trauma, or seizure activity. 12.

According to the Gate Control Theory, what interventions by the nurse could help to block pain impulses? A) Administration of opioid medications B) Administration of narcotic agonist-antagonists C) Back massage D) Acupuncture

Ans: C: Back massage Feedback: According to the gate control theory, the transmission of these impulses can be modulated or adjusted all along these tracts. All along the spinal cord, interneurons can act as gates by blocking the ascending transmission of pain impulses. It is thought that the gates can be closed by stimulation of the larger A fibers and by descending impulses coming down the spinal cord from higher levels in such areas as the cerebral cortex, the limbic system, and the reticular activating system. Administration of medications does not use the Gate Control Theory. Acupuncture uses the Gate Control Theory but is not performed by the nurse.

The nurse is teaching the soon-to-be-discharged patient, diagnosed with schizophrenia, about his medications. What is a priority teaching point for this patient? A) The patient must eat three nutritious meals daily. B) Over-the-counter medications may be taken with antipsychotic drugs. C) Cough medicines potentiate the actions of antipsychotic drugs. D) Alcohol consumption should be avoided.

Ans: D Feedback: Alcohol consumption should be avoided because it increases the central nervous system (CNS) effects of the drug and may cause excessive drowsiness and decreased awareness of safety hazards in the environment. Some patients may find it easier and more effective to eat five small meals rather than three nutritious meals. While promoting good nutrition is good practice, it is not the priority. Drugdrug interactions with antipsychotic drugs are common so the nurse would teach the patient not to take any medication without consulting with the doctor or a pharmacist to make sure it is safe.

The nurse is caring for four patients. Which patient would the nurse know that clozapine (Clozaril) is contraindicated for? A) 17-year-old adolescent B) 23-year-old with diabetes insipidus C) 32-year-old with osteoarthritis D) 45-year-old with bone marrow depression

Ans: D Feedback: Clozapine is associated with bone marrow suppression, a life-threatening decrease in white blood cells. Because of their wide-ranging adverse effects, antipsychotic drugs may cause or aggravate various conditions. They should be used very cautiously in patients with liver damage, coronary artery disease, cerebrovascular disease, Parkinsonism, bone marrow depression, severe hypotension or hypertension, coma, or severely depressed states. Options A, B, and C are incorrect.

For what purpose would the nurse choose to administer a hypnotic instead of another classification of antianxiety drug? A) Treating insomnia B) Treating seizure disorder C) Treating panic attach D) Treating confusion and agitation

Ans: D Feedback: Hypnotics are used to help people fall asleep by causing sedation. Drugs that are effective hypnotics act on the reticular activating system (RAS) and block the brains response to incoming stimuli. Hypnotics would not be the most effective drugs to treat seizure disorders, panic attack, or confusion with agitation.

An older adult African American patient comes to the clinic and is diagnosed with generalized anxiety disorder (GAD). The physician orders oral flurazepam 30 mg. What is the nurses priority action? A) Teach the patient about the prescribed medication. B) Administer the first dose of medication. C) Tell patient to take first dosage after driving home. D) Talk to the physician about the dosage.

Ans: D Feedback: If an anxiolytic or hypnotic agent is the drug of choice for an African American patient, the smallest possible dose should be used, and the patient should be monitored very closely during the first week of treatment. Dosage adjustments are necessary to achieve the most effective dose with the fewest adverse effects. In addition, older adults also require careful titration of dosage. Older patients may be more susceptible to the adverse effects of these drugs, from unanticipated central nervous system (CNS) adverse effects including increased sedation, dizziness, and even hallucinations. Dosages of all of these drugs should be reduced and the patient should be monitored very closely for toxic effects and to provide safety measures if CNS effects do occur. As a result, the priority action is to talk to the physician about the dosage. The other actions may be appropriate after a proper dosage is ordered.

The nurse is preparing to administer methylphenidate to the child admitted to the pediatric unit after breaking a leg when jumping off the garage roof at home. Where will the nurse find the medication? A) In the patients drawer B) In the refrigerator C) At the patients bedside D) In the controlled substance cabinet

Ans: D Feedback: Methylphenidate is a controlled medication due to risk for physical and psychological dependence. As a result, the drug would be found in the controlled substance cabinet.

A geriatric patient received a narcotic analgesic before leaving the post-anesthesia care unit to return to the regular unit. What is the priority nursing action for the nurse receiving the patient on the regular unit? A) Administer a non-steroidal anti-inflammatory drug. B) Encourage fluids. C) Create a restful, dark, quiet environment. D) Put side rails up and place bed in low position.

Ans: D Feedback: Older patients are more susceptible to the central nervous system effects of narcotics; it is important to ensure their safety by using side rails and placing the bed in the low position in case the patient tries to get up unaided. Postoperative patients are allowed nothing by mouth until bowel function returns so an oral medication or encouraging fluids would not be appropriate. This patient will require careful observation for respiratory depression, so a dark room would be unsafe.

Generalized Anxiety Disorder

Assessment: - Severe Anxiety - Motor Tension (restlessness, quickly fatigued, shakiness, tension - Autonomic Hyperactivity (SOB, palpitations, dizziness, frequency, diaphoresis) - Difficulty sleeping and concentrating; irritabity - Nervousness - Low self-esteem Interventions: - Assess anxiety level - Identify the relationship between the stressor and level of anxiety - Decrease environmental stimuli - encourage coping mechanisms

The nurse works on an inpatient mental health unit. When administering antipsychotic medications, what patient would the nurse expect to require a standard dosage? A) African American adolescent diagnosed with schizophrenia B) Malaysian middle adult diagnosed with bipolar disorder C) Iranian older adult diagnosed with schizophrenia D) Caucasian young adult diagnosed with bipolar disorder

Ans: D Feedback: Only the Caucasian young adult has no indications for administering a smaller than usual dosage. African Americans respond more rapidly to antipsychotic medications and have a greater risk for development of disfiguring adverse effects, such as tardive dyskinesia. Consequently, these patients should be started off at the lowest possible dose and monitored closely. Patients in Asian countries (e.g., India, Turkey, Malaysia, China, Japan, Indonesia) receive lower doses of neuroleptics and lithium to achieve the same therapeutic response as seen in patients in the United States. Arab American patients metabolize antipsychotic medications more slowly than Asian Americans do and may require lower doses to achieve the same therapeutic effects as in Caucasians.

The patient taking an antipsychotic drug asks the nurse how long he will continue to feel the effects of the drug after stopping the medication. What is the nurses best response? A) 2 to 4 hours B) 2 to 4 weeks C) 2 to 4 months D) 6 months

Ans: D Feedback: The antipsychotics are widely distributed in the tissues and are often stored there, being released for up to 6 months after the drug therapy has been stopped.

A nurse is caring for a 9-year-old patient and has received an order for diazepam (Valium) 10 mg given orally q.i.d. What is the nurses priority action? A) Perform hand hygiene and prepare the drug. B) Send the order to the hospital pharmacy. C) Determine when to administer the first dose. D) Call the physician and question the order.

Ans: D Feedback: The first action of the nurse would be to call the physician and question the order. The normal oral dosage for a pediatric patient is 1 to 2.5 mg t.i.d. or q.i.d. The ordered dose would be unsafe for this patient. If the dosage was changed and the correct amount administered, the nurse would order the medication from the pharmacy if necessary and determine what time to start the medication. She would then wash her hands in preparation for administering the medication, but not until obtaining an appropriate dosage of medication.

A patient is being discharged home from the hospital after receiving treatment for pneumonia. The patient is going home and continuing to take the same drugs he or she was taking before he or she was hospitalized. These drugs include an antianxiety medication and a medication for insomnia. The home care nurse is following this patient. On the initial visit what is the nurses priority teaching point? A) The names and purposes of medications prescribed B) How to contact the provider if needed C) The importance of taking medications for insomnia only occasionally D) Warning signs that may indicate serious adverse effects

Ans: D Feedback: The home care nurse should provide thorough patient teaching, with a priority teaching point being the warning signs the patient may experience that indicate a serious adverse effect. Although this may have been discussed by the discharging nurse in the hospital, this is essential information for the patient to thoroughly understand. By the time the home care nurse visits, the patient should already have filled the prescriptions and know the names and purposes of the medications prescribed from the hospital nurse but it is a good idea to review this information, although it is not a priority. Medications for insomnia should be taken as prescribed. The patient should have received the providers contact information when leaving the hospital but the home care nurse may need to review this, even though it is not the priority teaching point.

The nurse is caring for a patient who has not been able to sleep. The physician orders a barbiturate medication for this patient. What adverse effect should the nurse teach the patient about? A) Double vision B) Paranoia C) Tinnitus D) Thinking abnormalities

Ans: D Feedback: The most common adverse effects are related to general central nervous system (CNS) depression. CNS effects may include drowsiness, somnolence, lethargy, ataxia, vertigo, a resembling a hangover, thinking abnormalities, paradoxical excitement, anxiety, and hallucinations. Barbiturate drugs generally do not cause double vision, paranoia, or tinnitus.

The nurse is caring for a patient taking an oral neuroleptic medication. What is the nurses priority assessment to monitor for? A) Urge incontinence B) Orthostatic hypotension C) Bradycardia D) Tardive dyskinesia

Ans: D Feedback: The nurse would monitor for and teach the patient and family about tardive dyskinesias because it is such a common adverse effect with continued use of the drug. Oral neuroleptic agents do not cause urge incontinence, orthostatic hypotension, or bradycardia.

A nurse is caring for a 6-year-old patient after surgery. The child has an order for meperidine (Demerol) 1.8 mg/kg IM every 3 to 4 hour as needed for pain. The child weighs 30 kg and the meperidine is available as 50 mg/mL. How many mL will the nurse administer per dose? A) 1 mL B) 1.8 mL C) 0.8 mL D) 1.08 mL

Ans: D Feedback: To calculate the correct amount to be administered, first multiply 1.8 mg times 30 kg (54 mg). Next determine the volume in mL that 54 mg is equal to (50 mg: 1 mL as 54 mg: mL). Solve forx (50x is equal to 54 mg; 54 divided by 50 is equal to 1.08 mL).

A patient presents at the emergency department with respiratory depression and excessive sedation. The family tells the nurse that the patient has been taking medication throughout the evening and gives the nurse an almost empty bottle of benzodiazepines. What other adverse effects would the nurse assess this patient for? A) Seizures B) Tachycardia C) Headache D) Coma

Ans: D Feedback: Toxic effects of benzodiazepines include excessive sedation, respiratory depression, and coma. Flumazenil (Anexate) is a specific antidote that competes with benzodiazepines for benzodiazepine receptors and reverses toxicity. Seizures, tachycardia, and headache would not normally be associated with benzodiazepine toxicity.

A 72-year-old patient presents at the emergency department with respiratory depression and excessive sedation. The family tells the nurse that the patient has been taking medication throughout the evening. The nurse suspects benzodiazepine overdose and would expect what drug to be ordered? A) Valium B) Phenergan C) Hydroxyzine D) Flumazenil

Ans: D Feedback: Toxic effects of benzodiazepines include excessive sedation, respiratory depression, and coma. Flumazenil is an antidote for the benzodiazepines. Hydroxyzine is an antihistamine with anticholinergic effects and would not be appropriate for this patient. Valium would enhance the effects of benzodiazepines. Phenergan is not indicated for this patient; it is similar in actions to hydroxyzine.

A 12-year-old patient is hospitalized with severe depression. The patient has been taking a selective serotonin reuptake inhibitor (SSRI). What is the priority nursing action for the patient? A) Monitor food intake for levels of tyramine. B) Assess for weight loss and difficulty sleeping. C) Monitor the patient for severe headaches. D) Implement suicide precautions.

Ans: D : Implement suicide percautions Feedback: Recent studies have linked the incidence of suicide attempts to the use of SSRIs in pediatric patients (see box 21.3 Focus on the Evidence). The priority concern for the nurse would be safety for the patient. Severe headache and reactions to tyramine-containing foods are associated with monoamine oxidase therapy. Weight loss and difficulty sleeping are of a lower priority concern than the patients safety.

Depression treatment

Antidepressants SSRIs Prozac (dry mouth, insomnia, nausea, rash) TCAs amitriptyline (constipation, dry mouth, arrhythmia's, heart), MAOs isocarboxazid (dizzy, headache, insomnia, weight gain, forgetfulness, urinary hesitancy, weakness), ECT, individual or group therapy

Obsessive-Compulsive Disorder

Anxiety Associated with repetitive thoughts (obsessions) or irresistible impulses (compulsions) to perform an action. Major Symptoms: Fear of losing control Compulsions are due to anxiety which may or may not be related to the obsession Assessment: - Use of defense mechanisms (Repression, Isolation, Undoing) - Magical Thinking (belief that one's thoughts or wishes can control other people or events) - Evidence of delusional thought content - Difficult with relationships - Interference with normal activities (excessive washing hands) - Safety issues related to ritualistic activity - Recurring intrusive thoughts Interventions: - Provide for physical needs - Allow compulsions but consider safety - Explore the purpose of the behavior - Limit the allowed time for compulsions and work to gradually decrease it. - Administer anti-anxiety medications, SSRIs, and Tricyclic Antidepressants as prescribed - Best time to educate is at the end of compulsion (anxiety is lowest leading to the highest ability to learn) - Alleviate anxiety that causes the obsessions and compulsions Interfering with compulsions can increase anxiety; let them happen as long as they are safe

Domestic Violence

Any abuse, violence, or threatening act inflicted by one family member on another.

Affect

Appearance of observable emotions

Which nursing intervention strategy is most appropriate to implement initially with a suicidal client?

Ask a direct question such as, Do you ever think about killing yourself?

12. A client diagnosed with a neurocognitive disorder is exhibiting behavioral problems on a daily basis. At change of shift, the clients behavior escalates from pacing to screaming and flailing. Initially, which action should a nurse implement in this situation?

Assess environmental triggers and potential unmet needs. The initial nursing action is to assess environmental triggers and potential unmet needs. Due to the cognitive decline experienced in a client diagnosed with neurocognitive disorder, communication skills may be limited. The client may become disoriented and frustrated.

24. A client has been brought to the emergency department for signs and symptoms of Chronic Obstructive Pulmonary Disease (COPD). The client has a history of a suicide attempt 1 year ago. Which nursing intervention would take priority in this situation?

Assessing the clients pulse oximetry and vital signs It is important to prioritize client interventions that assess the symptoms of COPD prior to any other nursing intervention. Physical needs must be prioritized according to Maslows hierarchy of needs. This clients problems with oxygenation will take priority over assessing for current suicidal ideations

Alcoholism

Assessment: - Binge drinking - Morning drinking (eye opener) - Gastritis - Poor work ethic and legal problems - Defense Mechanisms (denial, projection, and rationalization) - Family history of alcoholism Withdrawal Symptoms: - Can occur after 4-6 hours of drinking - Autonomic hyperactivity (increase in vital signs) - Delirium tremens (48-72 hours after) - Grand mal seizures - Chronic Illnesses (gastritis, cirrhosis and hepatitis, korsakoff syndrome, malnutrition, pancreatitis, peripheral neuropathy) Interventions: - Maintain safety, nutrition, hygiene, and rest - Monitor vital signs, I&O, electrolytes - Prevent aspiration - Reduce environmental stimuli - Provide protein, vitamin supplements (especially B), and antianxiety medications (Librium/Ativan) - Provide emotional support Rehabilitation: - Set limits on behavior - Help increase self-esteem - Find alternative coping mechanisms - Identify a support group (Such as AA) - Identify activities and relationships not related to drinking

Intimate-Partner Violence

Assessment: - Delay between time of injury and time of treatment - Anxiousness when answering questions about injury - Abdominal injuries during pregnancy - Looks to abuser to answer questions about injuries - Feeling of responsibility for "provoking" partner Interventions: - Establish trust - Treat physical needs first - Document factual, objective info about the condition of the client, injuries, and interaction with partner or family - Determine potential for further violence - Interview when abuser is not present - Assist with referral to a shelter if necessary - Assist client with contacting authorities if charges are to be pressed

Bipolar Disorder (Assessment and Intervention)

Assessment: - Determine level of depression and mania - Assess nutrition and hydration status - Assess level of fatigue - Assess danger to self and other in relation to level of impulse impairment present Interventions: - Provide nutrition, rest, and hydration (small frequent meals) - Provide safe environment (altercations with other clients) - Decrease environmental stimulation - Use limit setting - Avoid giving attention to bizarre behavior - Engage in simple, active, noncompetitive activities - Administer lithium, sedatives, and anti-psychotics as prescribed (lithium level is 0.8-1.4) - Avoid arguing or becoming defensive when a bipolar puts you down (common) - If patient becomes abusive: (redirect negative behavior or verbal abuse in a calm. firm, nondefensive manner, suggest a walk or other physical activity, set limits for intrusive behavior, seclude or sedate as a last resort)

Depressive Disorders (Assessment and Interventions)

Assessment: - Determine type of depression (exogenous [caused by external factors] or endogenous [caused by an internal biologic deficiency]) - Determine degree of depression - Determine risk for suicide - Arrange for lab tests (Dexamethasone-suppression test [indirect marker of depression if level greater than 5mg/dL] and Biogenic Amines [Decreased seratonin and norepinephrine are indicative of depression]) Interventions: - Ask about plans to self-harm - Monitor sleep and nutrition habits - Assist with ADLs - Assess for elevation in mood (increases risk of suicide) - Insist on participation in activities (do not give a choice) - Administer antidepressent medication - Depressed client find it hard to accept compliments --> instead comment on signs of improved behavior (such as doing an ADL) - Improvement is noted when the client takes an interest in themselves

Panic Disorders

Assessment: - Incapacitating fear or discomfort - Fear of external subject - Client knows fear is excessive but "can't help it" - Use of coping mechanisms (displacement, projection, repression, sublimation) - Autonomic Hyperactivity - Panic Attacks (peak at 10 minutes and last up to 30 with gradual return to normal functioning) - Disruption of personal life - Possible self medicating with drugs or alcohol to decrease anxiety Intervention: - Acknowledge the fear without exposing the fear to the client - Establish trust and then start desensitization - Use positive reinforcement when a decrease in phobic reaction occurs - Reduce environmental stimuli

Child Abuse

Assessment: - Indicators of child abuse (injuries not congruent with developmental age or skills, injuries not correlated with stated cause, delay in seeking medical care) - Bruises in various stages of healing - Burns - Fractures in various stages of healing - Parent using child to meet own needs - Parent critical of child; seldom touching or responding to child - Child frightened in the presence of parent - Hx of frequent moves, unstable employment, marital discord, or family violence - Bedwetting, soiling - Failure to thrive Interventions: - Nurses are LEGALLY required to report all cases of suspected child abuse to the appropriate local or state agency - Take color photographs of injuries - Establish trust with the child and care for immediate, physical needs of the child - Recognize feelings of contempt for the parents - Support family therapy - Select only one nurse to work with the child. Trust is hard to get from a child, so one nurse helps build that trust.

Drug Abuse

Assessment: - Pattern of drug use (what and how much) - Physical evidence of drug usage (needle marks, cellulitis, poor nutrition, inflammation of nasal passages) - Drug dependency - Symptoms of withdrawal or overdose for that drug Interventions: - Assess LOC and vital signs - Monitor I&O and electrolytes - Confront denial - Identify stressors - Explore coping strategies

Rape and Sexual Assault

Assessment: - Physical assessment with careful documentation of injuries - Emotional status (self-blame, anxiety, fear, humiliation, disbelief, anger) - Coping behaviors - Identification of support system - Details of the assault Interventions: - Communicate nonjudgmental acceptance - Provide physical care to injuries - Tell them what you are doing before you do it - Document factual, objective statements of physical assessment; record EXACT words of the client - Notify police and encourage victim to prosecute - Notify rape crisis team or counselor if available - Allow discussion of feelings - Advise potential for venereal disease, pregnancy, and HIV - Support the client, family, and friends. - High risk for PTSD

How does a democratic form of self-government in the milieu contribute to client therapy?

By setting community expectations wherein all clients are treated on an equal basis

The physician orders Sertraline (Zoloft) 50 mg PO bid for Margaret, a 60-year-old woman with major depressive disorder. A fter three days of taking the medication Margaret says to the nurse "I don't think this medicine is doing any good I don't feel a bit better" What is the most appropriate response by the nurse? a. cheer up Margaret you have so much to be happy about b. sometimes it takes a few weeks for the medicine to bring about an improvement in symptoms c. I'll report that to the physician Margaret maybe he will order something different d. try not to do when your symptoms Margaret why don't you join the others in the dayroom

B

a newly admitted dressed client isolates herself in her room and just sits and stares off into space. How should the nurse begin an initial therapeutic relationship with the pt? a. say, "come with me. I will go to therapy with you" b. make frequent short visits to her room and sit with her c. offer to introduce her to the other pts d. help her to identify stressors in her life that precipitate crisis

B

A group of patients are being screened to see which patients would be the best candidate for a psychotherapeutic drug trial that helps people concentrate longer on activities. Which patient would be best suited for this trial? A) A 28-year-old salesperson who alternates between overactivity and periods of depression B) A 32-year-old hyperactive nursing student who cannot focus long enough to take a test C) A 55-year-old physician who suddenly falls asleep during the day without warning D) A 16-year-old youth who say he can make the light turn on by pointing at it and hears

B Feedback: Attention-deficit disorders involve various conditions characterized by an inability to concentrate on one activity for longer than a few minutes. The nursing student needing accommodations has an attention-deficit disorder. The salesperson exhibits signs of mania, which are characterized by periods of extreme overactivity and excitement followed by extreme depression. The physician is experiencing narcolepsy, which is defined as daytime sleepiness and sudden periods of loss of wakefulness. The teenager is schizophrenic and is exhibiting paranoia, hallucinations, and delusions.

In teaching a pt about his antidepressant medication, fluoxetine, which would the nurse include (select all that apply) a. don't eat chocolate while taking this med b. keep taking this med even if you don't feel that it is helping. It sometimes takes a while to take effect c. don't take this medication with the migraine drug "triptans" d. go to the lab each week to have your blood drawn for therapeutic levels of the drug e. this drug causes a high degree of sedation, so take it before bedtime

B, C

The client is placed on escitalopram 10 mg daily. For which adverse effect does the nurse instruct the family to observe? A. Photophobia B. Dizziness C. Epistaxis D. Hypertensive crisis

B. Dizziness Escitalopram is a selective serotonin reuptake inhibitor used to treat depression, OCD, adverse effects include insomnia, dizziness, diarrhea, nausea, sexual dysfunction, lack of sex drive; monitor for suicidal tendencies, administer in morning or evening without regard to food.

A nurse attends an interdisciplinary team meeting on an inpatient unit. Which of the following individuals are typically included as members of the interdisciplinary treatment team in psychiatry? Select all that apply.

B. Occupational therapist C. Recreational therapist D. Social worker E. Mental health technician

The young client is brought to the ER by a friend. The client is agitated and is screaming, "I can't stop seeing things. Help me, I'm going crazy." The friend reports to the nurse that the client took some lysergic acid diethylamide earlier in the day. It is most important for the nurse to take which action? A. Give the client reflective feedback. B. Stay with the client and quietly attempt to talk the client down. C. Set limits on the client's behavior. D. Place the client in a well-lighted room close to the nurse's station.

B. Stay with the client and quietly attempt to talk the client down. Clients who have taken LSD frequently panic in response to sensory perceptual changes; most important therapeutic nursing measure is to stay with the client and gently talk the client down, reassuring that response to the LSD will pass.

The nurse prepares to lead a group session for clients with a dependence on alcohol. The nurse knows that an alcoholic client drinks because of which reason? A. The alcoholic enjoys the feeling of being intoxicated. B. The alcoholic uses alcohol to escape from problems. C. The alcoholic has a greater alcohol tolerance than most people. D. The alcoholic performs more efficiently when drinking.

B. The alcoholic uses alcohol to escape from problems. Alcoholics use alcohol to escape from problems as a way to decrease anxiety and tension; frequently, alcoholics will use alcohol to cover depression or anxiety rather than seek out treatment for these problems.

The woman is admitted to the hospital for a possible mastectomy. On the evening before his wife's scheduled surgery, the husband appears tense and paces up and down the hall. Which comment by the nurse to the husband is best? A. We will do everything we can to help your wife. B. This is an upsetting experience for you and your wife. C. You will feel relieved once the surgery is over. D. I think it might help if we discussed your wife's surgery.

B. This is an upsetting experience for you and your wife. Good broad opening; comments on the husband's nonverbal behavior, and invites him to explore matters with the nurse.

The nurse cares for the clients in the mental health clinic. The client with depression joins an ongoing therapy group. What is the goal of the group therapy? A. to introduce the client to other clients. B. To communicate acceptance to the client. C. To encourage decision-making. D. To increase the client's sense of responsibility.

B. To communicate acceptance to the client. The most important thing to communicate initially is a sense of acceptance; acceptance by others paves the way for self-acceptance.

Which of the following are accurate descriptors of a therapeutic community? Select all that apply.

B. Unit responsibilities are assigned according to client capabilities. E. A democratic form of government exists.

Alprazolam

Benzodiazepine sedative−hypnotic: widely used in anxiety states, selectivity for panic attacks and phobias; possible antidepressant actions. Tox: psychologic and physical dependence, additive effects with other CNS depressants

Antianxiety Medications

Benzodiazepines Nonbenzodiazepines

anxiety treatments

Benzos for no more than 6 weeks, Buspirone, TCAs, SSRIs, relaxation techniques, biofeedback, cog therapy

Introjection

Incorporation of values or qualities of an admired person or group into one's own ego structure

Margaret age 68 is a widow of six months. Since her husband died, her sister reports that Margaret has become socially withdrawn, has lost weight, and does a little more each day then visit the cemetery where her husband was buried. She told her sister today that she "didn't have anything more to live for". She's been hospitalized with major depressive disorder. The priority nursing diagnosis for Margaret would be: a. in-balanced nutrition: less than body requirements b. complicated grieving c. Risk for suicide d. social isolation

C

a pt who's husband died 6 months ago is dx with MDD. She tells the nurse, " I start feeling angry that Harold died and left me all alone! He should've stopped smoking years ago. But then I feel guilty for saying this" What is an appropriate response by the nurse? a. Yes he should have stopped smoking, then he probably wouldn't have gotten lung cancer b. I can understand how you must feel c. those feelings are a normal part of the grief response d. just think about the good times you had while he was alive

C

The nurse plans care for the client with a history of substance abuse. It is most important for the nurse to select which approach? A. A structured but permissive setting. B. An environment that increases reality testing. C. A structured, nonpermissive setting. D. An environment that decreases stimuli and redirects behavior.

C. A structured, nonpermissive setting. A structured and nonpermissive setting is the best environment for a client who is a substance abuser; the goal of treatment is to have the client tolerate increasing amounts of anxiety without the need for substances; intervention involved limit-setting and having the client express frustration while exploring alternate coping patterns.

Which statement, if made by the alcoholic client to the nurse, indicates the client has an accurate understanding of the problem? A. When I can learn to stop after one drink, I will have my problems beat. B. When my family and work problems go away, I won't need alcohol anymore. C. I can't seem to cope with my problems without drinking. D. In my business, most people work hard and drink too much.

C. I can't seem to cope with my problems without drinking. When the client acknowledges that alcohol is used to cope with problems, the client is beginning to break through denial.

When intervening with a violent client, the nurse should take which action? A. Tell the client that they have no control over their behavior. B. Point out that the client is making others anxious. C. Identify the nurse to client and remain calm. D. Touch the client gently to offer reassurance.

C. Identify the nurse to client and remain calm. By identifying him/herself, the nurse establishes a sense of authority and trust; in remaining calm, the nurse helps the client to feel less frightened and panicked.

The nurse cares for the client with depression who attempts suicide. The nurse understands which is the most likely reason that the client attempts suicide? A. The client is suspicious and mistrustful. B. The client consciously wishes to manipulate others. C. The client feels overwhelmed and helpless. D. The client wants to gain attention.

C. The client feels overwhelmed and helpless. The reason a person attempts suicide is unclear, but there is a general agreement that the suicidal act follows a precipitating event; may be loss of loved one, or ill health; when it seems that everything has failed and nothing has brought relief, the person loses hope and may turn to thoughts of suicide; at this point, suicide seems to be the only solution to overwhelming emotional pain.

The client appears angry and demanding following a below-the-knee amputation. Which interpretation by the nurse of this client's behavior is most justifiable? A. The client is seeking attention to compensate for the loss. B. The client is placing the blame for difficulties on others. C. The client is having difficulty accepting the new body image. D. The client feels alienated by the hospital staff.

C. The client is having difficulty accepting the new body image. Client is having difficulty accepting altered body image, and may worry about how others will view the new body; is a problem after loss of a body part.

A client in the hypertension clinic expresses worry to the nurse that his wife has been unemployed for more than 6 months, and that he is afraid that soon they will be unable to pay rent. Which of these responses by the nurse is MOST appropriate? A. These things always seem worse than they really are. B. It's important for your blood pressure that you not worry too much about that. C. You're worried that you won't be able to pay rent? D. I will refer you to a social worker.

C. You're worried that you won't be able to pay rent? Reflects feelings of the client; allows the client to focus on what he said and what his feelings are.

Heroin Overdose

CNS depression. constricted pupils, slow shallow breathing, drowsiness, slurred speech.

Disruptive, Impulse-control, and Conduct Disorder (Oppositional Defiant Disorder)

Callous and unemotional interpersonal relationships. Symptoms cause significant impairment in social, educational, and occupational functioning Definition: ODD is characterized by behavior that causes problems at school, work, or home Assessment of Conduct Disorder: - Physical fighting - Running from home - Lying, stealing, vandalism, or arson - Cruelty to animals -Use of alcohol or drugs Assessment of Defiant Disorder: - Argumentativeness - Blaming others for own problems - Defying rules or authority - Use of obscene language Interventions: - Assess verbal and nonverbal cues for escalating behavior to decrease outbursts - Use "show of force" when the child is out of control - Use "quiet room" to control external stimuli - Teach coping skills (punching a pillow or punching bag) - Role-play coping strategies - Monitor for fulfilling prophesy ("Mom says I'm a trouble maker so I have to be one") - Confront bad behaviors to give a sense of security (such as lying) - Use consistent interventions to prevent manipulation and help the client develop self-control

Which nursing response is an example of the nontherapeutic communication block of requesting an explanation?

Can you tell me why you said that?

Panic Anxiety

Causes acute psychosis (client cannot tell what is real and unreal). Loss of ability to problem solve or think logically. Can exhibit angry, aggressive, or withdrawn behavior and may lead to crying. Requires immediate intervention

Benzodiazepines

Chlordiazepoxide (Librium) Diazepam (Valium) Clorazepate Dipotassium (Tranxene) Lorazepam (Ativan)

Phenothiazines

Chlorpromazine (Thorazine) Trifluoperazine (Stelazine) Thioridazine (Mellaril) Perphenazine (Trilafon) Triflupromazine (Vesprin) Loxapine (Loxitane)

Schizophrenia

Chronic illness resulting in psychotic behavior, autism (withdrawal from relationships and world), inappropriate or no display of feelings, hypochondriasis and depersonalization, hallucinations, delusions, short attention span, regression, inability to meet survival needs

Moderate Anxiety

Client is still able to function and focus. Speech and volume increase. May lead to nausea, headaches, diarrhea, and tachycardia. Sensory awareness dulled

Somatic Symptom Disorder

Client turns any symptom into worst case scenario with great anxiety and demands excessive testing from health care professionals. Assessment - Preoccupation with pain or bodily function for at least 6 months. - "Doctor shopping" - looking for a diagnosis - No emotional concern regarding the physical impairment - Elevated vitals (like a panic attack) - Excessive use of analgesics - Suicidal thoughts - Social or occupational impairment Interventions: - Record pain duration and severity with attention to factors that cause it. - Encourage expression of anger - Focus interactions away from self and pain - Help client identify the connection between the pain and anxiety - Increase attention given to client as a reward for the client no focusing on self or physical symptoms - Reduce anxiety - Use disorder specific medication for depression/anxiety - Acknowledge that symptom or complaint/reaffirm that the tests came back negative and reveal no pathology/determine secondary gains acquired by the client (gains for playing the sick role)

Rape Trauma

Clothing, hair samples, NPO, alert for internal injuries, focus on here and now, write out treatments and appointments for client due to anxiety causing forgetfulness, give client referral for legal, supportive psychotherapy, rape crisis center.

Undoing

Communication or behavior done to negate a previously unacceptable act

Milieu Management

Containment, support, structure, involvement, validation. regularly evaluate patients symptoms and therapeutic needs. oversee activities, keep a schedule. interact with others, respect others and the environment, keep follow up appointments with therapist.

nurse that she is having trouble attending to daily activities and the pain is "excruciating," so she wants stronger pain medication immediately. What should this patient's behavior suggest to the nurse? 1. The patient could have a substance abuse problem. 2. A nerve was cut during the procedure. 3. The patient was not given strong enough postoperative pain medication. 4. The patient is under stress to return to work.

Correct Answer: 1 Global Rationale: Abusers often have a low tolerance for frustration and pain. Since the patient had surgery the day before and is now reporting excruciating pain and demanding more pain medication, this could indicate that the patient has a substance abuse problem. There would be a sensorimotor deficit if the nerve had been cut during the procedure. There is not enough information to determine whether the patient was not given strong enough postoperative pain medication or whether the patient is under stress to return to work.

A patient complaining of back pain tells the nurse that he needs several refills on any prescription since he takes the medication more frequently than prescribed. What should this information suggest to the nurse? 1. The patient has lost control over the consumption of the pain medication. 2. The patient is impulsive. 3. The patient uses the pain medication to fit in with a peer group. 4. The patient uses the pain medication to overcome low self-esteem.

Correct Answer: 1 Global Rationale: Addictive behavior associated with substance use is characterized by loss of control over consumption. The patient states that he is taking the medication more frequently that prescribed, which could indicate a loss of control. There is not enough information to support the idea that the patient is being impulsive, is using the pain medication to fit in with a peer group, or is using the pain medication to overcome low self-esteem.

with alcohol. For which adverse effect should the nurse assess this patient? 1. respiratory depression 2. seizure activity 3. signs of withdrawal 4. signs of hallucinations

Correct Answer: 1 Global Rationale: Barbiturates are central nervous system depressants. Barbiturates and alcohol are a lethal combination. The patient who has ingested both items is at risk for varying degrees of sedation, up to coma and death. Seizure activity, signs of withdrawal, and hallucinations are not the greatest risks for this patient.

A patient is brought into the emergency department with a gunshot wound obtained while attempting to rob a convenience store. For which health problem should this patient be assessed? 1. drug and alcohol abuse 2. cardiovascular disease 3. respiratory disease 4. mental health disorder

Correct Answer: 1 Global Rationale: Because drug users are often rebellious against social norms and engage in risky behaviors such as stealing, the nurse should assess the patient for drug and alcohol abuse. Cardiovascular disease and respiratory disease are not usually manifested by engagement in risk-taking behavior. The patient may or may not have a mental health disorder. This type of disorder might be assessed while assessing for drug and alcohol abuse.

A patient involved in a minor accident reports having used "crank" an hour ago. The patient denies having used the drug before. What manifestations can the nurse anticipate occurring with this patient? 1. reports feelings of insomnia and confusion 2. shows increased strength and cognition 3. displays paranoia 4. exhibits hallucinations

Correct Answer: 1 Global Rationale: Crank is a form of methamphetamine. It will cause the patient to have insomnia and feel confusion. The patient will not show increased strength and cognition. Paranoia and hallucinations might be seen in an individual who has been using crank for a long period of time but not by someone who has used it once.

A patient is brought into the emergency department with dilated pupils, respiratory rate of six per minute, and seizure activity. What should the nurse prepare to administer to this patient? 1. nalozone (Narcan) 2. activated charcoal 3. ammonium chloride 4. diazepam (Valium)

Correct Answer: 1 Global Rationale: Dilated pupils, respiratory depression, and seizure activity are signs of heroin overdose. The nurse should prepare to administer nalozone (Narcan) to reverse the effects of central nervous system depression. Activated charcoal is used for alcohol or barbiturate overdose. Ammonium chloride is used for cocaine overdose. Diazepam (Valium) is used for LSD overdose.

A patient comes into the emergency department wanting to be checked for sexual activity since she was out the other night at a rave and cannot remember what occurred. The nurse realizes that which addictive substance would cause this effect? 1. ecstasy 2. crank 3. marijuana 4. alcohol

Correct Answer: 1 Global Rationale: Ecstasy is classified as a hallucinogen and has been associated with date rape. This drug will cause the user to have thoughts and feelings similar to those in dreams. Crank is a stimulant and would heighten the user's awareness. Marijuana does not usually cause an individual to forget events. Unless taken in high quantities over long periods of time, alcohol does not usually cause an individual to forget events.

A nurse is concerned about potential substance abuse by a coworker. What behavior warrants further investigation? 1. The coworker frequently wastes medications. 2. The coworker frequently requests the largest patient care assignment for the shift. 3. The coworker prefers not to be the "medication nurse" on the shift. 4. The coworker declines to take scheduled breaks.

Correct Answer: 1 Global Rationale: Excessive medication wasting could be a sign that a nurse is using or diverting drugs. Requesting a large patient care assignment would not be a characteristic of a nurse who is abusing substances. The nurse who is unable or unwilling to manage a patient care assignment could be a substance abuser. Requesting not to be the medication nurse would reduce access to potentially abusive substances. Taking frequent or lengthy breaks might signal substance abuse. Declining scheduled breaks is not characteristic of a substance abuser.

The mother of a patient admitted with alcohol abuse tells the nurse that alcohol is not consumed at home and the patient is adopted. What should the nurse suspect about the patient? 1. The patient's biological parents might have abused alcohol. 2. The patient spent time drinking with friends. 3. Consuming alcohol is a symptom of stress. 4. Alcoholism is a learned behavior.

Correct Answer: 1 Global Rationale: Genetic studies have been performed that suggest heredity plays a role in the development of alcoholism. Since the patient was adopted, the patient's biological parents may have abused alcohol. There is not enough information to support that the patient is spending time drinking with friends or the patient is consuming alcohol as a symptom of stress. There is also not enough evidence to support this patient's alcohol use as being a learned behavior.

Question 16 Type: MCSA A patient admitted with seizures is diagnosed with a perforated nasal septum. The nurse realizes that this patient most likely has abused which substance? 1. cocaine 2. marijuana 3. alcohol 4. barbiturates

Correct Answer: 1 Global Rationale: Long-term intranasal use of cocaine is associated with a perforated nasal septum. Severe overdose of cocaine can lead to a seizure disorder. Seizures and perforation of the nasal septum are not associated with marijuana, alcohol, or barbiturate abuse.

A patient comes to the emergency department with a PCP overdose. Which intervention should the nurse anticipate will be needed? 1. Administer haloperidol (Haldol) as prescribed 2. Induce vomiting 3. Talk the patient down 4. Administer nalozone (Narcan) as prescribed

Correct Answer: 1 Global Rationale: PCP overdose is associated with possible hypertensive crisis, respiratory arrest, hyperthermia, and seizures. The nurse should anticipate administering haloperidol (Haldol) as prescribed. Inducing vomiting is not treatment for PCP overdose. Talking the patient down is not recommended for PCP overdose. Nalozone (Narcan) is not a treatment for PCP overdose.

A nurse consistently arrives to work wearing a long-sleeved blouse despite the temperature being in the 90s and the air humid. When colleagues mention her attire, the nurse becomes defensive and isolates herself from others. What does the nurse's behavior suggest? 1. substance abuse 2. a long-standing illness 3. introverted behavior 4. low self-esteem

Correct Answer: 1 Global Rationale: Signs of drug use include wearing long sleeves in hot weather to cover up arms defensive behavior, and isolation. There is not enough information to support idea that the nurse has a long-standing illness, routinely engages in introverted behavior, or has low self-esteem.

While analyzing data collected during an assessment, the nurse realizes that a patient's risk for alcohol abuse is low. What information did the nurse use to come to this conclusion? 1. The patient is Asian American. 2. The patient is diagnosed with type 2 diabetes. 3. The patient has two children. 4. The patient is employed as an accountant

Correct Answer: 1 Global Rationale: Sociocultural factors often influence individuals' decisions regarding substance use. Asian Americans report the lowest prevalence of family history of alcoholism because of a deficiency of aldehyde dehydrogenase, which upon alcohol consumption results in toxic symptoms characterized by vomiting, flushing, and tachycardia. This is the assessment finding that would indicate that a patient's risk for alcohol abuse is low. There is no information to support the concept that an individual with type 2 diabetes, 2 children, or employment as an accountant would avoid alcohol.

A nurse has been convicted of driving while under the influence. What regulatory body will determine the status of the nurse's license? 1. state board of nursing 2. employee assistance program 3. state court system 4. American Nurses Association

Correct Answer: 1 Global Rationale: The nurse has been convicted, so the state board in his or her state will investigate and take action including censure, probation, or suspension of the nursing license. An employee assistance program may be involved but will not investigate the conviction. With the conviction, the court system has taken action and would be in the county, not the state. The American Nurses Association will not be involved.

A patient tells the nurse that he becomes very angry and abusive to his friends and family when he is unable to obtain an illegal substance. How should the nurse respond to this patient? 1. "Have you considered seeking treatment for this behavior?" 2. "You must not have many friends left." 3. "Are you this argumentative when at work?" 4. "What does your family say about this?"

Correct Answer: 1 Global Rationale: The patient demonstrates anger and abusive behavior when unable to obtain an illegal substance. This information should indicate to the nurse that the patient has a substance abuse problem. The best response for the nurse to make is to ask the patient if he has considered seeking treatment for this behavior. The nurse should not comment on the patient's number of friends. Asking if the patient is argumentative at work could incite the patient's anger and abusiveness. Asking the patient about his family could incite the patient's anger.

A patient tells the nurse that she started to have a glass of wine every evening at home after work to "unwind" and then realized that she cannot continue with her day unless she has the wine. The nurse realizes that this patient uses wine to 1. cope with day-to-day problems. 2. deal with difficulty expressing emotions. 3. suppress a genetic need for alcohol. 4. socialize with others.

Correct Answer: 1 Global Rationale: The use of a substance as a form of self-medication to cope with day-to-day problems can become a habit. Over time, it can become an addiction.There is no information to suggest that the patient is having difficulty expressing emotions or has a genetic need for alcohol. The patient is drinking at home after work, so there is no information to support the patient is using alcohol to socialize with others.

A patient with a history of chronic alcohol use is underweight and malnourished. What should the nurse expect to be prescribed to manage the patient's nutritional status? 1. thiamine (vitamin B1) 2. diazepam (Valium) 3. methadone 4. nalozone (Narcan)

Correct Answer: 1 Global Rationale: Thiamine (vitamin B1) is necessary to prevent the complications of chronic alcoholism such as Wernicke syndrome. Diazepam (Valium) is used in the acute treatment of LSD overdose. Methadone is prescribed to manage heroin cravings. Nalozone (Narcan) is used to treat the effects of central nervous system depression

After a patient undergoes surgery, the nurse notes that the analgesics prescribed are not relieving the patient's pain. A review of the patient's medical records reveals a history of alcohol abuse. What inferences can the nurse make? 1. The patient is likely cross-tolerant to the prescribed analgesic. 2. The patient has an unreported addiction to the pain medication being prescribed. 3. The patient has a history of using this medication at home. 4. The patient has a dual diagnosis relating to alcohol and drug addiction.

Correct Answer: 1 Rationale 1: Cross-tolerance results when tolerance to one substance also results in a tolerance to another drug. The patient's heavy use of alcohol has likely resulted in a tolerance to alcohol and, by association, to the prescribed analgesic. Rationale 2: There is no evidence that the patient is addicted to the medication. Rationale 3: There is no evidence that the patient takes the medication at home. Rationale 4: There is no evidence that the patient has a dual diagnosis related to alcohol and drug addiction.

The nurse is concerned that a colleague is experiencing depression from substance abuse. What does the nurse observe to make this assumption? 1. The colleague leaves the unit and cannot be located for long periods of time. 2. The colleague uses the visitor bathroom instead of employee bathroom. 3. The colleague often eats lunch away from the hospital. 4. The colleague complains of frequent headaches at work.

Correct Answer: 1 depression from substance abuse. Global Rationale: Leaving the unit and not being located for long periods of time are indicative of depression associated with substance abuse. Using the visitor bathroom instead of employee bathroom, eating lunch away from the hospital, and complaining of headaches at work are not indicative of the colleague experiencing depression because of substance abuse.

A patient tells the nurse that both of his parents are alcoholics and wonders about the likelihood of becoming an alcoholic as well. How should the nurse respond to the patient? 1. "There are studies that support a genetic link for developing alcoholism." 2. "Why are you concerned about becoming an alcoholic?" 3. "You will likely become an alcoholic." 4. "Don't worry about that."

Correct Answer: 1 Global Rationale: Genetic studies have been performed that suggest heredity plays a role in the development of alcoholism. The nurse should respond that that there are studies that support this link. The nurse should not question the patient's request for information about becoming an alcoholic like his parents. Although the patient does have an increased risk, stating that he will become an alcoholic is inappropriate. Telling the patient not to worry about becoming an alcoholic is also an inappropriate response.

The nurse is assessing a patient for alcohol abuse. On which mental health problems should the nurse focus during this assessment? Standard Text: Select all that apply. 1. psychosis 2. depression 3. malnutrition 4. Alzheimer disease 5. cerebrovascular accident

Correct Answer: 1, 2 Global Rationale: The most commonly co-occurring mental disorders in adults are alcohol abuse or alcohol dependence with depression or psychoses. Malnutrition is not a mental health problem. Alzheimer disease is not associated with alcohol abuse. It is not considered a mental health problem. Cerebrovascular accident is a cardiac disorder that affects blood flow to the brain. The patient may demonstrate alterations in behavior and consciousness; however, this is not a mental health problem.

The nurse is caring for a patient with chronic alcoholism. Which vitamins should the nurse expect this patient to be prescribed? Standard Text: Select all that apply. 1. thiamine (vitamin B1) 2. folic acid 3. cyanocobalamin (vitamin B12) 4. vitamin E 5. potassium chloride

Correct Answer: 1, 2 Global Rationale: Patients using alcohol over a long period of time will have a vitamin deficiency, especially of thiamine and folic acid. These two vitamins help prevent Wernicke encephalopathy. The other vitamins may be important, but would be found in a multivitamin. Potassium chloride is incorrect as it is a mineral/electrolyte.

The nurse reviews the long-term effects of alcohol use with a patient who has experienced chronic alcoholism for 25 years. Which patient statements indicate that addition teaching is required? Standard Text: Select all that apply. 1. "My yellow skin is really just caused by the thiamine deficiency." 2. "My problems with COPD weren't because I smoke; it's because I was drinking so much." 3. "I was surprised to learn that my coronary artery disease may have been worsened by my drinking." 4. "I think I would've stopped drinking a long time ago if I had known that it may have caused my impotence." 5. "My mom was an alcoholic and died from breast cancer and it may have been the result of her drinking."

Correct Answer: 1, 2 Global Rationale: Patients who have severely damaged their livers because of alcohol abuse may be more likely to exhibit yellow skin and sclera. Jaundice is unrelated to a vitamin B1 deficiency. The patient who smokes cigarettes is more likely to develop asthma. Asthma is not necessarily associated with drinking alcohol. The patient had an increased risk of developing heart problems by drinking alcohol. The patient had an increased chance of developing impotence by drinking. The patient's mother had an increased risk of developing breast cancer due to her alcoholism.

The patient has been brought to the emergency department after being found by his mother at home. The patient's blood alcohol level is currently 0.51%. Which findings are consistent with this information? Standard Text: Select all that apply. 1. A sternal rub was performed with no response elicited. 2. The patient's respiratory rate is eight breaths per minute. 3. The patient's radial and pedal pulses are bounding. 4. The patient states, "I sleep for a long time, but I never feel rested when I wake 5. The patient states, "I really think I can drive myself home. I am fine!"

Correct Answer: 1, 2 Global Rationale: With this blood alcohol level, the patient is likely to be in a coma. The patient's respiratory rate may be very depressed. The peripheral pulses are more likely to be weak and thready due to peripheral vascular collapse, which would make it difficult for the nurse to palpate. The patient is not likely to be talking to the nurse about the quality of his sleep or about his ability to drive.

The nurse is preparing material for a staff development presentation on substance abuse. Which information should the nurse include that addresses the family history of substance abuse? Standard Text: Select all that apply. 1. There is a genetic link between having an alcoholic parent and developing problems related to substance use. 2. These genetic issues are more often related to alcohol, marijuana, and tobacco use. 3. One type of genetically related alcoholism is associated with an antisocial personality disorder, early use, and an inability to stop drinking. 4. The patient has an increased likelihood of developing substance use problems when the mother of the patient is an alcoholic rather than the father. 5. Some studies indicate that adolescents who are more genetically prone to develop alcoholism were less likely to have other types of substance use problems.

Correct Answer: 1, 2, 3 Global Rationale: Children of alcoholics have a greater risk for developing substance use problems. These types of genetic issues lead to problems with alcohol, marijuana, and tobacco use. One type of alcoholism seen mostly in the sons of alcoholic fathers is associated with an inability to abstain, early onset, and an antisocial personality. These types of genetic issues are typically found in male relatives. Studies have shown that dopamine has been identified as the primary neurotransmitter responsible for sustaining the addictive quality of drugs and for increasing drug-seeking behavior.

The nurse identifies the problem of imbalanced nutrition due to insufficient intake for a patient hospitalized for substance abuse. What interventions should the nurse identify as appropriate for this patient? Standard Text: Select all that apply. 1. Monitor the CBC and liver enzymes 2. Collaborate with the dietician 3. Administer vitamins and diet supplements 4. Provide high fat, high carbohydrate diet 5. Restrict fluid intake to 1500 mL per day

Correct Answer: 1, 2, 3 Global Rationale: Lab values should be monitored to evaluate the extent of malnourishment. The dietician can help with meal planning for adequate nutrition and realistic weight gain. Vitamins and dietary supplements may be ordered to prevent complications from chronic alcoholism such as Wernicke syndrome. A high fat, high carbohydrate diet is not appropriate as the patient needs a balanced nutritional intake to provide for calories, proteins, vitamins, minerals, and carbohydrates. The fluid intake will not be restricted as there is no physiological reason unless the patient has a co-morbidity.

Standard Text: Select all that apply. 1. The blood pressure is 182/86. 2. The patient states, "Sometimes, I feel so nauseated after I smoke cigarettes." 3. The patient states, "I have never been able to stop smoking for more than two weeks at a time because the cravings get so bad." 4. An apical heart rate is 72 beats per minute. 5. A respiratory rate is 14 breaths per minute.

Correct Answer: 1, 2, 3 Global Rationale: Nicotine use results the release of norepinephrine and epinephrine which produces vasoconstriction. Vasoconstriction will increase the patient's blood pressure. Patients who use nicotine will find that it promotes vomiting. Quitting smoking is thought to be more difficult because of dopamine release, which reinforces the craving for more. Due to the effects of the norepinephrine and epinephrine the patient's heart rate and respiratory rate would both be higher than the stated levels.

The nurse manager is concerned that one staff nurse is demonstrating signs of substance abuse. Which behaviors did the manager observe in the staff nurse? Standard Text: Select all that apply. 1. calling for days off or illness before scheduled days off 2. using the bathroom frequently 3. excessive use of mouthwash 4. volunteering to transfer a patient to the intensive care unit 5. following up with nursing assistants on patient care needs

Correct Answer: 1, 2, 3 Global Rationale: Nurses experience many pressures in the workplace and have easy access to drugs, a temptation that may result in greater vulnerability for substance abuse and dependence. There are many observable warning signs of potential abuse, including calling for days off before scheduled days off, frequently using the bathroom, and excessive use of mouthwash. Volunteering to transfer a patient to the intensive care unit or following up with nursing assistants on patient care needs are not indications of substance abuse.

The patient has been included in a research study along with 4,205 other patients who live in the United States. Two thousand two hundred of these patients are male. Of the female patients, how many are likely to be smokers? Standard Text: Round the answer to the nearest whole number.

Correct Answer: 431 Rationale: An estimated 21.5% women in the United States are current smokers. If there are 4,205 patients included in this study and 2,200 patients are male, then 2,005 patients are female; 21.5% of 2,005 is 431.

The nurse is beginning the assessment of a patient with substance abuse. What should the nurse ask that demonstrates an open-ended question? Standard Text: Select all that apply. 1. On average, how many days a week do you drink or use drugs? 2. How often and how much do you usually use? 3. What is the greatest number of drinks you have had at any one time in the past month? 4. Do you drink beer or whiskey? 5. Did you drink beer before coming to the hospital?

Correct Answer: 1, 2, 3 Global Rationale: Options 1, 2, and 3 are open-ended questions that will allow the patient to discuss his or her use of drugs/alcohol. Since they are open-ended, the patient will need to answer more than "Yes" or "No." Options 4 and 5 are closed questions and require that the patient only answer "Yes" or "No."

The patient is exhibiting some addictive behaviors and has admitted to using illegal drugs. Which patient statements are consistent with addictive behaviors? Standard Text: Select all that apply. 1. "I think even as a child I was much more anxious than my friends." 2. "When I was in the hospital for appendicitis, they told me they had to give me more pain medications than normal because I was still in pain." 3. "Sometimes I steal things from stores just to see if I can get away with it." 4. "I like to play it safe. When my friends were bungee jumping off the bridge, I just watched." 5. "I have always been very slow to anger."

Correct Answer: 1, 2, 3 Global Rationale: People who are displaying addictive behaviors associated with substance use are more likely to be anxious, have a low tolerance for pain, participate in risky behaviors such as stealing and bungee jumping without regard for social norms or their own safety, and are more likely to become angry than others who are not using substances.

that the patient is using is dose-related? Standard Text: Select all that apply. 1. fetal CNS disturbances 2. tachycardia 3. asthma with long-term use 4. diuresis 5. hypertension

Correct Answer: 1, 2, 3 Global Rationale: The use of cannabis during pregnancy can cause fetal CNS changes because it crosses the placental barrier; this interferes with breastfeeding. It also causes tachycardia and with long term use; can cause asthma, bronchitis; and can increase the risk of respiratory cancer. Diuresis is not caused by cannabis, but is caused by the use of caffeine. Hypertension is found in cocaine users.

The nurse is determining a patient's degree of dependence on a substance. Which screening tools should the nurse use to help with this assessment? Standard Text: Select all that apply. 1. Michigan Alcohol Screening Test 2. CAGE questionnaire 3. Brief Drug Abuse Screening Test 4. Clinical Institute Withdrawal Assessment of Alcohol-Revised 5. Clinical Opiate Withdrawal Scale

Correct Answer: 1, 2, 3 Global Rationale: These screening tools provide a nonjudgmental, brief, and easy method to determine patterns of substance abuse behaviors. The Clinical Institute Withdrawal Assessment of Alcohol, Revised and the Clinical Opiate Withdrawal Scale are assessment tools for withdrawal from alcohol and drugs and can indicate the need for pharmacologic treatment to manage withdrawal.

The nurse suspects that a patient has a substance dependency. What observations did the nurse use to come to this conclusion? Standard Text: Select all that apply. 1. presence of tolerance 2. substance taken longer than intended 3. spends more time in private using the substance 4. desire to control substance use 5. discontinues use while experiencing intrapersonal problems

Correct Answer: 1, 2, 3, 4 Global Rationale: Substance dependence is demonstrated by tolerance to the drug, using the drug longer than intended, and spending more time using the substance in private. The dependent patient will also spend more time getting, taking, and recovering from use; there is an unsuccessful persistent desire to cut down or control the substance, there is more withdrawal from family and friends, and there is continued use despite knowledge of adverse effects. Substance abuse manifestations include continued use despite intrapersonal problems, failure to fulfill major roles, and involvement in physically hazardous situations while impaired.

A patient admitted to the hospital after a motor vehicle accident frequently smokes ice and had smoked some as recently as two hours prior to the accident. Which assessment findings are consistent with this information? Standard Text: Select all that apply. 1. Apical heart rate is 112 beats per minute. 2. Preventricular contractions are noted during electrocardiogram. 3. The patient weighs 92 pounds and is 5'5" tall. 4. The patient is complaining of chest pain. 5. Blood pressure is 96/72.

Correct Answer: 1, 2, 3, 4 Global Rationale: The patient will likely exhibit tachycardia and dysrhythmias. The patient's appetite has been suppressed by the methamphetamine use and the patient will likely be thin. Angina is a common complaint among people who use methamphetamines. The patient's blood pressure is likely to be elevated due to the vasoconstriction that is produced by this type of drug use.

Standard Text: Select all that apply. 1. frequently late to work 2. incomplete charting 3. errors in patient care judgment 4. erratic behavior 5. mood swings

Correct Answer: 1, 2, 3, 4 Global Rationale: Warning signs of role strain that could indicate substance abuse include frequent tardiness or absenteeism, especially before and after scheduled days off, haphazard, shoddy charting, patient care judgment errors, and unorganized, erratic behavior. Mood swings are a characteristic of depression.

The nurse is concerned that a patient being seen in the mental health clinic for psychosis is at risk for substance abuse. What assessment findings support this nurse's concern? Standard Text: Select all that apply. 1. age 19 2. male gender 3. younger siblings in high school 4. mother in rehabilitation for heroin 5. recently terminated from employer

Correct Answer: 1, 2, 4, 5 Global Rationale: Patients with co-occurring disorders are more likely to be unemployed younger males living in unstable conditions with more than one psychiatric diagnosis and a personality disorder. There is no evidence to support having younger siblings in high school increases the risk for substance abuse.

The nurse decides to include the CAGE questionnaire and B-DAST screening tool when assessing an adolescent admitted with panic and agitation. For which substances is this nurse most likely planning to assess in this patient? Standard Text: Select all that apply. 1. PCP 2. LSD 3. crack 4. heroin 5. alcohol

Correct Answer: 1, 2, 5 Global Rationale: A manifestation of PCP overdose is agitation. A manifestation of LSD overdose is panic. The patient is not demonstrating evidence of crack or heroin overdose. Alcohol is the most commonly used and abused legal substance in the United States.

A loading dose of magnesium sulfate 4g is ordered for a patient. The concentration available is 4g/250 mL to be given over 30 minutes. How many mL/hr should the nurse set the patient's infusion pump to deliver the medication?

Correct Answer: 500 mL/hr Rationale : 4g/30 min = X mL/h. 250mL/30 min = X mL/h. 250 mL/30 min X 2/2 = 500 mL/60 min = 500 mL/h.

A nurse is concerned that a newly hired colleague is exhibiting signs of a substance abuse problem. What did the nurse observe to come to this conclusion? Standard Text: Select all that apply. 1. The new nurse is always sucking on mints and has applied strong-smelling cologne. 2. The new nurse completes documentation and is prepared to give report at the end of the shift. 3. The new nurse has volunteered to give medications to patients. 4. The new nurse has displayed difficulty juggling his patient assignment and seems to be highly unorganized. 5. The narcotic count at the end of the shift is off; two doses of morphine sulfate are missing.

Correct Answer: 1, 3, 4, 5 Global Rationale: Eating an excessive number of mints and wearing strong-smelling cologne can be used to mask odors of alcohol on the nurse's breath. Offering to give medications to patients is consistent with a substance use problem. Unorganized thinking and erratic behavior are consistent with a substance use problem. The narcotic count is off while this nurse is working; this is consistent with a nurse who uses narcotics from the hospital supply. Completing documentation and being prepared to give report at the end of the shift does not indicate a substance abuse problem.

A patient admitted to an addiction detoxification unit has been prescribed medication to help with withdrawal symptoms. After reviewing the medication with the patient, the nurse realizes that further education is required when the patient makes which statements? Standard Text: Select all that apply. 1. "Naltrexone is an antidepressant." 2. "The Antabuse will help me with my cravings for heroin." 3. "The clordiazepoxide is also called Librium and it can help with my anxiety." 4. "The phenobarbital will help prevent me from having another seizure." 5. "I need folic acid and other vitamin supplements because I haven't eaten well for so long."

Correct Answer: 1,2 Global Rationale: Naltrexone (Vivitrol) helps diminish cravings for alcohol and opiates. It is not an antidepressant. Disulfiram (Antabuse) is given to patients to stop the breakdown of alcohol within the body and make the consequences of drinking alcohol more severe. Methadone will help block heroin cravings. Chlordiazepoxide (Librium) can be used to help with anxiety and prevent seizure activity. Phenobarbital can help prevent seizure activity. Vitamin supplements can help the patient with alcoholism because patients with alcoholism are more likely to have developed vitamin deficiencies.

The patient has been diagnosed with substance dependence. The patient has been using Bennies for the last three years. Which patient statements should the nurse attribute to substance dependence? Standard Text: Select all that apply. 1. "I am so tired and I feel so down about everything that is happening around me." 2. "I really tried to cut down my use, but I fail miserably every time." 3. "The only thing I care about right now is getting my fix." 4. "I have to use a lot more right now to get the same high as I did before." 5. "I have a great job where I work fulltime as a mechanical engineer, so that part of my life is very fulfilling."

Correct Answer: 1,2,3,4 Global Rationale: "Bennies" is a term used to describe amphetamines. Substance dependence can be associated with withdrawal symptoms from the substance. Fatigue and depression are withdrawal symptoms associated with the use of amphetamines. When patients are unsuccessfully attempting to cut down on their use of the substance, dependence can often be the cause. Fixation on obtaining more of the substance is characteristic of dependency. Patients are more likely to develop tolerance to the drug and require greater quantities to get the same result. It is unlikely that a patient with substance dependence is continuing to keep a fulltime job because the patient will usually spend a lot of time procuring and using the drug and invest less time with occupational activities.

A patient has overdosed on benzodiazepines and is admitted in a comatose state. What is the best order for treatment? Standard Text: Click and drag the options below to move them up or down. Choice 1. Clear airway Choice 2. Prepare to intubate Choice 3. Prepare for IV fluids Choice 4. Prepare for gastric lavage Choice 5. Institute seizure precautions Choice 6. Prepare for possible dialysis

Correct Answer: 1,2,3,4,5,6 Global Rationale: With the overdosed and comatose patient, the airway is the first consideration (follows the ABCs). Next, the nurse needs to prepare for intubation of the patient as this is the second priority. The nurse will need to prepare for IV fluids. Gastric lavage may be done to try and remove any remaining drug. Seizure precautions are instituted as the patient may experience seizure activity. The patient may need dialysis to remove the drug that has reached the blood.

A patient withdrawing from alcohol has an order for diazepam (Valium), 10 mg every 4 hours for 4 doses, then 5 mg every 4 hours for 4 doses. The drug comes in a concentration of 5 mg/mL. The total mL that the patient will receive would be _____. Standard Text: Fill in the blank with a numeric answer.

Correct Answer: 12 mL Rationale : The drug comes in 5 mg/mL, and at 10 mg ordered, each dose is 2 mL. 2 mL × 4 doses = 8 mL. The four doses of 5 mg = 4 mL. 4 + 8 = 12.

During an assessment a patient with multiple substance addictions asks why the need for drugs and alcohol is so great. What should the nurse include when responding to this patient? Standard Text: Select all that apply. 1. "Substance abuse is sign of weakness and boredom with life." 2. "Alcohol reinforces the transmission of opioids in the system." 3. "There is a human tendency to seek pleasure and avoid stress and pain." 4. "It really depends upon genetic makeup and if your parents used drugs." 5. "One substance in the brain, dopamine, is responsible for drug-seeking behavior."

Correct Answer: 2, 3, 5 Global Rationale: The human tendency to seek pleasure and avoid stress and pain is partially responsible for substance abuse. Although far from definite, evidence implicates the endogenous opioid system in the development and maintenance of addictive behaviors. Current data suggest that alcohol increases endogenous opioid neurotransmission and that this activation is partly responsible for its reinforcing effect. Dopamine has been identified as the primary neurotransmitter responsible for sustaining the addictive quality of drugs and for increasing drug-seeking behavior. The reinforcing properties of drugs can create a pleasurable experience and reduce the intensity of unpleasant experiences. There is no evidence to support that substance abuse is a sign of weakness and boredom with life. Genetic makeup and biological factors do contribute to substance abuse behaviors; however, there are other explanations.

Education for a pt taking MAOIs should include which of the following? a. fluid and sodium replacement when appropriate, frequent drug blood levels, signs and symptoms of toxicity b. life time of continuous use, possible tardive dyskinesia, advantages of an injection every 2-4 weeks c. short term use, possible tolerance to beneficial effects, careful tapering of the drug at the end of treatment d. tyramine restricted diet, prohibitive use of OTC meds w/o telling the dr

D

John is a pt at the mental health clinic. He has been experiencing suicidal idealizations and has been seeing a psych nurse every 3 days. He has been taking 100mg of sertraline daily for about a month receiving small amounts of the med from the nurse at each visit . Today he comes to the clinic in a cheerful mood, much different from how he seemed just 3 days ago. How might the nurse assess this behavioral change? a. sertraline is starting to take an effect b. he is no longer in need of antidepressant meds c. he has completed the grief response over his lost wife d. he made have decided to carry out with his suicide plan

D

The nurse cares for a patient diagnosed with depression and encourages the patient to join an activity. Which of the following approaches by the nurse is BEST? A. Offer several appealing choices to the patient. B. Tell the patient it is part of the physician's orders. C. Describe the activity in detail to the patient. D. Invite the patient to join in.

D. Invite the patient to join in. Good example of how a nurse might lead a patient into an activity by telling the client that you would like him/her to join in; it's important to demonstrate caring and acceptance.

A graduate nurse fails an examination and accuses the psychiatric instructor of being an unfit teacher and causing the failure. The nurse identifies this as an example of which behavior? A. Conversion B. Acting out C. Compensation D. Projection

D. Projection Unable to accept the sense of failure and resulting poor self-esteem, the student projects the failure onto the instructor, thereby saving face but coping ineffectively.

A patient diagnosed with bipolar depression is hospitalized in the elation phase of the illness. The patient says to the nurse, "I just bought myself a home computer and a large screen TV for the family." Which of these interpretations by the nurse is MOST accurate? A. The patient wants to impress the nurse with his generosity towards the family. B. The patient is insecure about his self-worth and needs to manipulate electronic devices. C. The patient has completely lost contact with reality and his thought patterns are disturbed. D. The patient has a mood disturbance and his judgment is poor at this time.

D. The patient has a mood disturbance and his judgment is poor at this time. Person in a manic state may have delusions of grandeur and/or an exalted opinion of himself and his abilities.

One morning the nurse finds the client crying and approaches. The client says, "What do you want? Go away, you can't help me. I hate you and I hate myself." Which response by the nurse is best? A. Why is it that you dislike me and yourself? B. I'll come back later when you feel in a better mood. C. It's difficult for me to communicate with you when you talk this way. D. You seem to be in pain, I'll stay with you for a while.

D. You seem to be in pain, I'll stay with you for a while. Conveys support and understanding; client's statement reflects the low self-esteem common in the depressed person; depressed clients are frequently angry, but displace the anger inwardly, and suffer shame and guilt; basic interpersonal task of the nurse is to gradually help lessen the client's feelings of worthlessness.

Crisis

Dangerous point in disease process or time frame in life. stages: denial, increased tension/anxiety, disorganization, attempt to escape or pretend it doesn't exist, blame others, general reorganization. nursing: focusing on client's immediate problems, exploring client's understanding of problem, help client become aware of feelings and validating them.

Neurocognitive Disorder Delirium and Dementia

Delirium: Acute impairment of cognitive functioning that is secondary to another ailment - Infection, Drug Reaction, Substance intoxication/withdrawal, Electrolyte Imbalance, Head Trauma, Sleep Deprivation - Correct the cause of delirium Dementia: Chronic, gradual, progressive, and irreversible degeneration of judgement, memory, abstract thinking, social behavior, and other aspects of cognition - Can develop aphasia, apraxia, and agnosia Assessment: - Limited attention span, easily distracted - Disorientation - Delusions, illusions, or hallucinations - Loss of recent or remote memory - Confabulation - Impaired coordination - Day-night reversal/sleep deprivation - Incontinence, constipation Interventions: - Provide safe, consistent environment - Maintain health, nutrition, hygiene, and rest - Assist with ADLs - Reorient client as needed - Provide a consistent caregiver

Substance- Medication Induced Depression

Depressed mood and physiological affects due to direct use of a substance, drug of abuse/ medication/ toxin exposure

Depressive Disorder due to another medical condition

Depressed mood as a physiological response to a medical condition Chronic disease?

A client states, You wont believe what my husband said to me during visiting hours. He has no right treating me that way. Which nursing response would best assess the situation that occurred?

Describe what happened during your time with your husband.

Attention-Deficit (Hyperactivity) Disorder

Description: Developmentally inappropriate attention, impulsiveness, and hyperactivity Assessment: - More prevalent in boys - Failure to listen/follow instructions - Difficulty playing quietly or sitting still - Disruptive behavior - Easily distracted by external stimuli - Excessive talking - Shifting from task to task without finishing them - Underachievement in school Interventions: - Decrease environmental stimuli - Set limits on behavior - Initiate a behavior contract to help the child manage own behavior - Administer medications as prescribed

Bulimia Nervosa

Eating excessive amounts of food followed by self-induced purging (vomiting, laxatives, diuretics, fasting, or exercise) Report loss of control over eating during the binging Assessment: - Diarrhea, constipation, abdominal pain, and bloating - Dental damage due to vomiting - Sore throat and chronic inflammation of the esophageal lining with possible ulceration - Financial stressors related to food budget - Concerns with body shape and weight; usually not underweight Interventions: - Monitor weight, vital signs, and electrolytes - Provide structures environment around mealtime - Encourage expression of anger - Use positive reinforcement to stop vomiting and laxative use - Administer antidepressants as prescribed - Family therapy is most effective

Electroconvulsive Therapy (ECT)

Electrically included seizures to treat severe depression. anesthetic for sedation and muscle relaxant given. side effects include confusion, short term memory loss. nursing: inform client about potential temp. memory loss, confusion. keep NPO after midnight for early morning procedure. have client void, remove dentures, glasses, jewelry, take vitals, stay with client, orient client

Schizophrenia Catatonic vs. schizoaffective Assessment and Interventions

Description: Psychiatric disorder characterized by thought disturbance, altered affect, withdrawal from reality, regressive behavior, difficulty with communication, and impaired interpersonal relationships as well as impaired ability to perceive reality. Types: Catatonic and Schizoaffective Catatonic: - Stupor (decrease in reaction to the environment)/mutism - Rigidity (maintenance of a posture against efforts to be moved - Posturing (waxy flexibility - Negativism (resistance to instructions) - Potential of violence to self or others during stupor or excitement Schizoaffective: - Schizophrenia with the presence of a major mood episode for the majority of the disorder's duration Assessment: - Symbolism: meaning given to words by client to screen thoughts and feelings that would be difficult to handle directly - Delusions: Fixed, false beliefs that may be persecutory, grandiose, religious, or somatic (cannot be change with reason or reality) - Ideas of Reference: belief that all conversation or actions are in reference to the client (world revolves around them) - Looseness of Association: Lack of clear connection from one thought to the next - Tangential or circumstantial speech: failing to address the original point (too many nonessential details - Echolalia: constantly repeating what is heard - Neologism: Creating new words - Preservation: repeating same word or phrase in response to a different questions - Word Salad: Speaking with a jumbled mixture of real and made-up words - Blocking: gap or interruption in speech due to absent thoughts - Concrete thinking: thinking based on facts rather than abstract points (takes everything literally) - Illusions: Misinterpretation of external environment (can be fixed with reality) - Hallucinations: false sensory perception - Depersonalization: perceives self as alienated or detached from real body - Echopraxia: repeating movements of others - Difficult to establish trust or intimacy Interventions: - Establish trust and provide a safe and secure environment - Use a matter-of-fact approach - Assist with ADLs - Use concrete terms - Use clarification: accept and support client feelings - Set limits - Avoid stresors - Praise socially acceptable behavior 4 A's of Schizophrenia: - Autism (preoccupied with self) - Affect (flat) - Associations (Loose) - Ambivalence (difficulty making decisions)

Reaction Formation

Development of conscious attitudes and behaviors that are the opposite of what is really felt.

Group Intervention

Development of interactive relationships with patients that have at least one common goal Can be open or closed, small or large Phases of a group: - Orientation: high anxiety, superficial interaction, limit testing of therapist - Working: problem identification, beginning of problem solving, sense of group belonging - Termination: evaluation of experience, expression of feelings (could be anger or joy) Advantages: - Development of socializing techniques - Clients don't feel alone in their situation - Opportunity to try new behaviors, gain feedback from peers, and look at alternative perspectives of dealing with issues

Stimulants

Dextroamphetamine (Dexedrine) Methylphenidate (Ritalin) Pemoline (Cylert) Lisdexamfetamine (Vyvanse) Amphetamine/Dextroamphetamine (Adderall) Dexmethylphenidate (Focalin)

Which client data indicate that a suicidal client is participating in a plan for safety?

Disclosing a plan for suicide to staff A degree of the responsibility for the suicidal clients safety is given to the client. When a client shares with staff a plan for suicide, the client is participating in a plan for safety by communicating thoughts of self-harm that would initiate interventions to prevent suicide.

Post-Traumatic Stress Disorder (PTSD)

Disorder characterized by re-experiencing o the traumatic event through flashbacks or nightmares, hyper vigilance to one's surroundings, and avoidance of situations related to the stressful event

Anorexia Nervosa

Distorted body image and intense fear of becoming obese drive excessive dieting and exercise. 15-20% die More common in females and adolescents/young adults Results from dependency issues with parents (feel that their body is the only thing they can control) Causes: - Dysfunctional family system - Unrealistic expectations of perfection - Ambivalence about maturation and the assumption of independence Assessment: - Loss of at least 15% of ideal or original body weight - Excessive exercise - Distorted body image (thinks they are fat even when emaciated) - Dry skin/loss of hair - Delayed sexual development or disinterest in sex - Dehydration and electrolyte imbalance (due to diet pill abuse, enema or laxative abuse, diuretic abuse, or self-induced vomiting) Intervention: - Monitor weight, vital signs, and electrolytes - Provide a structured, supportive environment especially during mealtime - Set a time limit for eating - Monitor food and fluid intake - Prevent excessive exercise - Provide snacks between meals - Devise a behavior-modification program if indicated (Include weight goal and weigh on regular schedule, Weigh in same clothes with back to scale, Praise weight gain rather than food intake) - Sudden withdrawal from medications can cause seizures - Gain pleasure in providing food and watching others eat (do not let them plan or prepare foods)

Electroconvulsive Therapy (ECT)

Electronically induced seizures for clients who do not respond to antidepressants or need immediate intervention (such as for suicide patients). Teach about treatment; avoid the word shock Anticholinergic (atropine): dries oral secretions Quick-acting muscle relaxant (Succinylcholine): reduces bone and muscle trauma Emergency cart, suction, and O2 should be available

Elder Abuse

Emotional, physical, and sexual abuse, financial exploitation, or neglect of older adults

To promote self-reliance, how should a psychiatric nurse best conduct medication administration?

Encourage clients to request their medications at the appropriate times.

Therapeutic Communication Nursing Judgements

Encourage patient. Help to understand condition. Set goals. Patient willing to participate in health care. Patient informed. Make referrals. Broad openings. Avoid false assurance. Touch for elderly patients. Answer any questions, encourage goal setting. Encourage patient to be honest and converse about topics that are not health related.

An angry client on an inpatient unit approaches a nurse, stating, "Someone took my lunch! People need to respect others, and you need to do something about this now!" The nurse's response should be guided by which basic assumption of milieu therapy?

Every interaction is an opportunity for therapeutic intervention.

Claustrophobia

Fear of closed-in places

Agoraphobia

Fear of crowds or open places

Thanatophobia

Fear of death

Acrophobia

Fear of heights

Nyctophobia

Fear of the dark

Hydrophobia

Fear of water

Premenstrual Dysphoric Depression

Feeling of sadness/depression/mood swings/anxiety that onset a week before menstruation and cease during/after menstruation

Predisposing Factors: Gender Age Socioeconomic Group Marital Status Seasonality?

Females Low Socioeconomic groups Singles, around the time to be married Winter months

Nonphenothiazines

Haloperidol (Haldol) Thiothixene (Navane) Pimozide (Orap)

Obsessive Compulsive Disorder (OCD)

Have extreme need to control and predict outcomes. Making decisions and changes is anxiety-producing. Repetitive, uncontrollable thoughts (obsessions) and actions (compulsions)

Suicidal Behavior

Highest priority is protecting pts from inflicting harm on themselves. Clues for impending suicide include: sudden change in pt behavior, becoming energetic after period of severe depression, improved mood 10-14 days after taking antidepressant. finalizing business or personal affairs, giving away valuable posession or pets, SI. Nurse should monitor for signs of suicidal behavior, remove all potentially dangerous items, one-to-one observation of the patient, evaluate pt resources, convey sincere desire to help and treat pt in dignified manner.

10. After years of dialysis, an 84-year-old states, Im exhausted, depressed, and done with these attempts to keep me alive. Which question should the nurse ask the spouse when preparing a discharge plan of care?

How often is your spouse left alone? This client has many risk factors for suicide. The client should have increased supervision to decrease likelihood of self-harm.

Why are we concerned with MAOIs...what do they have the potential to cause

Hypertensive crisis, serotonin syndrome

Which nursing statement is a good example of the therapeutic communication technique of giving recognition?

I notice you are wearing a new dress and you have washed your hair.

An instructor is correcting a nursing students clinical worksheet. Which instructor statement is the best example of effective feedback?

I noticed that you used the clients name in your written process recording. That is a breach of confidentiality.

After fasting from 10 p.m. the previous evening, a client finds out that the blood test has been canceled. The client swears at the nurse and states, You are incompetent! Which is the nurses best response?

I see that you are upset, but I feel uncomfortable when you swear at me.

A client with a history of three suicide attempts has been taking fluoxetine Prozac for 1 month. The client suddenly presents with a bright affect, rates mood at 9/10, and is much more communicative. Which action should be the nurses priority at this time?

Increase frequency of client observation. The nurse should be aware that a sudden increase in mood rating and change in affect could indicate that the client is at risk for suicide and client observation should be more frequent.

What is the black box warning on antidepressants for?

Increased suicide risk

Child Abuse

Indications include inconsistency of type/location of injury with the history of incidents, severe CNS or abdominal injuries, obvious disturbance in parent-child interaction, sexual abuse (genital lacerations, STD's) emotional neglect, failure to thrive. Nursing care includes providing for physical needs first, mandatory reporting to supervisor or appropriate agency, nonjudgmental tx of parents, encouraging expression of feelings, teaching growth and development concepts especially safety, discipline, age appropriate activities, and nutrition. Provide emotional support for child, play therapy. Initiate protective placement and or appropriate referrals for long term follow up.

Passive-aggression

Indirectly expressing aggression toward others; a facade of overt compliance masks covert resentment

MAOIs work by..

Inhibit monoamine oxidase, an enzyme known to inactivate norepinephrine, serotonin, and dopamine

MAOIs

Isocarboxazid (Marplan) Phenelzine (Nardil) Selegiline (Eldepryl) Tranylcypromine (Parnate)

3. A client diagnosed with neurocognitive disorder due to Alzheimers disease can no longer ambulate, does not recognize family members, and communicates with agitated behaviors and incoherent verbalizations. The nurse recognizes these symptoms as indicative of which stage of the illness?

Late stage The nurse should recognize that this client is in the late stage of Alzheimers disease. The late stage is characterized by a severe cognitive decline

22. Which statement indicates that the nurse is acting as an advocate for a client who was hospitalized after a suicide attempt and is now nearing discharge?

Lets confer with the treatment team about the resources that you may need after discharge.The nurse is functioning in an advocacy role when collaborating with the client and treatment team to discuss client problems and needs

A client diagnosed with dependent personality disorder states, Do you think I should move from my parents house and get a job? Which nursing response is most appropriate?

Lets discuss and explore all of your options.

A newly admitted client diagnosed with obsessive-compulsive disorder (OCD) washes hands continually. This behavior prevents unit activity attendance. Which nursing statement best addresses this situation?

Lets figure out a way for you to attend unit activities and still wash your hands.

Therapeutic Communication Nursing

Listening to and understanding the client while promoting clarification and insight; the goal is to understand the client's message, facilitate client's verbalization of feelings, communicate bur's understanding and acceptance, identify problems, goals, and objectives; includes using silence. General leads, broad openings, clarification, reflection.

Mood Stabilizing Drugs

Lithium Anticonvulsant Mood Stabilizers: - Valproic Acid (depakote) - Carbamazepine (Tegritol) - Lamotrigene (Lamictal)

Tranylcypromine

MAO inhibitor used for atypical depression. Eating certain cheeses or wines can predispose to serotonin syndrome.

Depression is

MDD= 2 weeks or longer. Abnormal feelings of sadness, low self-esteem, helplessness, hopelessness, obsessive thoughts, fears, sense of doom or failure, somatic complaints: GI, appetite, pain, irritability, sleep disturbances, lack of energy, sex drive, dizziness.

Nursing Care After ECT

Maintain patent airway: nausea and vomiting after is common Check vital signs every 15 minutes until consciousness is regained Reorient client Common complaints: headache, nausea, muscle soreness, retrograde amnesia

Which therapeutic communication technique is being used in this nurseclient interaction? Client: When I get angry, I get into a fistfight with my wife or I take it out on the kids. Nurse: I notice that you are smiling as you talk about this physical violence.

Making observations

A clients younger daughter is ignoring curfew. The client states, Im afraid she will get pregnant. The nurse responds, Hang in there. Dont you think she has a lot to learn about life? This is an example of which communication block?

Making stereotyped comments

N-Methyl D-Aspartate Antagonist

Memantine (Nemenda)

Bipolar Disorder

Mood disorder. manic episodes usually begin suddenly, with rapid escalation. Indications: elevated or expansive mood, agitation, accelerated speech, thought and movement, distractibility, self-confidence, aggression, sarcasm, inappropriate dress, inattention to hygiene, anorexia, weight loss, constipation, insomnia.

5. A client is experiencing progressive changes in memory that have interfered with personal, social, and occupational functioning. The client exhibits poor judgment and has a short attention span. A nurse should recognize these as classic signs of which condition?

Neurocognitive disorder The nurse should recognize that the client is exhibiting signs of neurocognitive disorder (NCD). In NCD, impairment is evident in abstract thinking, judgment, and impulse control. Behavior may be uninhibited and inappropriate.

10. After 1 week of continuous mental confusion, an elderly African American client is admitted with a preliminary diagnosis of major neurocognitive disorder due to Alzheimers disease. What should cause the nurse to question this diagnosis?

Neurocognitive disorder does not develop suddenly. The nurse should know that neurocognitive disorder (NCD) does not develop suddenly and should question this diagnosis. The onset of NCD symptoms is slow and insidious and is unrelated to race, culture, or creed. The disease is generally progressive and debilitating

16. Which symptom should a nurse identify that would differentiate clients diagnosed with neurocognitive disorders from clients diagnosed with amnesic disorders?

Neurocognitive disorders involve impairment of abstract thinking and judgment. Amnestic disorders are characterized by an inability to learn new information and to recall previously learned information, with no impairment in higher cortical functioning or personality change.

Failure to warn

Newer area of potential liability for nurse managers that involves the responsibility to warn subsequent or potential employers of nurses' incompetence or impairment.

A nurse maintains an uncrossed arm and leg posture. This nonverbal behavior is reflective of which letter of the SOLER acronym for active listening?

O

Post-Traumatic Stress Disorder

Occurs in clients who have experienced or witnessed a highly traumatic event or repeatedly exposed to stories of traumatic events. Assessment: (Four Symptom Clusters) 1. Avoidance of events or situations that are reminders 2. Persistent negative alterations in cognition and mood 3. Mood including numbing symptoms, as well as persistent negative emotional stress 4. Alterations in arousal and reactivity including irritable or aggressive behavior and reckless or self-destructive behavior (suicidal ideation and substance abuse) Interventions: - Provide a consistent, non-threatening environment - Assess for self-harm and provide precautions - Help the client regain a sense of control by identifying past situations where they successfully handled something - Administer anti-anxiety and anti-psychotic medications as prescribed - Promote rest - Utilize group therapy with clients who went through a similar event

Rationalization

Offering an acceptable, logical explanation to make unacceptable feelings and behavior acceptable

Autism

One of five disorders along the autism spectrum it is characterized by extreme social withdrawal and impairments in communication. Individuals with autism often have stereotypical behaviors, over reaction to environmental or sensory stimuli and any changes to their routine. It is usually manifested by age three and can vary widely in its level of severity. Individuals with autism often (but not always) have impairments in cognitive ability, language and social skills.

Commonly abused drugs

Opiates: - Heroin, morphine, codeine, opium, methadone - Cocaine - Amphetamines - Hallucinogenics Antianxiey Drugs

Personality Disorders

PDs are maladaptive responses to anxiety that cause difficulty in relating to and working with others These people are comfortable with their disorders and believe that they are right and the world is wrong (little motivation to change)

Depressive Disorders (Severity)

Pathologic grief reactions ranging from mild to severe states. Mild: - Feelings of sadness - Difficulty concentrating and performing usual activities - Difficulty maintaining usual level of activity Moderate: - Feelings of helplessness and powerlessness - Anergia - Sleep and appetite disturbances - Slowed speech, thought, and movement - Rumination on negative feelings Severe: - Feelings of hopelessness, worthlessness, and guilt - Flat affect - Indecisiveness - Anergia, lack of motivation - Suicidal thoughts - Delusions or hallucinations - Sleep and appetite disturbances - Loss of interest in sex - Constipation MOST IMPORTANT SYMPTOM: Loss of interest in the pleasures of life

Antisocial Personality Disorder

Pervasive pattern of disregard for and violation of the rights of others. lying, cheating, stealing, promiscuous behaviors, appears chambering and intellectual, smooth talking, unlawful, aggressive, reckless, lack of guilt and remorse, rationalizes/denies behavior, acts entitled. substance abuse and dependence problems.

Chlorpromazine

Phenothiazine antipsychotic drug prototype: blocks most dopamine receptors in the CNS. Tox: atropine−like, EPS dysfunction, hyperprolactinemia, postural hypotension, sedation, seizures (in overdose), additive effects with other CNS depressants

Antipsychotic Drugs

Phenothiazines Nonpenothiazines Long-acting drugs Atypical Antipsychotics

Maslow's Hierarchy of Basic Human Needs

Physical Needs - Oxygen, water, food, sleep, sex Safety Needs - physical, security, order Love and Belonging - Affection, companionship, identification Esteem and recognition- Status, success, prestige Self Actualization - self-fulfillment, creativity

Focus of sexual abuse questions

Physical manifestations of abuse Client Safety Legal Responsibilities of the Nurse: - Required to report child abuse - Adults decision to report otherwise

Milieu Therapy

Planned use of people, resources and activities in the environment to assist interpersonal skills, social functioning, and performance of ADLs while protecting safety of all clients. Uses limit setting Clients make decisions about their care and nurses support privacy and autonomy

Care for Suicidal Patients

Precautions: - Previous attempt is the highest risk factor - Giving away possessions - Sudden happiness Interventions: - Express concern for the client - Tell the client that you will share information with the staff - Offer hope (You're feeling this way not but there is treatment to help this feeling pass) - Stay with the client - Sitting in silence with the client is the best intervention to offer support

Behavioral Therapy - Positive Reinforcement - Negative Reinforcement

Process of changing ineffective behavior patterns. Focus on consequences rather than peer pressure. Uses role modeling and teaching new behaviors Positive Reinforcement - strengthens behavior with praise or reward Negative Reinforcement - eliminates inappropriate behavior with removing a privilege or ignoring undesirable behavior

Alzheimer's disease

Progressive degeneration of the brain that ultimately results in dementia.

Defense Mechanisms

Projection: person attributes to another feelings and or beavers and or impulses unacceptable to self. Conversion: anxiety is repressed and converted to physical symptom. Compensation: attempt to overcome real or imagined shortcomings.

4. A client is in the late stage of Alzheimers disease. To address the clients symptoms, which nursing intervention should take priority?

Promote dignity by providing comfort, safety, and self-care measures.

Therapeutic Communication

Purpose: develop trust. Promote open communication. four distinct phases: pre-interaction, orientation, working, termination. Discuss fears without being judgmental. Present warm, caring manner. Close curtain and sit down. Act with integrity. Recognize and respect patient's understanding of her body and condition. Express genuine concern that patient will receive the treatment she needs.

A student nurse tells the instructor, Im concerned that when a client asks me for advice I wont have a good solution. Which should be the nursing instructors best response?

Remember, clients, not nurses, are responsible for their own choices and decisions.

Cognitive Therapy

Replaces irrational beliefs and distorted attitudes. Short term. Goal is cognitive restructuring

Mild Anxiety

Required for daily functioning to motivate learning and promote sensory awareness and alertness. Allows for concentration and logical thought.

Which therapeutic communication technique is being used in this nurseclient interaction? Client: My father spanked me often. Nurse: Your father was a harsh disciplinarian.

Restatement

Regression

Reverting to an earlier level of development when anxious or highly stressed

15. A client diagnosed with neurocognitive disorder due to Alzheimers disease is disoriented and ataxic, and he wanders. Which is the priority nursing diagnosis?

Risk for injury The priority nursing diagnosis for this client is risk for injury. Both ataxia (muscular incoordination) and purposeless wandering place the client at an increased risk for injury.

Atypical Antipsychotics

Risperidone (Risperdal) Olanzapine (Zyprexa) Quetiapine (Seroquel) Aripiprazole (Abilify) Ziprasidone (Geodon) Clozapine (Clozaril)

9. A client diagnosed with neurocognitive disorder exhibits progressive memory loss, diminished cognitive functioning, and verbal aggression upon experiencing frustration. Which nursing intervention is most appropriate?

Schedule structured daily routines. The most appropriate nursing intervention for this client is to schedule structured daily routines. A structured routine will reduce frustration and thereby reduce verbal aggression

In the role of milieu manager, which activity should the nurse prioritize?

Searching newly admitted clients for hazardous objects

Isolation

Separation of an unacceptable feeling, idea, or impulse from one's thought process

14. An older client has recently moved to a nursing home. The client has trouble concentrating and socially isolates. A physician believes the client would benefit from medication therapy. Which medication should the nurse expect the physician to prescribe?

Sertraline (Zoloft) The nurse should expect the physician to prescribe sertraline (Zoloft) to improve the clients social functioning and concentration levels. Sertraline (Zoloft) is an SSRI (selective serotonin reuptake inhibitor) antidepressant. Depression is the most common mental illness in older adults and is often misdiagnosed as neurocognitive disorder.

When interviewing a client, which nonverbal behavior should a nurse employ?

Sitting squarely, facing the client

Substance Abuse: Lysergic Acid Diethylamide (LSD)

Stay with patient and gently talk him down, reassuring that response to LSD will pass. decrease environmental stimuli, medicate as ordered for extreme anxiety., apply physical restraints as ordered, frequently monitor vitals, provide extra blankets or hypothermia blanket if necessary

ADHD Medications

Stimulants

Severe Anxiety

Stimulate fight or flight. Causes perceptions to be distorted. Impairs concentration and problem solving ability. Results in verbalization of emotional pain. Tremors and increased motor activity

A nurse working on an inpatient psychiatric unit is assigned to conduct a 45-minute education group. What should the nurse identify as an appropriate group topic?

Stress management

A client diagnosed with schizophrenia functions well and is bright, spontaneous, and interactive during hospitalization but then decompensates after discharge. What does the milieu provide that may be missing in the home environment?

Structured programming

Sublimation

Substitution of an unacceptable feeling by a more socially acceptable one (a skinny kid that can't play football becomes a champion swimmer)

Delusional Disorders

Suspicious, strange behaviors which can be precipitated by a stressful event and manifest as intense hypochondriasis Assessment: - Assess degree of anxiety, insecurity, mistrust - Assess whether delusions are present (reference or control, persecution, grandeur, somatic, jealousy) Interventions for Delusional Client: - Divert focus from the delusion to reality - Do not agree with or support delusions - Avoid arguing about the delusion. Be matter-of-fact - Administer antipsychotic and antiparkinsonian medications - Avoid touching the client Interventions for Hallucinating Client: - Protect from injury and pay attention to content - May remarks to interrupt the hallucinations - Discuss observations with client such as looking around as if listening to something) - Administer anti-psychotic and anti-cholinergic drugs Know the side effects of drugs to determine if behavior changes are due to schizophrenia or side effects.

Action/Antidepressants

TCAs take 10 to 14 days for effects, patients may have enough physical and emotional energy to act upon suicidal thoughts.. don't stop abruptly. get up slowly. may take 4-6 weeks to work.

Acetylcholinesterase Inhibitors

Tacrine (Cognex) Donepezil (Aricept) Rivastigmine (Exelon) Galantamine (Razadyne)

1. A geriatric nurse is teaching student nurses about the risk factors for development of delirium in older adults. Which student statement indicates that learning has occurred?

Taking multiple medications may lead to adverse interactions or toxicity. The nurse should identify that taking multiple medications may lead to adverse reactions or toxicity and put an older adult at risk for the development of delirium. Symptoms of delirium include difficulty sustaining and shifting attention. The client with delirium is disoriented to time and place and may also have impaired memory

Depression

Tell client you would like them to join in. demonstrate caring and acceptance. Be alert for signs of self-destructive behavior, assist in physical needs, support self esteem, decrease social withdrawal.

Suppression

The INTENTIONAL exclusion of feelings and ideas (I'll think about it tomorrow)

Repression

The INVOLUNTARY exclusion of a painful thought or memory from awareness

2. A client diagnosed with vascular dementia is discharged to home under the care of his wife. Which information should cause the nurse to question the clients safety?

The client smokes one pack of cigarettes per day. Forgetfulness is an early symptom of dementia that would alert the nurse to question the clients safety at home if the client smokes cigarettes. Vascular dementia is a clinical syndrome of dementia due to significant cerebrovascular disease. The cause of vascular dementia is related to an interruption of blood flow to the brain. High blood pressure and hypertension are significant factors in the etiology.

23. A client is newly admitted to an inpatient psychiatric unit. Which of the following is most critical to assess when determining risk for suicide?

The clients history of suicide attempts A history of suicide attempts places a client at a higher risk for current suicide behaviors. Knowing this specific data will alert the nurse to the clients risk

A client has undergone psychological testing. With which member of the interdisciplinary team should a nurse collaborate to review these results?

The clinical psychologist

Family Therapy

The entire family is the client. Focuses on family interaction and roles of each member. Goal is to decrease conflict and anxiety and develop appropriate role relationships

A nurse recently admitted a client to an inpatient unit after a suicide attempt. A health-care provider orders amitriptyline Elavil for the client. Which intervention related to this medication should be initiated to maintain this clients safety upon discharge?

The health-care provider should provide a 1-week supply of Elavil with refills contingent on follow-up appointments as an appropriate intervention to maintain the clients safety. Tricyclic antidepressants have a narrow therapeutic range and can be used in overdose to commit suicide.

12. A nurse is caring for a client who has threatened to commit suicide by hanging. The client states, Im going to use a knotted shower curtain when no one is around. Which information would determine the nurses plan of care for this client?

The more specific the plan is, the more likely the client will attempt suicide

After assertiveness training, a formerly passive client appropriately confronts a peer in group therapy. The group leader states, Im so proud of you for being assertive. You are so good! Which communication technique has the leader employed?

The nontherapeutic technique of giving approval

A nurse states to a client, Things will look better tomorrow after a good nights sleep. This is an example of which communication technique?

The nontherapeutic technique of giving false reassurance

A client diagnosed with major depressive disorder with psychotic features hears voices commanding self-harm. The client refuses to commit to developing a plan for safety. What should be the nurses priority intervention at this time?

The nurses priority intervention when a client hears voices commanding self-harm is to place the client on one-to-one observation while continuing to monitor suicidal ideation.

During the planning of care for a suicidal client, which correctly written outcome should be a nurses first priority?

The nurses priority should be that the client will remain safe during the hospital stay. Client safety should always be the nurses priority.

A nurse discovers a clients suicide note that details the time, place, and means to commit suicide. What should be the priority nursing intervention and the rationale for this action?

The priority nursing action should be to place this client on one-to-one suicide precautions, because the more specific the plan, the more likely the client will attempt suicide. The appropriate nursing diagnosis for this client would be risk for suicide.

Displacement

The transference of feelings to another person or object

6. A nursing instructor is teaching about donepezil (Aricept). A student asks, How does this work? Will this cure Alzheimers disease (AD)? Which is the appropriate instructor reply?

This medication delays the destruction of acetylcholine, a chemical in the brain necessary for memory processes. Although most effective in the early stages, it serves to delay, but not stop, the progression of the AD.

The nurse says to a newly admitted client, Tell me more about what led up to your hospitalization. What is the purpose of this therapeutic communication technique?

To explore a subject, idea, experience, or relationship

What is the best rationale for including the client's family in therapy within the inpatient milieu?

To facilitate discharge from the hospital

What is the purpose of a nurse providing appropriate feedback?

To give the client critical information

A student nurse is learning about the appropriate use of touch when communicating with clients diagnosed with psychiatric disorders. Which statement by the instructor best provides information about this aspect of therapeutic communication?

Touch carries a different meaning for different individuals.

Atypical Antidepressants

Trazadone (Desyrel)

Imipramine

Tricyclic antidepressant prototype: blocks reuptake of norepinephrine and serotonin. Tox: ANS (alpha and muscarinic) blockade, cardiac arrhythmias

Anticholinergics

Trihexypgenidyl (Artane) Benztropine (Cogentin) Amantadine (Symmetrel)

T/F: women are at an increased risk for depression during the transition into menopause

True

Cluster B Personality Disorders

Types: Antisocial, Borderline, Histrionic, Narcissistic

Cluster C Personality Disorders

Types: Avoidant, Dependent, Obsessive-compulsive Assessment: - Assess degree of social impairment and anxiety - Determine risk of self-directed violence Intervention: - Establish Trust - Protect from injury - Set limits on manipulative behaviors - Reinforce independence - Encourage socialization PDs are maladaptive responses to anxiety that cause difficulty in relating to and working with others

Cluster A Personality Disorders

Types: Paranoid, Schizoid, and Schizotypal Description: Suspicious strange behavior that may be odd or eccentric. Assessment: - Determine degree of mistrust and anxiety - Determine if delusions are present (Reference or control, Persecution, Grandeur, or somatic) Interventions: - Establish trust - Avoid confrontation with client over delusions - Avoid talking or laughing where the client can see but not hear you. - Engage in noncompetitive activities

Fluphenazine

Typical High Potency Antipsychotic. good for positive symptoms of schizo.

Anxiety (defined)

Unpleasant feeling that occurs when a person feels a threat to their self. May be real or imagined (subjective experience). It is contagious can can be transferred from client to nurse and vice versa

Intellectualization

Using reason to avoid emotional conflicts

13. A client with a history of cerebrovascular accident (CVA) is brought to an emergency department experiencing memory problems, confusion, and disorientation. On the basis of this clients assessment data, which diagnosis would the nurse expect the physician to assign?

Vascular neurocognitive disorder The nurse should expect that this client would be diagnosed with vascular neurocognitive disorder (NCD), which is due to significant cerebrovascular disease. Vascular NCD often has an abrupt onset. This disease often occurs in a fluctuating pattern of progression.

Name some SNRIs

Venlafaxine (Effexor) Duloxetine (Cymbalta)

A client slammed a door on the unit several times. The nurse responds, You seem angry. The client states, Im not angry. What therapeutic communication technique has the nurse employed, and what defense mechanism is the client unconsciously demonstrating?

Verbalizing the implied and the defense mechanism of denial

Which example of a therapeutic communication technique would be effective in the planning phase of the nursing process?

We've discussed past coping skills. Lets see if these coping skills can be effective now.

A client diagnosed with post-traumatic stress disorder is admitted to an inpatient psychiatric unit for evaluation and medication stabilization. Which therapeutic communication technique used by the nurse is an example of a broad opening?

What would you like to talk about?

Which nursing statement is a good example of the therapeutic communication technique of offering self?

Would you like me to accompany you to your electroconvulsive therapy treatment?

The nurse is interviewing a newly admitted psychiatric client. Which nursing statement is an example of offering a general lead?

Yes, I see. Go on.

Can SAD occur during the summer?

Yes, called summer depression. Begins in late spring and early summer

A nurse is assessing a client diagnosed with schizophrenia for the presence of hallucinations. Which therapeutic communication technique used by the nurse is an example of making observations?

You appear to be talking to someone I do not see.

A client tells the nurse, I feel bad because my mother does not want me to return home after I leave the hospital. Which nursing response is therapeutic?

You feel that your mother does not want you to come back home?

Which nursing statement is a good example of the therapeutic communication technique of focusing?

You mentioned your relationship with your father. Lets discuss that further.

A client is trying to explore and solve a problem. Which nursing statement would be an example of verbalizing the implied?

You seem to be motivated to change your behavior.

A mother rescues two of her four children from a house fire. In an emergency department, she cries, I should have gone back in to get them. I should have died, not them. Which of the following responses by the nurse is an example of reflection?

Youre feeling guilty because you werent able to save your children.

A client on an inpatient psychiatric unit tells the nurse, I should have died, because I am totally worthless. In order to encourage the client to continue talking about feelings, which should be the nursing initial response?

Youve been feeling sad and alone for some time now?

Obsessive Compulsive Disorder (OCD) nursing interventions

accept ritualistic behaviors, structure environment, provide for physical needs, offer alternative activities especially ones using hands, guide decisions, minimize choices, encourage socialization, administer TCAs, SSRIs, teach thought stopping and response prevention techniques

Panic Attack

an episode of sudden onset of acute anxiety, occurring unpredictably, with feelings of acute apprehensiion, dyspnnea, dizzinesss, sweeating, aand/or chest pain, depresonizaation, paresthesia and fear of dying, loss of mind or control

Rape Trauma nursing

ask patient to describe what happened, provide emotional support, do not rush, assist physician with physical exam, help relieve the patients feeling of helpless and dependency, let victim assume as much control as possible, sign consent before photos or hair samples, prophylactic treatment for STIs/HIV.

anxiety nursing

assess level of anxiety, keep environmental stress/stimulation low when anxiety is high, assist client to cope with anxiety more effectively, maintain accepting helpful attitude. administer meds, advice patient not to stop abruptly, discuss feelings, relaxation techniques, time management skills, referral to mental health.

Crisis nursing

assess risk of suicide, administer med, assess patients perception of the problem and coping capabilities, provide support during expression of problems, assess availability to support mechanisms both emotional and financial. use of crisis services, refer patient to mental health specialist

Informed Consent/Psychiatry

before ECT need this and need consent before taking photos or hair samples of rape victim

Narcissistic Personality Disorder

characterized by inflated or grandiose sense of themselves and an extreme need for admiration (expect others to notice their special qualities, even when their accomplishments are ordinary, and they enjoy basking in the light of adulation; self-absorbed, lack empathy for others; strong sense of entitlement; tend to be preoccupied with fantasies of success and power, ideal love, or recognition for brilliance or beauty)

Restraints

check every 15 min, remember hydration, nutrition, and elimination

Childhood Problems: School-Age

children express themselves in play.

Alcoholism acute

continuously monitor vitals and urine, watch for OD and withdrawal complications, quiet , safe environment, remove harmful objects, prevent suicide, nonthreatening approach, limit sustained eye contact, institute seizure precautions, administer IV fluids, give meds.

Ego defense mechanism

done to provide feeling of safety and self worth by keeping painful unconscious material out of awareness.

Anorexia Nervosa

dramatic weightless, distorted body image, fear of obesity, anemia, amenorrhea, endocrine dysfunction, electrolyte imbalance, regression, feelings of powerlessness, depression. treatment includes individual, group, family, behavioral modification,.

Crisis treatment

drug therapy: benzodiazepines for short term, crisis intervention directive or supportive. counseling, social referral systems.

Most common side effects of TCAs

dry mouth, constipation, bladder problems, sexual problems, blurred vision

Schizophrenia more nursing

engage patient in reality-oriented activities, administer prescribed meds, monitor for adverse effects and report promptly, encourage patient to comply with medication regiment to prevent relapse. Encourage family to participate in support groups.

Delusions

false fixed belief

Anxiety

feeling of dread or fear in the absence of external threat or disproportionate to the nature of the threat. pounding heart, cold clammy skin, poor concentration. four levels: mild, moderate, severe, panic.

Antisocial Personality Disorder nursing

firm limit setting, confront behaviors consistently, enforce consequences, conduct support/group therapy. straight forward matter of fact approach, reinforce positive behavior, teach social skills, clearly convey expectations and consequences, avoid power struggles/confrontations, encourage to express feelings.

Most common side effects of SSRIs and SNRIs

headache, sexual dysfunction, sleeplessness, drowsiness, agitation

Setting Limits

implemented to prevent and/or respond to unclear or inappropriate behaviors and to maximize the therapeutic experience; clearly conveys expectations, consequences of not meeting those expectations; may be verbal, physical, and/or written

Schizophrenia more nursing

institute suicide precautions, establish trust, do not touch patient without first informing, postpone procedures if suspicious or agitated, use accepting, consistent approach, clear language, asses ability to do ADLs, meet needs, but do for patient only what patient cannot do for self. Reward positive behavior, monitor nutrition, encourage meaningful interpersonal relationships, help learn social skills

Barbiturate Withdrawal

is far MORE severe than opiate withdrawal. Delirium, life-threatening cardiovascular collapse.

Seclusion

isolation from others, solitude

Bipolar Disorder treatment

lithium or particular anticonvulsant drugs (carbamazepine, valproate, or lamotrigine)

Antisocial personality disorder signs/symptoms

long standing disregard of other's rights, repeat unlawful acts, reckless disregard for his or other's safety, deceitfulness, lack of remorse, consistent irresponsibility, power-seeking behavior, impulsivity and failure to plan. manipulative nature, argumentative

Bipolar Disorder nursing

maintain physical health and safety (nutrition, rest, sleep, elimination), monitor for lithium or anticonvulsant toxicity, orient client to reality, limit stimuli, set limits, be consistent, maintain calm environment protect from overstimulation, provide emotional support, set realistic goals, watch for early signs of frustration.

Schizophrenia nursing

maintain safety, administer antipsychotic, decrease sensory stimuli, remove from areas of tension, validate reality, not arguing, recognizing that client is experiencing hallucinations, responding to feeling or tone of hallucination or delusion

Alcoholism when acute over

monitor and promote nutrition, administer drugs carefully to prevent hoarding, warn that even small alcohol when taking disulfiram will induce adverse reactions. refer for rehab, set limits when dealing with demanding, manipulative behavior.

Anorexia Nervosa nursing

monitor vitals, nutritional, I/Os, nutritionally complete liquids during acute phase, establish target weight, negotiate an adequate food intake, allow her to maintain control over types and amounts of foods, weigh patient daily on same scale at same time, 1 lb per week gain, maintain one to one supervision during meals, teach patient to keep food journal and feelings . recognize and assert feelings freely.

Lithium

mood stabilizer. control manic episodes of bipolar. side effects: dizzy, tremor, impaired vision. monitor blood levels 2 to 3 times per week, monitor monthly when maintenance, fluid intake of 2,500 to 3,000 per day, adequate salt. keep taking med, don't drive, check blood levels 8-12 hours after dose. 1-1.5 is target. can take with food.

Venlafaxine contraindications

narrow angle glaucoma, do not use 7 days before taking an MAOI, or do not use until at least 14 days after taking an MAOI

Anxiety types of anxiety disorders

persistent irrational fears of activity, object, situation, desire to avoid or escape. Acrophobia (heights), agoraphobia (open spaces?ok kaplan?), nursing: avoid confrontation and humiliation, do not focus on getting client to stop being afraid, teach relaxation techniques, model unafraid behaviors, assist with cog behavioral techniques such as systematic desensitization.

Conduct Disorder

persistent pattern of behaviors that violate the rights of others and age appropriate social rules. 1. aggression to people and animals, bullies, threatens or intimidates others. IF over eighteen only diagnosed if criteria for antisocial personality disorder is not met.

Conversion Disorder

physical symptoms with no organic basis. lack of concern about symptoms. unconscious. assist with diagnostic evaluation, discuss feelings rather than symptoms, promote therapeutic relationship, avoid supporting secondary gains of client, do not force to talk, seek psychiatric care, use measures to maintain integrity of affected body (exercise "paralyzed limb") convey caring attitude to share feelings

depression nursing care

provide patients physical needs, administer meds, monitor adverse effects, may take several weeks to produce desired effect, recored all observations/convos, plan activities for when patient has energy, structured routines, ease depression by expressing feelings, engage in activities, improve forming and hygiene, write feelings down. Distorted perceptions and link to depression, alert for suicide.

Suicide Prevention

solutions, ask if they have a plan, ask if they have access to a weapon, and if they have a timeframe in mind, ask the person to delay (contract), encourage the person to seek help

Cocaine/Crack Cocaine

stimulant . dilated pupils

Defense Mechanisms 2

sublimation: mental process to change aggressive behaviors into acceptable heavers or activities, uses creative mental focus. Undoing: an attempt by the client to erase an action, may be expressed by excessive apologies. Rationalization: individual makes excuses for one's behavior. Reaction formation: expressing attitude opposite to an unconscious with or fear.

Bipolar Disorder more nursing

tell patient firmly that threats and hitting are unacceptable. alert het health care team promptly when acting out escalates. do not try to do everything yourself, make use of available help. administer med as ordered, monitor for adverse effects

Thiothixene

typical AP+ can cause ocular pigment changes

Schizophrenia treatment

typical antipsychotics like thorazine, atypical like clorazil, zyprexa, risperidone (sedation, hypotension, EPS, anticholinergic, NMS) conjunctive psychotherapy, rehab, ECT.


संबंधित स्टडी सेट्स

Am. Nat. Gov: Inquizitive (Ch. 3)

View Set

Chapter 13 Program Development and Programming Languages

View Set

GA Property and Casualty Guarantee Test Attempt #1

View Set

A History of Global Climate Change

View Set

Chapter 17 - Mental Health_Bipolar Disorders

View Set

FAA Private Pilot Oral Exam "Aeromedical Factors"

View Set

Exam 5: Chapter 15, 16, 17, and 18

View Set